NCLEX Pharmacology 3

¡Supera tus tareas y exámenes ahora con Quizwiz!

195.) A nurse is caring for a client who is taking metoprolol (Lopressor). The nurse measures the client's blood pressure (BP) and apical pulse (AP) immediately before administration. The client's BP is 122/78 mm/Hg and the AP is 58 beats/min. Based on this data, which of the following is the appropriate action? 1. Withhold the medication. 2. Notify the registered nurse immediately. 3. Administer the medication as prescribed. 4. Administer half of the prescribed medication.

1. Withhold the medication. Rationale: Metoprolol (Lopressor) is classified as a beta-adrenergic blocker and is used in the treatment of hypertension, angina, and myocardial infarction. Baseline nursing assessments include measurement of BP and AP immediately before administration. If the systolic BP is below 90 mm/Hg and the AP is below 60 beats/min, the nurse should withhold the medication and document this action. Although the registered nurse should be informed of the client's vital signs, it is not necessary to do so immediately. The medication should not be administered because the data is outside of the prescribed parameters for this medication. The nurse should not administer half of the medication, or alter any dosages at any point in time.

1 mg to ng

1000 ng

Which instruction should be included in the discharge teaching for a patient with a transdermal nitroglycerin patch?

"Apply the patch to a nonhairy, nonfatty area of the upper torso or arms"

Discharge teaching for a patient receiving an antilipemic medication would include

"Continue your exercise program, and maintain a diet high in omega-3 fatty acids."

The patient asks what the difference is between dalteparin (Fragmin) and heparin. What is the nurse's best respons?

"Dalteparin is a low molecular weight heparin that has a more predictable anticoagulant effect."

The nurse evaluates understanding of discharge instructions regarding warfarin (Coumadin) when the patient states

"I should use a soft toothbrush for dental hygiene."

Which statement, if made by your patient, signifies that additional patient teaching regarding antihypertensive treatment is required?

"I will check my blood pressure every day and take my medication when it is over 140/90."

Which statement will indicate to the nurse that the patient understands the discharge instructions regarding cholestyramine (Questran)?

"I will decrease fiber in my diet."

Which statement when made by the patient indicates deficient knowledge regarding warfarin?

"I will increase the dark green leafy vegetables in my diet."

A patient is started on oral anticoagulant therapy while still receiving intravenous heparin. The patient is concerned about risk for bleeding. What is the nurse's best response?

"It is usually takes 4 to 5 days to achieve a full therapeutic effect for warfarin, so the heparin is continued to help prevent blood clots until the warfarin is working up to speed."

Patient teaching regarding sublingual nitroglycerin should include which statement?

"Sit or lie down after you take a nitroglycerin tablet to prevent dizziness."

Which statement would be most appropriate during discharge teaching for a patient receiving transdermal clonidine (Catapres)?

"The patch should be applied to a nonhairy site, and abrupt withdrawal should be avoided."

A patient who is taking sublingual nitroglycerin is complaining of flushing and headaches. What is the nurse's best response?

"These are the most common side effects of nitroglycerin. They should subside with continued use of nitroglycerin."

A patient asks about taking postassium supplements while taking spironolactone (Aldactone). What is the nurse's best response?

"You are on a diuretic that is potassium-sparing, so there is no need for extra potassium."

The patient asks how nitroglcerin should be stored while traveling. What is the nurse's best response?

"it's best to keep it in its original container away from heat and light."

When titrating intravenous nitroglycerin, what does the nurse monitor? Select all that apply

*Blood pressure *Heart rate *Chest pain

Nursing care for a patient receiving alteplase (Activase) would include which action? Select all that apply

*Record vital signs and report changes *Observe for signs and symptoms of bleeding *Assess for cardiac dysrhythmias

Which are common side effects of fenofibrate (Tricor), a fibric acid derivative? Select all that apply

*ausea, vomiting, and abdominal pain *Increase in gallstone formation *Impotence

When teaching a patient about carvedilol (Coreg), the nurse explains that this medication reduces blood pressure by (Select all that apply)

*reducing heart rate *vasodilation

Thrombolytics

-ase

Antifungals

-azole

Local Anesthetic

-caine

Cephalosporin

-cef/ceph

Neuromuscular Blocking Agent

-curium/curonium

Tetracycline

-cycline

Calcium Channel Blockers

-dipine

Fluoroquinolone

-floaxin

Aminoglycoside

-micin/mycin

Beta Blockers

-olol

Antivirals

-ovir/avir

Proton Pump Inhibitor (PPI)

-prazole

ACE Inhibitors

-pril

Angiotensin Receptor Blockades (ARBs)

-sartan

Corticosteroid

-sone

Antilipedemic

-statin

Cholinegergics

-stigmine

Bronchodilator

-terol

Thiazide Diuretics

-thiazide

H2 Histamine Blockers

-tidine

Benzodiazepenes

-zepam

Alpha Adrenergic Blocker

-zosin

anaphylaxis Nursing Considerations

.3ml of epi 1:1000 solution massage site to speed up absorption can repeat in 15-20 minutes

The patient weighs 44 lb and is to be digitalized. The digitazlizing dose is to be 0.03 mg/kg in three divided doses. How much will the nurse administer per dose?

0.2 mg

sucralfate is a gastric protectant and is given

1 hour before meals and at bedtime

1 L to kg

1 kg

the lipid infusion should be connected to TPN solution tubing below the filter at a rate of for first 1/2 hour

1 ml/minute

127.) The nurse provides medication instructions to an older hypertensive client who is taking 20 mg of lisinopril (Prinivil, Zestril) orally daily. The nurse evaluates the need for further teaching when the client states which of the following? 1. "I can skip a dose once a week." 2. "I need to change my position slowly." 3. "I take the pill after breakfast each day." 4. "If I get a bad headache, I should call my doctor immediately."

1. "I can skip a dose once a week." Rationale: Lisinopril is an antihypertensive angiotensin-converting enzyme (ACE) inhibitor. The usual dosage range is 20 to 40 mg per day. Adverse effects include headache, dizziness, fatigue, orthostatic hypotension, tachycardia, and angioedema. Specific client teaching points include taking one pill a day, not stopping the medication without consulting the health care provider (HCP), and monitoring for side effects and adverse reactions. The client should notify the HCP if side effects occur.

688. The nurse is caring for a client with a diagnosis of influenza who first began to experience symptoms yesterday. Antiviral therapy is prescribed and the nurse provides instructions to the client about the therapy. Which statement by the client indicates an understanding of the instructions? 1. "I must take the medication exactly as prescribed." 2. "Once I start the medication, I will no longer be contagious." 3. "I will not get any colds or infections while taking this medication." 4. "This medication has minimal side effects and I can return to normal activities."

1. "I must take the medication exactly as prescribed."

719. A client who is receiving digoxin (Lanoxin) daily has a serum potassium level of 3 mEq/L and is complaining of anorexia. A physician prescribes a digoxin level to rule out digoxin toxicity. A nurse checks the results, knowing that which of the following is the therapeutic serum level (range) for digoxin?

1. 0.5 to 2 ng/mL

719. A client who is receiving digoxin (Lanoxin) daily has a serum potassium level of 3 mEq/L and is complaining of anorexia. A physician prescribes a digoxin level to rule out digoxin toxicity. A nurse checks the results, knowing that which of the following is the therapeutic serum level (range) for digoxin? 1. 0.5 to 2 ng/mL 2. 1.2 to 2.8 ng/mL 3. 3 to 5 ng/mL 4. 3.5 to 5.5 ng/mL

1. 0.5 to 2 ng/mL

220.) A adult client with muscle spasms is taking an oral maintenance dose of baclofen (Lioresal). The nurse reviews the medication record, expecting that which dose should be prescribed? 1. 15 mg four times a day 2. 25 mg four times a day 3. 30 mg four times a day 4. 40 mg four times a day

1. 15 mg four times a day Rationale: Baclofen is dispensed in 10- and 20-mg tablets for oral use. Dosages are low initially and then gradually increased. Maintenance doses range from 15 to 20 mg administered three or four times a day.

A nurse prepares to administer digoxin (Lanoxin) to a 3-year-old with a diagnosis of congestive heart failure and notes that the apical heart rate is 120 beats per minute. Which nursing action is appropriate? 1. Administer the digoxin. 2. Recheck the apical heart rate in 15 minutes. 3. Notify the registered nurse. 4. Hold the medication.

1. Administer the digoxin. Rationale: The normal apical heart rate for a 3-year-old is 80 to 125 beats per minute. Because the apical heart rate is within normal range, options 2, 3, and 4 are inappropriate.

142.) A health care provider has written a prescription for ranitidine (Zantac), once daily. The nurse should schedule the medication for which of the following times? 1. At bedtime 2. After lunch 3. With supper 4. Before breakfast

1. At bedtime Rationale: A single daily dose of ranitidine is usually scheduled to be given at bedtime. This allows for a prolonged effect, and the greatest protection of the gastric mucosa. **recall that ranitidine suppresses secretions of gastric acids**

202.) A nurse is collecting data from a client about medications being taken, and the client tells the nurse that he is taking herbal supplements for the treatment of varicose veins. The nurse understands that the client is most likely taking which of the following? 1. Bilberry 2. Ginseng 3. Feverfew 4. Evening primrose

1. Bilberry Rationale: Bilberry is an herbal supplement that has been used to treat varicose veins. This supplement has also been used to treat cataracts, retinopathy, diabetes mellitus, and peripheral vascular disease. Ginseng has been used to improve memory performance and decrease blood glucose levels in type 2 diabetes mellitus. Feverfew is used to prevent migraine headaches and to treat rheumatoid arthritis. Evening primrose is used to treat eczema and skin irritation.

156.) A nurse is reviewing the laboratory results for a client receiving tacrolimus (Prograf). Which laboratory result would indicate to the nurse that the client is experiencing an adverse effect of the medication? 1. Blood glucose of 200 mg/dL 2. Potassium level of 3.8 mEq/L 3. Platelet count of 300,000 cells/mm3 4. White blood cell count of 6000 cells/mm3

1. Blood glucose of 200 mg/dL Rationale: A blood glucose level of 200 mg/dL is elevated above the normal range of 70 to 110 mg/dL and suggests an adverse effect. Other adverse effects include neurotoxicity evidenced by headache, tremor, insomnia; gastrointestinal (GI) effects such as diarrhea, nausea, and vomiting; hypertension; and hyperkalemia.

689. A client has begun therapy with theophylline (Theo-24). A nurse plans to teach the client to limit the intake of which of the following while taking this medication? 1. Coffee, cola, and chocolate 2. Oysters, lobster, and shrimp 3. Melons, oranges, and pineapple 4. Cottage cheese, cream cheese, and dairy creamers

1. Coffee, cola, and chocolate

32.) Desmopressin acetate (DDAVP) is prescribed for the treatment of diabetes insipidus. The nurse monitors the client after medication administration for which therapeutic response? 1. Decreased urinary output 2. Decreased blood pressure 3. Decreased peripheral edema 4. Decreased blood glucose level

1. Decreased urinary output Rationale: Desmopressin promotes renal conservation of water. The hormone carries out this action by acting on the collecting ducts of the kidney to increase their permeability to water, which results in increased water reabsorption. The therapeutic effect of this medication would be manifested by a decreased urine output. Options 2, 3, and 4 are unrelated to the effects of this medication.

193.) Sodium hypochlorite (Dakin's solution) is prescribed for a client with a leg wound containing purulent drainage. The nurse is assisting in developing a plan of care for the client and includes which of the following in the plan? 1. Ensure that the solution is freshly prepared before use. 2. Soak a sterile dressing with solution and pack into the wound. 3. Allow the solution to remain in the wound following irrigation. 4. Apply the solution to the wound and on normal skin tissue surrounding the wound.

1. Ensure that the solution is freshly prepared before use. Rationale: Dakin solution is a chloride solution that is used for irrigating and cleaning necrotic or purulent wounds. It can be used for packing necrotic wounds. It cannot be used to pack purulent wounds because the solution is inactivated by copious pus. It should not come into contact with healing or normal tissue, and it should be rinsed off immediately if used for irrigation. Solutions are unstable and the nurse must ensure that the solution has been prepared fresh before use. **Eliminate options 2 and 3 first because they are comparable or alike. It makes sense to ensure that the solution is freshly prepared; therefore, select option 1**

957. A hospitalized client is started on phenelzine (Nardil) for the treatment of depression. A nurse instructs the client to avoid consuming which foods while taking this medication? Select all that apply. 1. Figs 2. Yogurt 3. Crackers 4. Aged cheese 5. Tossed salad 6. Oatmeal cookies

1. Figs 2. Yogurt 4. Aged cheese

93.) The client who is human immunodeficiency virus seropositive has been taking stavudine (d4t, Zerit). The nurse monitors which of the following most closely while the client is taking this medication? 1. Gait 2. Appetite 3. Level of consciousness 4. Hemoglobin and hematocrit blood levels

1. Gait Rationale: Stavudine (d4t, Zerit) is an antiretroviral used to manage human immunodeficiency virus infection in clients who do not respond to or who cannot tolerate conventional therapy. The medication can cause peripheral neuropathy, and the nurse should monitor the client's gait closely and ask the client about paresthesia. Options 2, 3, and 4 are unrelated to the use of the medication.

A client with multiple sclerosis is receiving dantrolene (Dantrium) for relief of muscle spasticity. The nurse checks the results of which laboratory value periodically prescribed while the client is taking this medication? 1. Liver function studies 2. Creatinine 3. Blood urea nitrogen 4. Hematocrit level

1. Liver function studies (LFT's) Rationale: Dantrolene can cause liver damage, and the nurse should monitor the results of liver function studies. Baseline liver function studies are done before therapy starts, and regular liver function studies are performed throughout therapy. Dantrolene is discontinued if no relief of spasticity is achieved in 6 weeks.

43.) A histamine (H2)-receptor antagonist will be prescribed for a client. The nurse understands that which medications are H2-receptor antagonists? Select all that apply. 1. Nizatidine (Axid) 2. Ranitidine (Zantac) 3. Famotidine (Pepcid) 4. Cimetidine (Tagamet) 5. Esomeprazole (Nexium) 6. Lansoprazole (Prevacid)

1. Nizatidine (Axid) 2. Ranitidine (Zantac) 3. Famotidine (Pepcid) 4. Cimetidine (Tagamet) Rationale: H2-receptor antagonists suppress secretion of gastric acid, alleviate symptoms of heartburn, and assist in preventing complications of peptic ulcer disease. These medications also suppress gastric acid secretions and are used in active ulcer disease, erosive esophagitis, and pathological hypersecretory conditions. The other medications listed are proton pump inhibitors. H2-receptor antagonists medication names end with -dine. Proton pump inhibitors medication names end with -zole.

109.) A client taking buspirone (BuSpar) for 1 month returns to the clinic for a follow-up visit. Which of the following would indicate medication effectiveness? 1. No rapid heartbeats or anxiety 2. No paranoid thought processes 3. No thought broadcasting or delusions 4. No reports of alcohol withdrawal symptoms

1. No rapid heartbeats or anxiety Rationale: Buspirone hydrochloride is not recommended for the treatment of drug or alcohol withdrawal, paranoid thought disorders, or schizophrenia (thought broadcasting or delusions). Buspirone hydrochloride is most often indicated for the treatment of anxiety and aggression.

171.) A nurse is preparing to administer furosemide (Lasix) to a client with a diagnosis of heart failure. The most important laboratory test result for the nurse to check before administering this medication is: 1. Potassium level 2. Creatinine level 3. Cholesterol level 4. Blood urea nitrogen

1. Potassium level Rationale: Furosemide is a loop diuretic. The medication causes a decrease in the client's electrolytes, especially potassium, sodium, and chloride. Administering furosemide to a client with low electrolyte levels could precipitate ventricular dysrhythmias. Options 2 and 4 reflect renal function. The cholesterol level is unrelated to the administration of this medication.

729. Intravenous heparin therapy is prescribed for a client. While implementing this prescription, a nurse ensures that which of the following medications is available on the nursing unit?

1. Protamine sulfate

729. Intravenous heparin therapy is prescribed for a client. While implementing this prescription, a nurse ensures that which of the following medications is available on the nursing unit? 1. Protamine sulfate 2. Potassium chloride 3. Aminocaproic acid (Amicar) 4. Vitamin K (AquaMEPHYTON)

1. Protamine sulfate

102.) A client with human immunodeficiency virus is taking nevirapine (Viramune). The nurse should monitor for which adverse effects of the medication? Select all that apply. 1. Rash 2. Hepatotoxicity 3. Hyperglycemia 4. Peripheral neuropathy 5. Reduced bone mineral density

1. Rash 2. Hepatotoxicity Rationale: Nevirapine (Viramune) is a non-nucleoside reverse transcriptase inhibitors (NRTI) that is used to treat HIV infection. It is used in combination with other antiretroviral medications to treat HIV. Adverse effects include rash, Stevens-Johnson syndrome, hepatitis, and increased transaminase levels. Hyperglycemia, peripheral neuropathy, and reduced bone density are not adverse effects of this medication.

90.) A nurse is reviewing the record of a client who has been prescribed baclofen (Lioresal). Which of the following disorders, if noted in the client's history, would alert the nurse to contact the health care provider? 1. Seizure disorders 2. Hyperthyroidism 3. Diabetes mellitus 4. Coronary artery disease

1. Seizure disorders Rationale: Clients with seizure disorders may have a lowered seizure threshold when baclofen is administered. Concurrent therapy may require an increase in the anticonvulsive medication. The disorders in options 2, 3, and 4 are not a concern when the client is taking baclofen.

221.) A nurse is reviewing the health care provider's prescriptions for an adult client who has been admitted to the hospital following a back injury. Carisoprodol (Soma) is prescribed for the client to relieve the muscle spasms; the health care provider has prescribed 350 mg to be administered four times a day. When preparing to give this medication, the nurse determines that this dosage is: 1. The normal adult dosage 2. A lower than normal dosage 3. A higher than normal dosage 4. A dosage requiring further clarification

1. The normal adult dosage Rationale: The normal adult dosage for carisoprodol is 350 mg orally three or four times daily.

821. The client arrives at the emergency department complaining of back spasms. The client states, "I have been taking two to three aspirin every 4 hours for the last week, and it hasn't helped my back." Aspirin intoxication is suspected, and the nurse assesses the client for which of the following? 1. Tinnitus 2. Diarrhea 3. Constipation 4. Photosensitivity

1. Tinnitus

21.) A nurse is assisting with caring for a client with cancer who is receiving cisplatin. Select the adverse effects that the nurse monitors for that are associated with this medication. Select all that apply. 1. Tinnitus 2. Ototoxicity 3. Hyperkalemia 4. Hypercalcemia 5. Nephrotoxicity 6. Hypomagnesemia

1. Tinnitus 2. Ototoxicity 5. Nephrotoxicity 6. Hypomagnesemia Rationale: Cisplatin is an alkylating medication. Alkylating medications are cell cycle phase-nonspecific medications that affect the synthesis of DNA by causing the cross-linking of DNA to inhibit cell reproduction. Cisplatin may cause ototoxicity, tinnitus, hypokalemia, hypocalcemia, hypomagnesemia, and nephrotoxicity. Amifostine (Ethyol) may be administered before cisplatin to reduce the potential for renal toxicity.

3.) Salicylic acid is prescribed for a client with a diagnosis of psoriasis. The nurse monitors the client, knowing that which of the following would indicate the presence of systemic toxicity from this medication? 1. Tinnitus 2. Diarrhea 3. Constipation 4. Decreased respirations

1. Tinnitus Rationale: Salicylic acid is absorbed readily through the skin, and systemic toxicity (salicylism) can result. Symptoms include tinnitus, dizziness, hyperpnea, and psychological disturbances. Constipation and diarrhea are not associated with salicylism.

954. A client taking lithium carbonate (Lithobid) reports vomiting, abdominal pain, diarrhea, blurred vision, tinnitus, and tremors. The lithium level is 2.5 mEq/L. The nurse interprets this level as: 1. Toxic 2. Normal 3. Slightly above normal 4. Excessively below normal

1. Toxic

110.) A client taking lithium carbonate (Lithobid) reports vomiting, abdominal pain, diarrhea, blurred vision, tinnitus, and tremors. The lithium level is checked as a part of the routine follow-up and the level is 3.0 mEq/L. The nurse knows that this level is: 1. Toxic 2. Normal 3. Slightly above normal 4. Excessively below normal

1. Toxic Rationale: The therapeutic serum level of lithium is 0.6 to 1.2 mEq/L. A level of 3 mEq/L indicates toxicity.

8.) A client with severe acne is seen in the clinic and the health care provider (HCP) prescribes isotretinoin. The nurse reviews the client's medication record and would contact the (HCP) if the client is taking which medication? 1. Vitamin A 2. Digoxin (Lanoxin) 3. Furosemide (Lasix) 4. Phenytoin (Dilantin)

1. Vitamin A Rationale: Isotretinoin is a metabolite of vitamin A and can produce generalized intensification of isotretinoin toxicity. Because of the potential for increased toxicity, vitamin A supplements should be discontinued before isotretinoin therapy. Options 2, 3, and 4 are not contraindicated with the use of isotretinoin.

239.) Which of the following precautions will the nurse specifically take during the administration of ribavirin (Virazole) to a child with respiratory syncytial virus (RSV)? 1. Wearing goggles 2. Wearing a gown 3. Wearing a gown and a mask 4. Handwashing before administration

1. Wearing goggles Rationale: Some caregivers experience headaches, burning nasal passages and eyes, and crystallization of soft contact lenses as a result of administration of ribavirin. Specific to this medication is the use of goggles. A gown is not necessary. A mask may be worn. Handwashing is to be performed before and after any child contact.

79.) Ibuprofen (Advil) is prescribed for a client. The nurse tells the client to take the medication: 1. With 8 oz of milk 2. In the morning after arising 3. 60 minutes before breakfast 4. At bedtime on an empty stomach

1. With 8 oz of milk Rationale: Ibuprofen is a nonsteroidal anti-inflammatory drug (NSAID). NSAIDs should be given with milk or food to prevent gastrointestinal irritation. Options 2, 3, and 4 are incorrect.

24.) A nurse is reinforcing teaching for a client regarding how to mix regular insulin and NPH insulin in the same syringe. Which of the following actions, if performed by the client, indicates the need for further teaching? 1. Withdraws the NPH insulin first 2. Withdraws the regular insulin first 3. Injects air into NPH insulin vial first 4. Injects an amount of air equal to the desired dose of insulin into the vial

1. Withdraws the NPH insulin first Rationale: When preparing a mixture of regular insulin with another insulin preparation, the regular insulin is drawn into the syringe first. This sequence will avoid contaminating the vial of regular insulin with insulin of another type. Options 2, 3, and 4 identify the correct actions for preparing NPH and regular insulin.

234.) A hospitalized client is started on phenelzine sulfate (Nardil) for the treatment of depression. At lunchtime, a tray is delivered to the client. Which food item on the tray will the nurse remove? 1. Yogurt 2. Crackers 3. Tossed salad 4. Oatmeal cookies

1. Yogurt Rationale: Phenelzine sulfate is a monoamine oxidase inhibitor (MAOI). The client should avoid taking in foods that are high in tyramine. These foods could trigger a potentially fatal hypertensive crisis. Foods to avoid include yogurt, aged cheeses, smoked or processed meats, red wines, and fruits such as avocados, raisins, or figs.

lidocaine (Xylocaine)

1.5-6.0

6.) The burn client is receiving treatments of topical mafenide acetate (Sulfamylon) to the site of injury. The nurse monitors the client, knowing that which of the following indicates that a systemic effect has occurred? 1.Hyperventilation 2.Elevated blood pressure 3.Local pain at the burn site 4.Local rash at the burn site

1.Hyperventilation Rationale: Mafenide acetate is a carbonic anhydrase inhibitor and can suppress renal excretion of acid, thereby causing acidosis. Clients receiving this treatment should be monitored for signs of an acid-base imbalance (hyperventilation). If this occurs, the medication should be discontinued for 1 to 2 days. Options 3 and 4 describe local rather than systemic effects. An elevated blood pressure may be expected from the pain that occurs with a burn injury.

what is the maximum % of dextrose used for a PPN

10% maximum of dextrose to avoid vein irritation

phenytoin (Dilantin)

10-20

theophylline (Theo-Dur)

10-20

salicylate (Aspirin)

100-200 (>200 is toxic)

1g to mg

1000 mg

tbsp to ml

15 ml

what size and type of catheter is used for acentral line TPN

18G triple lumen

A patient presents to the emergency room with insecticide poisoning. The nurse prepares to administer which dose of atropine intravenously?

2 mg

quinidine (Quinalgute)

2-6

824. The nurse has given medication instructions to the client receiving phenytoin (Dilantin). The nurse determines that the client has an adequate understanding if the client states that: 1. "Alcohol is not contraindicated while taking this medication." 2. "Good oral hygiene is needed, including brushing and flossing." 3. "The medication dose may be self-adjusted, depending on side effects." 4. "The morning dose of the medication should be taken before a serum drug level is drawn."

2. "Good oral hygiene is needed, including brushing and flossing."

78.) A client is taking phenytoin (Dilantin) for seizure control and a sample for a serum drug level is drawn. Which of the following indicates a therapeutic serum drug range? 1. 5 to 10 mcg/mL 2. 10 to 20 mcg/mL 3. 20 to 30 mcg/mL 4. 30 to 40 mcg/mL

2. 10 to 20 mcg/mL Rationale: The therapeutic serum drug level range for phenytoin (Dilantin) is 10 to 20 mcg/mL. ** A helpful hint may be to remember that the theophylline therapeutic range and the acetaminophen (Tylenol) therapeutic range are the same as the phenytoin (Dilantin) therapeutic range.**

215.) A client with rheumatoid arthritis is taking acetylsalicylic acid (aspirin) on a daily basis. Which medication dose should the nurse expect the client to be taking? 1. 1 g daily 2. 4 g daily 3. 325 mg daily 4. 1000 mg daily

2. 4 g daily Rationale: Aspirin may be used to treat the client with rheumatoid arthritis. It may also be used to reduce the risk of recurrent transient ischemic attack (TIA) or brain attack (stroke) or reduce the risk of myocardial infarction (MI) in clients with unstable angina or a history of a previous MI. The normal dose for clients being treated with aspirin to decrease thrombosis and MI is 300 to 325 mg/day. Clients being treated to prevent TIAs are usually prescribed 1.3 g/day in two to four divided doses. Clients with rheumatoid arthritis are treated with 3.6 to 5.4 g/day in divided doses. **Eliminate options 1 and 4 because they are alike**

64.) Nalidixic acid (NegGram) is prescribed for a client with a urinary tract infection. On review of the client's record, the nurse notes that the client is taking warfarin sodium (Coumadin) daily. Which prescription should the nurse anticipate for this client? 1. Discontinuation of warfarin sodium (Coumadin) 2. A decrease in the warfarin sodium (Coumadin) dosage 3. An increase in the warfarin sodium (Coumadin) dosage 4. A decrease in the usual dose of nalidixic acid (NegGram)

2. A decrease in the warfarin sodium (Coumadin) dosage Rationale: Nalidixic acid can intensify the effects of oral anticoagulants by displacing these agents from binding sites on plasma protein. When an oral anticoagulant is combined with nalidixic acid, a decrease in the anticoagulant dosage may be needed.

A client with atrial fibrillation is receiving a continuous heparin infusion at 1000 units/hr. The nurse would determine that the client is receiving the therapeutic effect based on which of the following results?

2. Activated partial thromboplastin time of 60 seconds

88.) Dantrolene sodium (Dantrium) is prescribed for a client experiencing flexor spasms, and the client asks the nurse about the action of the medication. The nurse responds, knowing that the therapeutic action of this medication is which of the following? 1. Depresses spinal reflexes 2. Acts directly on the skeletal muscle to relieve spasticity 3. Acts within the spinal cord to suppress hyperactive reflexes 4. Acts on the central nervous system (CNS) to suppress spasms

2. Acts directly on the skeletal muscle to relieve spasticity Rationale: Dantrium acts directly on skeletal muscle to relieve muscle spasticity. The primary action is the suppression of calcium release from the sarcoplasmic reticulum. This in turn decreases the ability of the skeletal muscle to contract. **Options 1, 3, and 4 are all comparable or alike in that they address CNS suppression and the depression of reflexes. Therefore, eliminate these options.**

9.) The nurse is applying a topical corticosteroid to a client with eczema. The nurse would monitor for the potential for increased systemic absorption of the medication if the medication were being applied to which of the following body areas? 1. Back 2. Axilla 3. Soles of the feet 4. Palms of the hands

2. Axilla Rationale: Topical corticosteroids can be absorbed into the systemic circulation. Absorption is higher from regions where the skin is especially permeable (scalp, axilla, face, eyelids, neck, perineum, genitalia), and lower from regions in which permeability is poor (back, palms, soles).

A client has been given a prescription for gemfibrozil (Lopid). The nurse plans to instruct the client to limit intake of which of the following foods while taking this medication? 1. Fish 2. Beef 3. Spicy foods 4. Citrus products

2. Beef Rationale: Gemfibrozil is a lipid-lowering agent. It is given as part of a therapeutic regimen that also includes dietary counseling, specifically, the limitation of saturated and other fats in the diet. Therefore the intake of red meats is limited. Fish, foods that are spicy, and citrus products will not affect the cholesterol level.

224.) Neuroleptic malignant syndrome is suspected in a client who is taking chlorpromazine. Which medication would the nurse prepare in anticipation of being prescribed to treat this adverse effect related to the use of chlorpromazine? 1. Protamine sulfate 2. Bromocriptine (Parlodel) 3. Phytonadione (vitamin K) 4. Enalapril maleate (Vasotec)

2. Bromocriptine (Parlodel) Rationale: Bromocriptine is an antiparkinsonian prolactin inhibitor used in the treatment of neuroleptic malignant syndrome. Vitamin K is the antidote for warfarin (Coumadin) overdose. Protamine sulfate is the antidote for heparin overdose. Enalapril maleate is an antihypertensive used in the treatment of hypertension.

20.) The client with metastatic breast cancer is receiving tamoxifen. The nurse specifically monitors which laboratory value while the client is taking this medication? 1. Glucose level 2. Calcium level 3. Potassium level 4. Prothrombin time

2. Calcium level Rationale: Tamoxifen may increase calcium, cholesterol, and triglyceride levels. Before the initiation of therapy, a complete blood count, platelet count, and serum calcium levels should be assessed. These blood levels, along with cholesterol and triglyceride levels, should be monitored periodically during therapy. The nurse should assess for hypercalcemia while the client is taking this medication. Signs of hypercalcemia include increased urine volume, excessive thirst, nausea, vomiting, constipation, hypotonicity of muscles, and deep bone and flank pain.

34.) A client with Crohn's disease is scheduled to receive an infusion of infliximab (Remicade). The nurse assisting in caring for the client should take which action to monitor the effectiveness of treatment? 1. Monitoring the leukocyte count for 2 days after the infusion 2. Checking the frequency and consistency of bowel movements 3. Checking serum liver enzyme levels before and after the infusion 4. Carrying out a Hematest on gastric fluids after the infusion is completed

2. Checking the frequency and consistency of bowel movements Rationale: The principal manifestations of Crohn's disease are diarrhea and abdominal pain. Infliximab (Remicade) is an immunomodulator that reduces the degree of inflammation in the colon, thereby reducing the diarrhea. Options 1, 3, and 4 are unrelated to this medication.

84.) Baclofen (Lioresal) is prescribed for the client with multiple sclerosis. The nurse assists in planning care, knowing that the primary therapeutic effect of this medication is which of the following? 1. Increased muscle tone 2. Decreased muscle spasms 3. Increased range of motion 4. Decreased local pain and tenderness

2. Decreased muscle spasms Rationale: Baclofen is a skeletal muscle relaxant and central nervous system depressant and acts at the spinal cord level to decrease the frequency and amplitude of muscle spasms in clients with spinal cord injuries or diseases and in clients with multiple sclerosis. Options 1, 3, and 4 are incorrect.

732. The nurse is monitoring a client who is taking digoxin (Lanoxin) for adverse effects. Which findings are characteristic of digoxin toxicity. Select all that apply. 1. Tremors 2. Diarrhea 3. Irritability 4. Blurred vision 5. Nausea and vomiting

2. Diarrhea 4. Blurred vision 5. Nausea and vomiting

209.) A client with multiple sclerosis is receiving diazepam (Valium), a centrally acting skeletal muscle relaxant. Which of the following would indicate that the client is experiencing a side effect related to this medication? 1. Headache 2. Drowsiness 3. Urinary retention 4. Increased salivation

2. Drowsiness Rationale: Incoordination and drowsiness are common side effects resulting from this medication. Options 1, 3, and 4 are incorrect.

240.) A client with Parkinson's disease has been prescribed benztropine (Cogentin). The nurse monitors for which gastrointestinal (GI) side effect of this medication? 1. Diarrhea 2. Dry mouth 3. Increased appetite 4. Hyperactive bowel sounds

2. Dry mouth Rationale: Common GI side effects of benztropine therapy include constipation and dry mouth. Other GI side effects include nausea and ileus. These effects are the result of the anticholinergic properties of the medication. **Eliminate options 1 and 4 because they are comparable or alike. Recall that the medication is an anticholinergic, which causes dry mouth**

952. A nurse is performing a follow-up teaching session with a client discharged 1 month ago. The client is taking fluoxetine (Prozac). What information would be important for the nurse to obtain during this client visit regarding the side effects of the medication? 1. Cardiovascular symptoms 2. Gastrointestinal dysfunctions 3. Problems with mouth dryness 4. Problems with excessive sweating

2. Gastrointestinal dysfunctions

108.) A nurse is performing a follow-up teaching session with a client discharged 1 month ago who is taking fluoxetine (Prozac). What information would be important for the nurse to gather regarding the adverse effects related to the medication? 1. Cardiovascular symptoms 2. Gastrointestinal dysfunctions 3. Problems with mouth dryness 4. Problems with excessive sweating

2. Gastrointestinal dysfunctions Rationale: The most common adverse effects related to fluoxetine include central nervous system (CNS) and gastrointestinal (GI) system dysfunction. This medication affects the GI system by causing nausea and vomiting, cramping, and diarrhea. Options 1, 3, and 4 are not adverse effects of this medication.

136.) A nurse performs an admission assessment on a client who visits a health care clinic for the first time. The client tells the nurse that propylthiouracil (PTU) is taken daily. The nurse continues to collect data from the client, suspecting that the client has a history of: 1. Myxedema 2. Graves' disease 3. Addison's disease 4. Cushing's syndrome

2. Graves' disease Rationale: PTU inhibits thyroid hormone synthesis and is used to treat hyperthyroidism, or Graves' disease. Myxedema indicates hypothyroidism. Cushing's syndrome and Addison's disease are disorders related to adrenal function.

41.) The client has been taking omeprazole (Prilosec) for 4 weeks. The ambulatory care nurse evaluates that the client is receiving optimal intended effect of the medication if the client reports the absence of which symptom? 1. Diarrhea 2. Heartburn 3. Flatulence 4. Constipation

2. Heartburn Rationale: Omeprazole is a proton pump inhibitor classified as an antiulcer agent. The intended effect of the medication is relief of pain from gastric irritation, often called heartburn by clients. Omeprazole is not used to treat the conditions identified in options 1, 3, and 4.

947. A nurse is describing the medication side effects to a client who is taking oxazepam (Serax). The nurse incorporates in discussions with the client the need to: 1. Consume a low-fiber diet. 2. Increase fluids and bulk in the diet. 3. Rest if the heart begins to beat rapidly. 4. Take antidiarrheal agents if diarrhea occurs.

2. Increase fluids and bulk in the diet.

686. A client with tuberculosis is being started on antituberculosis therapy with isoniazid (INH). Before giving the client the first dose, a nurse ensures that which of the following baseline studies has been completed? 1. Electrolyte levels 2. Liver enzyme levels 3. Serum creatinine level 4. Coagulation times

2. Liver enzyme levels

823. The nurse is caring for a client with severe back pain. Codeine sulfate has been prescribed for the client. Which of the following does the nurse specifically include in the plan of care while the client is taking this medication? 1. Monitor fluid balance. 2. Monitor bowel activity. 3. Monitor peripheral pulses. 4. Monitor for hypertension.

2. Monitor bowel activity.

949. A nurse is teaching a client who is being started on imipramine (Tofranil) about the medication. The nurse informs the client that the maximum desired effects may: 1. Start during the first week of administration 2. Not occur for 2 to 3 weeks of administration 3. Start during the second week of administration 4. Not occur until after 2 months of administration

2. Not occur for 2 to 3 weeks of administration

227.) When teaching a client who is being started on imipramine hydrochloride (Tofranil), the nurse would inform the client that the desired effects of the medication may: 1. Start during the first week of administration 2. Not occur for 2 to 3 weeks of administration 3. Start during the second week of administration 4. Not occur until after a month of administration

2. Not occur for 2 to 3 weeks of administration Rationale: The therapeutic effects of administration of imipramine hydrochloride may not occur for 2 to 3 weeks after the antidepressant therapy has been initiated. Therefore options 1, 3, and 4 are incorrect.

169.) Insulin glargine (Lantus) is prescribed for a client with diabetes mellitus. The nurse tells the client that it is best to take the insulin: 1. 1 hour after each meal 2. Once daily, at the same time each day 3. 15 minutes before breakfast, lunch, and dinner 4. Before each meal, on the basis of the blood glucose level

2. Once daily, at the same time each day Rationale: Insulin glargine is a long-acting recombinant DNA human insulin used to treat type 1 and type 2 diabetes mellitus. It has a 24-hour duration of action and is administered once a day, at the same time each day.

47.) A client has been taking isoniazid (INH) for 2 months. The client complains to a nurse about numbness, paresthesias, and tingling in the extremities. The nurse interprets that the client is experiencing: 1. Hypercalcemia 2. Peripheral neuritis 3. Small blood vessel spasm 4. Impaired peripheral circulation

2. Peripheral neuritis Rationale: A common side effect of the TB drug INH is peripheral neuritis. This is manifested by numbness, tingling, and paresthesias in the extremities. This side effect can be minimized by pyridoxine (vitamin B6) intake. Options 1, 3, and 4 are incorrect.

50.) A nurse has given a client taking ethambutol (Myambutol) information about the medication. The nurse determines that the client understands the instructions if the client states that he or she will immediately report: 1. Impaired sense of hearing 2. Problems with visual acuity 3. Gastrointestinal (GI) side effects 4. Orange-red discoloration of body secretions

2. Problems with visual acuity Rationale: Ethambutol causes optic neuritis, which decreases visual acuity and the ability to discriminate between the colors red and green. This poses a potential safety hazard when a client is driving a motor vehicle. The client is taught to report this symptom immediately. The client is also taught to take the medication with food if GI upset occurs. Impaired hearing results from antitubercular therapy with streptomycin. Orange-red discoloration of secretions occurs with rifampin (Rifadin).

186.) A nurse prepares to administer sodium polystyrene sulfonate (Kayexalate) to a client. Before administering the medication, the nurse reviews the action of the medication and understands that it: 1. Releases bicarbonate in exchange for primarily sodium ions 2. Releases sodium ions in exchange for primarily potassium ions 3. Releases potassium ions in exchange for primarily sodium ions 4. Releases sodium ions in exchange for primarily bicarbonate ions

2. Releases sodium ions in exchange for primarily potassium ions Rationale: Sodium polystyrene sulfonate is a cation exchange resin used in the treatment of hyperkalemia. The resin either passes through the intestine or is retained in the colon. It releases sodium ions in exchange for primarily potassium ions. The therapeutic effect occurs 2 to 12 hours after oral administration and longer after rectal administration.

168.) Colcrys (colchicine) is prescribed for a client with a diagnosis of gout. The nurse reviews the client's medical history in the health record, knowing that the medication would be contraindicated in which disorder? 1. Myxedema 2. Renal failure 3. Hypothyroidism 4. Diabetes mellitus

2. Renal failure Rationale: Colchicine is contraindicated in clients with severe gastrointestinal, renal, hepatic or cardiac disorders, or with blood dyscrasias. Clients with impaired renal function may exhibit myopathy and neuropathy manifested as generalized weakness. This medication should be used with caution in clients with impaired hepatic function, older clients, and debilitated clients. **Note that options 1, 3, and 4 are all endocrine-related disorders: Myxedema=Hypothyroidism**

683. A client is to begin a 6-month course of therapy with isoniazid (INH). A nurse plans to teach the client to: 1. Use alcohol in small amounts only. 2. Report yellow eyes or skin immediately. 3. Increase intake of Swiss or aged cheeses. 4. Avoid vitamin supplements during therapy.

2. Report yellow eyes or skin immediately.

23.) A client who has been newly diagnosed with diabetes mellitus has been stabilized with daily insulin injections. Which information should the nurse teach when carrying out plans for discharge? 1. Keep insulin vials refrigerated at all times. 2. Rotate the insulin injection sites systematically. 3. Increase the amount of insulin before unusual exercise. 4. Monitor the urine acetone level to determine the insulin dosage.

2. Rotate the insulin injection sites systematically. Rationale: Insulin dosages should not be adjusted or increased before unusual exercise. If acetone is found in the urine, it may possibly indicate the need for additional insulin. To minimize the discomfort associated with insulin injections, the insulin should be administered at room temperature. Injection sites should be systematically rotated from one area to another. The client should be instructed to give injections in one area, about 1 inch apart, until the whole area has been used and then to change to another site. This prevents dramatic changes in daily insulin absorption.

687. A nurse has an order to give a client salmeterol (Serevent Diskus), two puffs, and beclomethasone dipropionate (Qvar), two puffs, by metered-dose inhaler. The nurse administers the medication by giving the: 1. Beclomethasone first and then the salmeterol 2. Salmeterol first and then the beclomethasone 3. Alternating a single puff of each, beginning with the salmeterol 4. Alternating a single puff of each, beginning with the beclomethasone

2. Salmeterol first and then the beclomethasone

216.) A nurse is caring for a client with gout who is taking Colcrys (colchicine). The client has been instructed to restrict the diet to low-purine foods. Which of the following foods should the nurse instruct the client to avoid while taking this medication? 1. Spinach 2. Scallops 3. Potatoes 4. Ice cream

2. Scallops Rationale: Colchicine is a medication used for clients with gout to inhibit the reabsorption of uric acid by the kidney and promote excretion of uric acid in the urine. Uric acid is produced when purine is catabolized. Clients are instructed to modify their diet and limit excessive purine intake. High-purine foods to avoid or limit include organ meats, roe, sardines, scallops, anchovies, broth, mincemeat, herring, shrimp, mackerel, gravy, and yeast.

956. A hospitalized client has begun taking bupropion (Wellbutrin) as an antidepressant agent. A nurse monitors this client for which side effect indicating that the client is taking an excessive amount of medication? 1. Constipation 2. Seizure activity 3. Increased weight 4. Dizziness when getting upright

2. Seizure activity

114.) A postoperative client requests medication for flatulence (gas pains). Which medication from the following PRN list should the nurse administer to this client? 1. Ondansetron (Zofran) 2. Simethicone (Mylicon) 3. Acetaminophen (Tylenol) 4. Magnesium hydroxide (milk of magnesia, MOM)

2. Simethicone (Mylicon) Rationale: Simethicone is an antiflatulent used in the relief of pain caused by excessive gas in the gastrointestinal tract. Ondansetron is used to treat postoperative nausea and vomiting. Acetaminophen is a nonopioid analgesic. Magnesium hydroxide is an antacid and laxative.

172.) A nurse provides dietary instructions to a client who will be taking warfarin sodium (Coumadin). The nurse tells the client to avoid which food item? 1. Grapes 2. Spinach 3. Watermelon 4. Cottage cheese

2. Spinach Rationale: Warfarin sodium is an anticoagulant. Anticoagulant medications act by antagonizing the action of vitamin K, which is needed for clotting. When a client is taking an anticoagulant, foods high in vitamin K often are omitted from the diet. Vitamin K-rich foods include green, leafy vegetables, fish, liver, coffee, and tea.

730. A client is receiving thrombolytic therapy with a continuous infusion of streptokinase (Streptase). The client suddenly becomes extremely anxious and complains of itching. A nurse hears stridor and on examination of the client notes generalized urticaria and hypotension. Which of the following should be the priority action of the nurse?

2. Stop the infusion and call the physician.

45.) A client has a prescription to take guaifenesin (Humibid) every 4 hours, as needed. The nurse determines that the client understands the most effective use of this medication if the client states that he or she will: 1. Watch for irritability as a side effect. 2. Take the tablet with a full glass of water. 3. Take an extra dose if the cough is accompanied by fever. 4. Crush the sustained-release tablet if immediate relief is needed.

2. Take the tablet with a full glass of water. Rationale: Guaifenesin is an expectorant. It should be taken with a full glass of water to decrease viscosity of secretions. Sustained-release preparations should not be broken open, crushed, or chewed. The medication may occasionally cause dizziness, headache, or drowsiness as side effects. The client should contact the health care provider if the cough lasts longer than 1 week or is accompanied by fever, rash, sore throat, or persistent headache.

951. A nurse notes that a client with schizophrenia and receiving an antipsychotic medication is moving her mouth, protruding her tongue, and grimacing as she watches television. The nurse determines that the client is experiencing: 1. Parkinsonism 2. Tardive dyskinesia 3. Hypertensive crisis 4. Neuroleptic malignant syndrome

2. Tardive dyskinesia

57.) A nurse is monitoring a client who is taking propranolol (Inderal LA). Which data collection finding would indicate a potential serious complication associated with propranolol? 1. The development of complaints of insomnia 2. The development of audible expiratory wheezes 3. A baseline blood pressure of 150/80 mm Hg followed by a blood pressure of 138/72 mm Hg after two doses of the medication 4. A baseline resting heart rate of 88 beats/min followed by a resting heart rate of 72 beats/min after two doses of the medication

2. The development of audible expiratory wheezes Rationale: Audible expiratory wheezes may indicate a serious adverse reaction, bronchospasm. β-Blockers may induce this reaction, particularly in clients with chronic obstructive pulmonary disease or asthma. Normal decreases in blood pressure and heart rate are expected. Insomnia is a frequent mild side effect and should be monitored.

126.) A nurse is caring for a client who has been prescribed furosemide (Lasix) and is monitoring for adverse effects associated with this medication. Which of the following should the nurse recognize as a potential adverse effect Select all that apply. 1. Nausea 2. Tinnitus 3. Hypotension 4. Hypokalemia 5. Photosensitivity 6. Increased urinary frequency

2. Tinnitus 3. Hypotension 4. Hypokalemia Rationale: Furosemide is a loop diuretic; therefore, an expected effect is increased urinary frequency. Nausea is a frequent side effect, not an adverse effect. Photosensitivity is an occasional side effect. Adverse effects include tinnitus (ototoxicity), hypotension, and hypokalemia and occur as a result of sudden volume depletion.

51.) Cycloserine (Seromycin) is added to the medication regimen for a client with tuberculosis. Which of the following would the nurse include in the client-teaching plan regarding this medication? 1. To take the medication before meals 2. To return to the clinic weekly for serum drug-level testing 3. It is not necessary to call the health care provider (HCP) if a skin rash occurs. 4. It is not necessary to restrict alcohol intake with this medication.

2. To return to the clinic weekly for serum drug-level testing Rationale: Cycloserine (Seromycin) is an antitubercular medication that requires weekly serum drug level determinations to monitor for the potential of neurotoxicity. Serum drug levels lower than 30 mcg/mL reduce the incidence of neurotoxicity. The medication must be taken after meals to prevent gastrointestinal irritation. The client must be instructed to notify the HCP if a skin rash or signs of central nervous system toxicity are noted. Alcohol must be avoided because it increases the risk of seizure activity.

826. Meperidine hydrochloride (Demerol) has been prescribed for a client to treat pain. Select the side effects of this medication. Select all that apply. 1. Diarrhea 2. Tremors 3. Drowsiness 4. Hypotension 5. Urinary frequency 6. Increased respiratory rate

2. Tremors 3. Drowsiness 4. Hypotension

82.) A client is receiving meperidine hydrochloride (Demerol) for pain. Which of the following are side effects of this medication. Select all that apply. 1. Diarrhea 2. Tremors 3. Drowsiness 4. Hypotension 5. Urinary frequency 6. Increased respiratory rate

2. Tremors 3. Drowsiness 4. Hypotension Rationale: Meperidine hydrochloride is an opioid analgesic. Side effects include respiratory depression, drowsiness, hypotension, constipation, urinary retention, nausea, vomiting, and tremors.

135.) A nurse reinforces medication instructions to a client who is taking levothyroxine (Synthroid). The nurse instructs the client to notify the health care provider (HCP) if which of the following occurs? 1. Fatigue 2. Tremors 3. Cold intolerance 4. Excessively dry skin

2. Tremors Rationale: Excessive doses of levothyroxine (Synthroid) can produce signs and symptoms of hyperthyroidism. These include tachycardia, chest pain, tremors, nervousness, insomnia, hyperthermia, heat intolerance, and sweating. The client should be instructed to notify the HCP if these occur. Options 1, 3, and 4 are signs of hypothyroidism.

A client is taking trihexyphenidyl for the treatment of Parkinson's disease. The nurse would monitor for which adverse effect of this medication? 1. Diarrhea 2. Urinary retention 3. Urinary incontinence 4. Excessive perspiration

2. Urinary retention Rationale: Trihexyphenidyl is an anticholinergic medication. Because of this, it can cause urinary hesitancy and retention, constipation, dry mouth, and decreased sweating as side effects.

238.) Ribavirin (Virazole) is prescribed for the hospitalized child with respiratory syncytial virus (RSV). The nurse prepares to administer this medication via which of the following routes? 1. Orally 2. Via face mask 3. Intravenously 4. Intramuscularly

2. Via face mask Rationale: Ribavirin is an antiviral respiratory medication used mainly in hospitalized children with severe RSV and in high-risk children. Administration is via hood, face mask, or oxygen tent. The medication is most effective if administered within the first 3 days of the infection.

1 kg to lb

2.2 lb

1 cup (8 oz) to ml

240 ml

the appropriate size needle for intradermal injection

27 gauge 1/2 inch needle

106.) Fluoxetine (Prozac) is prescribed for the client. The nurse reinforces instructions to the client regarding the administration of the medication. Which statement by the client indicates an understanding about administration of the medication? 1. "I should take the medication with my evening meal." 2. "I should take the medication at noon with an antacid." 3. "I should take the medication in the morning when I first arise." 4. "I should take the medication right before bedtime with a snack."

3. "I should take the medication in the morning when I first arise." Rationale: Fluoxetine hydrochloride is administered in the early morning without consideration to meals. **Eliminate options 1, 2, and 4 because they are comparable or alike and indicate taking the medication with an antacid or food.**

197.) Collagenase (Santyl) is prescribed for a client with a severe burn to the hand. The nurse provides instructions to the client regarding the use of the medication. Which statement by the client indicates an accurate understanding of the use of this medication? 1. "I will apply the ointment once a day and leave it open to the air." 2. "I will apply the ointment twice a day and leave it open to the air." 3. "I will apply the ointment once a day and cover it with a sterile dressing." 4. "I will apply the ointment at bedtime and in the morning and cover it with a sterile dressing."

3. "I will apply the ointment once a day and cover it with a sterile dressing." Rationale: Collagenase is used to promote debridement of dermal lesions and severe burns. It is usually applied once daily and covered with a sterile dressing.

164.) A client receives a prescription for methocarbamol (Robaxin), and the nurse reinforces instructions to the client regarding the medication. Which client statement would indicate a need for further instructions? 1. "My urine may turn brown or green." 2. "This medication is prescribed to help relieve my muscle spasms." 3. "If my vision becomes blurred, I don't need to be concerned about it." 4. "I need to call my doctor if I experience nasal congestion from this medication."

3. "If my vision becomes blurred, I don't need to be concerned about it." Rationale: The client needs to be told that the urine may turn brown, black, or green. Other adverse effects include blurred vision, nasal congestion, urticaria, and rash. The client needs to be instructed that, if these adverse effects occur, the health care provider needs to be notified. The medication is used to relieve muscle spasms.

42.) A client with a peptic ulcer is diagnosed with a Helicobacter pylori infection. The nurse is reinforcing teaching for the client about the medications prescribed, including clarithromycin (Biaxin), esomeprazole (Nexium), and amoxicillin (Amoxil). Which statement by the client indicates the best understanding of the medication regimen? 1. "My ulcer will heal because these medications will kill the bacteria." 2. "These medications are only taken when I have pain from my ulcer." 3. "The medications will kill the bacteria and stop the acid production." 4. "These medications will coat the ulcer and decrease the acid production in my stomach."

3. "The medications will kill the bacteria and stop the acid production." Rationale: Triple therapy for Helicobacter pylori infection usually includes two antibacterial drugs and a proton pump inhibitor. Clarithromycin and amoxicillin are antibacterials. Esomeprazole is a proton pump inhibitor. These medications will kill the bacteria and decrease acid production.

141.) The nurse has reinforced instructions to a client who has been prescribed cholestyramine (Questran). Which statement by the client indicates a need for further instructions? 1. "I will continue taking vitamin supplements." 2. "This medication will help lower my cholesterol." 3. "This medication should only be taken with water." 4. "A high-fiber diet is important while taking this medication."

3. "This medication should only be taken with water." Rationale: Cholestyramine (Questran) is a bile acid sequestrant used to lower the cholesterol level, and client compliance is a problem because of its taste and palatability. The use of flavored products or fruit juices can improve the taste. Some side effects of bile acid sequestrants include constipation and decreased vitamin absorption. **Note the closed-ended word "only" in option 3**

A nurse is caring for a client with myasthenia gravis who has received edrophonium (Enlon) intravenously to test for myasthenic crisis. The client asks the nurse how long the improvement in muscle strength will last. The nurse's response is based on the understanding that the effects have a duration of approximately: 1. 5 minutes 2. 15 minutes 3. 30 minutes 4. 60 minutes

3. 30 minutes Rationale: Edrophonium may be given to test for myasthenic crisis. If the client is in myasthenic crisis, muscle strength improves after administration of the medication and lasts for about 30 minutes.

120.) A client is taking lansoprazole (Prevacid) for the chronic management of Zollinger-Ellison syndrome. The nurse advises the client to take which of the following products if needed for a headache? 1. Naprosyn (Aleve) 2. Ibuprofen (Advil) 3. Acetaminophen (Tylenol) 4. Acetylsalicylic acid (aspirin)

3. Acetaminophen (Tylenol) Rationale: Zollinger-Ellison syndrome is a hypersecretory condition of the stomach. The client should avoid taking medications that are irritating to the stomach lining. Irritants would include aspirin and nonsteroidal antiinflammatory drugs (ibuprofen). The client should be advised to take acetaminophen for headache. **Remember that options that are comparable or alike are not likely to be correct. With this in mind, eliminate options 1 and 2 first.**

35.) The client has a PRN prescription for loperamide hydrochloride (Imodium). The nurse understands that this medication is used for which condition? 1. Constipation 2. Abdominal pain 3. An episode of diarrhea 4. Hematest-positive nasogastric tube drainage

3. An episode of diarrhea Rationale: Loperamide is an antidiarrheal agent. It is used to manage acute and also chronic diarrhea in conditions such as inflammatory bowel disease. Loperamide also can be used to reduce the volume of drainage from an ileostomy. It is not used for the conditions in options 1, 2, and 4.

139.) Prednisone is prescribed for a client with diabetes mellitus who is taking Humulin neutral protamine Hagedorn (NPH) insulin daily. Which of the following prescription changes does the nurse anticipate during therapy with the prednisone? 1. An additional dose of prednisone daily 2. A decreased amount of daily Humulin NPH insulin 3. An increased amount of daily Humulin NPH insulin 4. The addition of an oral hypoglycemic medication daily

3. An increased amount of daily Humulin NPH insulin Rationale: Glucocorticoids can elevate blood glucose levels. Clients with diabetes mellitus may need their dosages of insulin or oral hypoglycemic medications increased during glucocorticoid therapy. Therefore the other options are incorrect.

107.) A client receiving a tricyclic antidepressant arrives at the mental health clinic. Which observation indicates that the client is correctly following the medication plan? 1. Reports not going to work for this past week 2. Complains of not being able to "do anything" anymore 3. Arrives at the clinic neat and appropriate in appearance 4. Reports sleeping 12 hours per night and 3 to 4 hours during the day

3. Arrives at the clinic neat and appropriate in appearance Rationale: Depressed individuals will sleep for long periods, are not able to go to work, and feel as if they cannot "do anything." Once they have had some therapeutic effect from their medication, they will report resolution of many of these complaints as well as demonstrate an improvement in their appearance.

69.) A nurse who is administering bethanechol chloride (Urecholine) is monitoring for acute toxicity associated with the medication. The nurse checks the client for which sign of toxicity? 1. Dry skin 2. Dry mouth 3. Bradycardia 4. Signs of dehydration

3. Bradycardia Rationale: Toxicity (overdose) produces manifestations of excessive muscarinic stimulation such as salivation, sweating, involuntary urination and defecation, bradycardia, and severe hypotension. Treatment includes supportive measures and the administration of atropine sulfate subcutaneously or intravenously.

49.) A client has been started on long-term therapy with rifampin (Rifadin). A nurse teaches the client that the medication: 1. Should always be taken with food or antacids 2. Should be double-dosed if one dose is forgotten 3. Causes orange discoloration of sweat, tears, urine, and feces 4. May be discontinued independently if symptoms are gone in 3 months

3. Causes orange discoloration of sweat, tears, urine, and feces Rationale: Rifampin should be taken exactly as directed as part of TB therapy. Doses should not be doubled or skipped. The client should not stop therapy until directed to do so by a health care provider. The medication should be administered on an empty stomach unless it causes gastrointestinal upset, and then it may be taken with food. Antacids, if prescribed, should be taken at least 1 hour before the medication. Rifampin causes orange-red discoloration of body secretions and will permanently stain soft contact lenses.

38.) An older client recently has been taking cimetidine (Tagamet). The nurse monitors the client for which most frequent central nervous system side effect of this medication? 1. Tremors 2. Dizziness 3. Confusion 4. Hallucinations

3. Confusion Rationale: Cimetidine is a histamine 2 (H2)-receptor antagonist. Older clients are especially susceptible to central nervous system side effects of cimetidine. The most frequent of these is confusion. Less common central nervous system side effects include headache, dizziness, drowsiness, and hallucinations.

16.) The clinic nurse is reviewing a teaching plan for the client receiving an antineoplastic medication. When implementing the plan, the nurse tells the client: 1. To take aspirin (acetylsalicylic acid) as needed for headache 2. Drink beverages containing alcohol in moderate amounts each evening 3. Consult with health care providers (HCPs) before receiving immunizations 4. That it is not necessary to consult HCPs before receiving a flu vaccine at the local health fair

3. Consult with health care providers (HCPs) before receiving immunizations Rationale: Because antineoplastic medications lower the resistance of the body, clients must be informed not to receive immunizations without a HCP's approval. Clients also need to avoid contact with individuals who have recently received a live virus vaccine. Clients need to avoid aspirin and aspirin-containing products to minimize the risk of bleeding, and they need to avoid alcohol to minimize the risk of toxicity and side effects.

819. The client with trigeminal neuralgia tells the nurse that acetaminophen (Tylenol) is taken daily for the relief of generalized discomfort. Which laboratory value would indicate toxicity associated with the medication? 1. Sodium level of 140 mEq/L 2. Prothrombin time of 12 seconds 3. Direct bilirubin level of 2 mg/dL 4. Platelet count of 400,000/mm3

3. Direct bilirubin level of 2 mg/dL

727. A 66-year-old client complaining of not feeling well is seen in a clinic. The client is taking several medications for the control of heart disease and hypertension. These medications include atenolol (Tenormin), digoxin (Lanoxin), and chlorothiazide (Diuril). A tentative diagnosis of digoxin toxicity is made. Which of the following assessment data would support this diagnosis?

3. Double vision, loss of appetite, and nausea

727. A 66-year-old client complaining of not feeling well is seen in a clinic. The client is taking several medications for the control of heart disease and hypertension. These medications include atenolol (Tenormin), digoxin (Lanoxin), and chlorothiazide (Diuril). A tentative diagnosis of digoxin toxicity is made. Which of the following assessment data would support this diagnosis? 1. Dyspnea, edema, and palpitations 2. Chest pain, hypotension, and paresthesia 3. Double vision, loss of appetite, and nausea 4. Constipation, dry mouth, and sleep disorder

3. Double vision, loss of appetite, and nausea

85.) A nurse is monitoring a client receiving baclofen (Lioresal) for side effects related to the medication. Which of the following would indicate that the client is experiencing a side effect? 1. Polyuria 2. Diarrhea 3. Drowsiness 4. Muscular excitability

3. Drowsiness Rationale: Baclofen is a central nervous system (CNS) depressant and frequently causes drowsiness, dizziness, weakness, and fatigue. It can also cause nausea, constipation, and urinary retention. Clients should be warned about the possible reactions. Options 1, 2, and 4 are not side effects.

138.) A daily dose of prednisone is prescribed for a client. A nurse reinforces instructions to the client regarding administration of the medication and instructs the client that the best time to take this medication is: 1. At noon 2. At bedtime 3. Early morning 4. Anytime, at the same time, each day

3. Early morning Rationale: Corticosteroids (glucocorticoids) should be administered before 9:00 AM. Administration at this time helps minimize adrenal insufficiency and mimics the burst of glucocorticoids released naturally by the adrenal glands each morning. **Note the suffix "-sone," and recall that medication names that end with these letters are corticosteroids.**

71.) After kidney transplantation, cyclosporine (Sand immune) is prescribed for a client. Which laboratory result would indicate an adverse effect from the use of this medication? 1. Decreased creatinine level 2. Decreased hemoglobin level 3. Elevated blood urea nitrogen level 4. Decreased white blood cell count

3. Elevated blood urea nitrogen level Rationale: Nephrotoxicity can occur from the use of cyclosporine (Sandimmune). Nephrotoxicity is evaluated by monitoring for elevated blood urea nitrogen (BUN) and serum creatinine levels. Cyclosporine is an immunosuppressant but does not depress the bone marrow.

244.) A client has a prescription for valproic acid (Depakene) orally once daily. The nurse plans to: 1. Administer the medication with an antacid. 2. Administer the medication with a carbonated beverage. 3. Ensure that the medication is administered at the same time each day. 4. Ensure that the medication is administered 2 hours before breakfast only, when the client's stomach is empty.

3. Ensure that the medication is administered at the same time each day. Rationale: Valproic acid is an anticonvulsant, antimanic, and antimigraine medication. It may be administered with or without food. It should not be taken with an antacid or carbonated beverage because these products will affect medication absorption. The medication is administered at the same time each day to maintain therapeutic serum levels. **Use general pharmacology guidelines to assist in eliminating options 1 and 2. Eliminate option 4 because of the closed-ended word "only."**

154.) A nurse is reinforcing dietary instructions to a client who has been prescribed cyclosporine (Sandimmune). Which food item would the nurse instruct the client to avoid? 1. Red meats 2. Orange juice 3. Grapefruit juice 4. Green, leafy vegetables

3. Grapefruit juice Rationale: A compound present in grapefruit juice inhibits metabolism of cyclosporine. As a result, the consumption of grapefruit juice can raise cyclosporine levels by 50% to 100%, thereby greatly increasing the risk of toxicity. Grapefruit juice needs to be avoided. Red meats, orange juice, and green leafy vegetables are acceptable to consume.

97.) Amikacin (Amikin) is prescribed for a client with a bacterial infection. The client is instructed to contact the health care provider (HCP) immediately if which of the following occurs? 1. Nausea 2. Lethargy 3. Hearing loss 4. Muscle aches

3. Hearing loss Rationale: Amikacin (Amikin) is an aminoglycoside. Adverse effects of aminoglycosides include ototoxicity (hearing problems), confusion, disorientation, gastrointestinal irritation, palpitations, blood pressure changes, nephrotoxicity, and hypersensitivity. The nurse instructs the client to report hearing loss to the HCP immediately. Lethargy and muscle aches are not associated with the use of this medication. It is not necessary to contact the HCP immediately if nausea occurs. If nausea persists or results in vomiting, the HCP should be notified. **(most aminoglycoside medication names end in the letters -cin)**

724. A nurse is planning to administer hydrochlorothiazide (HydroDIURIL) to a client. The nurse understands that which of the following are concerns related to the administration of this medication?

3. Hypokalemia, hyperglycemia, sulfa allergy

724. A nurse is planning to administer hydrochlorothiazide (HydroDIURIL) to a client. The nurse understands that which of the following are concerns related to the administration of this medication? 1. Hypouricemia, hyperkalemia 2. Increased risk of osteoporosis 3. Hypokalemia, hyperglycemia, sulfa allergy 4. Hyperkalemia, hypoglycemia, penicillin allergy

3. Hypokalemia, hyperglycemia, sulfa allergy

A client is receiving diazepam (Valium) for its skeletal muscle relaxant effects. The nurse should monitor this client for which side effect of this medication? 1. Urinary retention 2. Headache 3. Incoordination 4. Increased salivation

3. Incoordination Rationale: Diazepam is a centrally acting skeletal muscle relaxant. Incoordination and drowsiness are common side effects resulting from this medication. The other options are incorrect.

5.) Mafenide acetate (Sulfamylon) is prescribed for the client with a burn injury. When applying the medication, the client complains of local discomfort and burning. Which of the following is the most appropriate nursing action? 1. Notifying the registered nurse 2. Discontinuing the medication 3. Informing the client that this is normal 4. Applying a thinner film than prescribed to the burn site

3. Informing the client that this is normal Rationale: Mafenide acetate is bacteriostatic for gram-negative and gram-positive organisms and is used to treat burns to reduce bacteria present in avascular tissues. The client should be informed that the medication will cause local discomfort and burning and that this is a normal reaction; therefore options 1, 2, and 4 are incorrect

243.) A hospitalized client is having the dosage of clonazepam (Klonopin) adjusted. The nurse should plan to: 1. Weigh the client daily. 2. Observe for ecchymosis. 3. Institute seizure precautions. 4. Monitor blood glucose levels.

3. Institute seizure precautions. Rationale: Clonazepam is a benzodiazepine used as an anticonvulsant. During initial therapy and during periods of dosage adjustment, the nurse should initiate seizure precautions for the client. Options 1, 2, and 4 are not associated with the use of this medication.

A nurse has provided instructions to a client regarding the method for instilling eye drops into the left eye. The nurse determines that the client needs further instruction if the client does which of the following during a return demonstration? 1. Lies with the head to one side, puts the drop in the inner canthus, and slowly turns to the other side while blinking 2. Lies supine, pulls down on the lower lid, and puts the drop in the lower lid 3. Lies supine, pulls up on the upper lid, and puts the drop in the upper lid 4. Tilts the head back, pulls down on the lower lid, and puts the drop in the lower lid

3. Lies supine, pulls up on the upper lid, and puts the drop in the upper lid Rationale: It is correct procedure for the client to either lie down or sit with the head tilted back. The thumb or finger is used to pull down on the lower lid. The client holds the bottle like a pencil (tip facing downward) and squeezes the bottle so that the drop falls into the sac. The client then gently closes the eye. An alternative method for clients who blink very easily is to place the client in the supine position with the head turned to one side. The eye to be used is uppermost. The client squeezes the drop onto the inner canthus. The client turns from this side to the other while blinking. Surface tension and gravity then cause the drop to move into the conjunctival sac.

681. Zafirlukast (Accolate) is prescribed for a client with bronchial asthma. Which laboratory test does the nurse expect to be prescribed before the administration of this medication? 1. Platelet count 2. Neutrophil count 3. Liver function tests 4. Complete blood count

3. Liver function tests

A nurse is caring for a client with glaucoma who is receiving acetazolamide (Diamox Sequels) daily. Which of the following indicates to the nurse that the client is experiencing an adverse effect related to the medication? 1. Diarrhea 2. Lacrimation 3. Low back pain and dysuria 4. Irritability

3. Low back pain and dysuria Rationale: Acetazolamide is a carbonic anhydrase inhibitor. Nephrotoxicity and hepatotoxicity can occur and are manifested by dark urine and stools, jaundice, pain in the lower back, dysuria, crystalluria, renal colic, and calculi. Bone marrow depression also may occur. The remaining options are not adverse effects of the medication.

723. A client is diagnosed with an acute myocardial infarction and is receiving tissue plasminogen activator, alteplase (Activase, tPA). Which of the following is a priority nursing intervention? 1. Monitor for renal failure. 2. Monitor psychosocial status. 3. Monitor for signs of bleeding. 4. Have heparin sodium available.

3. Monitor for signs of bleeding

37.) The client has begun medication therapy with pancrelipase (Pancrease MT). The nurse evaluates that the medication is having the optimal intended benefit if which effect is observed? 1. Weight loss 2. Relief of heartburn 3. Reduction of steatorrhea 4. Absence of abdominal pain

3. Reduction of steatorrhea Rationale: Pancrelipase (Pancrease MT) is a pancreatic enzyme used in clients with pancreatitis as a digestive aid. The medication should reduce the amount of fatty stools (steatorrhea). Another intended effect could be improved nutritional status. It is not used to treat abdominal pain or heartburn. Its use could result in weight gain but should not result in weight loss if it is aiding in digestion.

111.) A client arrives at the health care clinic and tells the nurse that he has been doubling his daily dosage of bupropion hydrochloride (Wellbutrin) to help him get better faster. The nurse understands that the client is now at risk for which of the following? 1. Insomnia 2. Weight gain 3. Seizure activity 4. Orthostatic hypotension

3. Seizure activity Rationale: Bupropion does not cause significant orthostatic blood pressure changes. Seizure activity is common in dosages greater than 450 mg daily. Bupropion frequently causes a drop in body weight. Insomnia is a side effect, but seizure activity causes a greater client risk.

98.) The nurse is assigned to care for a client with cytomegalovirus retinitis and acquired immunodeficiency syndrome who is receiving foscarnet. The nurse should check the latest results of which of the following laboratory studies while the client is taking this medication? 1. CD4 cell count 2. Serum albumin 3. Serum creatinine 4. Lymphocyte count

3. Serum creatinine Rationale: Foscarnet is toxic to the kidneys. Serum creatinine is monitored before therapy, two to three times per week during induction therapy, and at least weekly during maintenance therapy. Foscarnet may also cause decreased levels of calcium, magnesium, phosphorus, and potassium. Thus these levels are also measured with the same frequency.

820. The client is taking the prescribed dose of phenytoin (Dilantin) to control seizures. Results of a phenytoin blood level study reveal a level of 35 mcg/mL. Which of the following symptoms would be expected as a result of this laboratory result? 1. Hypotension 2. Tachycardia 3. Slurred speech 4. No symptoms, because this is a normal therapeutic level

3. Slurred speech

211.) A client with epilepsy is taking the prescribed dose of phenytoin (Dilantin) to control seizures. A phenytoin blood level is drawn, and the results reveal a level of 35 mcg/ml. Which of the following symptoms would be expected as a result of this laboratory result? 1. Nystagmus 2. Tachycardia 3. Slurred speech 4. No symptoms, because this is a normal therapeutic level

3. Slurred speech Rationale: The therapeutic phenytoin level is 10 to 20 mcg/mL. At a level higher than 20 mcg/mL, involuntary movements of the eyeballs (nystagmus) appear. At a level higher than 30 mcg/mL, ataxia and slurred speech occur.

187.) A clinic nurse prepares to administer an MMR (measles, mumps, rubella) vaccine to a child. How is this vaccine best administered? 1. Intramuscularly in the deltoid muscle 2. Subcutaneously in the gluteal muscle 3. Subcutaneously in the outer aspect of the upper arm 4. Intramuscularly in the anterolateral aspect of the thigh

3. Subcutaneously in the outer aspect of the upper arm Rationale: The MMR vaccine is administered subcutaneously in the outer aspect of the upper arm. The gluteal muscle is most often used for intramuscular injections. The MMR vaccine is not administered by the intramuscular route.

NCLEX book chapter 59 676. A client has a prescription to take guaifenesin (Mucinex). The nurse determines that the client understands the proper administration of this medication if the client states that he or she will: 1. Take an extra dose if fever develops. 2. Take the medication with meals only. 3. Take the tablet with a full glass of water. 4. Decrease the amount of daily fluid intake.

3. Take the tablet with a full glass of water.

245.) A client taking carbamazepine (Tegretol) asks the nurse what to do if he misses one dose. The nurse responds that the carbamazepine should be: 1. Withheld until the next scheduled dose 2. Withheld and the health care provider is notified immediately 3. Taken as long as it is not immediately before the next dose 4. Withheld until the next scheduled dose, which should then be doubled

3. Taken as long as it is not immediately before the next dose Rationale: Carbamazepine is an anticonvulsant that should be taken around the clock, precisely as directed. If a dose is omitted, the client should take the dose as soon as it is remembered, as long as it is not immediately before the next dose. The medication should not be double dosed. If more than one dose is omitted, the client should call the health care provider.

229.) A client who is taking lithium carbonate (Lithobid) is scheduled for surgery. The nurse informs the client that: 1. The medication will be discontinued a week before the surgery and resumed 1 week postoperatively. 2. The medication is to be taken until the day of surgery and resumed by injection immediately postoperatively. 3. The medication will be discontinued 1 to 2 days before the surgery and resumed as soon as full oral intake is allowed. 4. The medication will be discontinued several days before surgery and resumed by injection in the immediate postoperative period.

3. The medication will be discontinued 1 to 2 days before the surgery and resumed as soon as full oral intake is allowed. Rationale: The client who is on lithium carbonate must be off the medication for 1 to 2 days before a scheduled surgical procedure and can resume the medication when full oral intake is prescribed after the surgery. **lithium carbonate is an oral medication and is not given as an injection**

817. The home health nurse visits a client who is taking phenytoin (Dilantin) for control of seizures. During the assessment, the nurse notes that the client is taking birth control pills. Which of the following information should the nurse include in the teaching plan? 1. Pregnancy should be avoided while taking phenytoin. 2. The client may stop the medication if it is causing severe gastrointestinal effects. 3. There is the potential of decreased effectiveness of birth control pills while taking phenytoin. 4. There is the increased risk of thrombophlebitis while taking phenytoin and birth control pills together.

3. There is the potential of decreased effectiveness of birth control pills while taking phenytoin.

129.) Megestrol acetate (Megace), an antineoplastic medication, is prescribed for the client with metastatic endometrial carcinoma. The nurse reviews the client's history and contacts the registered nurse if which diagnosis is documented in the client's history? 1. Gout 2. Asthma 3. Thrombophlebitis 4. Myocardial infarction

3. Thrombophlebitis Rationale: Megestrol acetate (Megace) suppresses the release of luteinizing hormone from the anterior pituitary by inhibiting pituitary function and regressing tumor size. Megestrol is used with caution if the client has a history of thrombophlebitis. **megestrol acetate is a hormonal antagonist enzyme and that a side effect is thrombotic disorders**

22.) A nurse is caring for a client after thyroidectomy and notes that calcium gluconate is prescribed for the client. The nurse determines that this medication has been prescribed to: 1. Treat thyroid storm. 2. Prevent cardiac irritability. 3. Treat hypocalcemic tetany. 4. Stimulate the release of parathyroid hormone.

3. Treat hypocalcemic tetany. Rationale: Hypocalcemia can develop after thyroidectomy if the parathyroid glands are accidentally removed or injured during surgery. Manifestations develop 1 to 7 days after surgery. If the client develops numbness and tingling around the mouth, fingertips, or toes or muscle spasms or twitching, the health care provider is notified immediately. Calcium gluconate should be kept at the bedside.

27.) Sildenafil (Viagra) is prescribed to treat a client with erectile dysfunction. A nurse reviews the client's medical record and would question the prescription if which of the following is noted in the client's history? 1. Neuralgia 2. Insomnia 3. Use of nitroglycerin 4. Use of multivitamins

3. Use of nitroglycerin Rationale: Sildenafil (Viagra) enhances the vasodilating effect of nitric oxide in the corpus cavernosum of the penis, thus sustaining an erection. Because of the effect of the medication, it is contraindicated with concurrent use of organic nitrates and nitroglycerin. Sildenafil is not contraindicated with the use of vitamins. Neuralgia and insomnia are side effects of the medication.

945. A client with schizophrenia has been started on medication therapy with clozapine (Clozaril). A nurse assesses the results of which laboratory study to monitor for adverse effects from this medication? 1. Platelet count 2. Blood glucose level 3. White blood cell count 4. Liver function studies

3. White blood cell count

81.) A client with trigeminal neuralgia is being treated with carbamazepine (Tegretol). Which laboratory result would indicate that the client is experiencing an adverse reaction to the medication? 1. Sodium level, 140 mEq/L 2. Uric acid level, 5.0 mg/dL 3. White blood cell count, 3000 cells/mm3 4. Blood urea nitrogen (BUN) level, 15 mg/dL

3. White blood cell count, 3000 cells/mm3 Rationale: Adverse effects of carbamazepine (Tegretol) appear as blood dyscrasias, including aplastic anemia, agranulocytosis, thrombocytopenia, leukopenia, cardiovascular disturbances, thrombophlebitis, dysrhythmias, and dermatological effects. Options 1, 2, and 4 identify normal laboratory values.

Your patient is to receive dopamine 5 mcg/kg/min. He weighs 176 lb. You have dopamine 400 mg in 500 mL D5W. You will infuse this at

30 mL/hr.

1 oz to ml

30 ml

vancomycin (Vancocin)

30-40 (peak), 5-10 (trough)

1 soda (12 oz) to ml

360 ml

3. A client is taking benzphetamine. The nurse teaches the client which information about this drug? a. That it may cause drowsiness b. That it may lead to hypotension c. That it is a respiratory stimulant d. That it is safe during pregnancy 4. The nurse monitoring a client for methylphenidate withdrawal should observe the client for which condition? a. Tremors b. Insomnia d. Tachycardia

3= b. That it may lead to hypotension 4= c. Weakness

When preparing to administer furosemide (Lasix) intravenously to a patient with renal dysfunction, the nurse plans implementation based on knowledge that the medication should be administered no faster than which rate?

4 mg/min

Which would be the most appropriate application time for a patient prescribed a scopolamine patch for motion sickness?

4 to 5 hours before travel

carbamezepine (Tegretol)

4-12

procainamide (Pronestyl)

4-8

54.) A nurse reinforces discharge instructions to a postoperative client who is taking warfarin sodium (Coumadin). Which statement, if made by the client, reflects the need for further teaching? 1. "I will take my pills every day at the same time." 2. "I will be certain to avoid alcohol consumption." 3. "I have already called my family to pick up a Medic-Alert bracelet." 4. "I will take Ecotrin (enteric-coated aspirin) for my headaches because it is coated."

4. "I will take Ecotrin (enteric-coated aspirin) for my headaches because it is coated." Rationale: Ecotrin is an aspirin-containing product and should be avoided. Alcohol consumption should be avoided by a client taking warfarin sodium. Taking prescribed medication at the same time each day increases client compliance. The Medic-Alert bracelet provides health care personnel emergency information.

725. A home health care nurse is visiting a client with elevated triglyceride levels and a serum cholesterol level of 398 mg/dL. The client is taking cholestyramine (Questran). Which of the following statements, if made by the client, indicates the need for further education?

4. "I'll continue my nicotinic acid from the health food store."

726. A client is on nicotinic acid (niacin) for hyperlipidemia and the nurse provides instructions to the client about the medication. Which statement by the client would indicate an understanding of the instructions?

4. "Ibuprofen (Motrin) taken 30 minutes before the nicotinic acid should decrease the flushing."

726. A client is on nicotinic acid (niacin) for hyperlipidemia and the nurse provides instructions to the client about the medication. Which statement by the client would indicate an understanding of the instructions? 1. "It is not necessary to avoid the use of alcohol." 2. "The medication should be taken with meals to decrease flushing." 3. "Clay-colored stools are a common side effect and should not be of concern." 4. "Ibuprofen (Motrin) taken 30 minutes before the nicotinic acid should decrease the flushing."

4. "Ibuprofen (Motrin) taken 30 minutes before the nicotinic acid should decrease the flushing."

690. The nurse has just administered the first dose of omalizumab (Xolair) to a client. Which statement by the client would alert the nurse that the client may be experiencing a life threatening adverse reaction? 1. "I have a severe headache." 2. "My feet are quite swollen." 3. "I am nauseated and may vomit." 4. "My lips and tongue are swollen."

4. "My lips and tongue are swollen."

198.) Coal tar has been prescribed for a client with a diagnosis of psoriasis, and the nurse provides instructions to the client about the medication. Which statement by the client indicates a need for further instructions? 1. "The medication can cause phototoxicity." 2. "The medication has an unpleasant odor." 3. "The medication can stain the skin and hair." 4. "The medication can cause systemic effects."

4. "The medication can cause systemic effects." Rationale: Coal tar is used to treat psoriasis and other chronic disorders of the skin. It suppresses DNA synthesis, mitotic activity, and cell proliferation. It has an unpleasant odor, can frequently stain the skin and hair, and can cause phototoxicity. Systemic toxicity does not occur. **The name of the medication will assist in eliminating options 2 and 3**

A client with Parkinson's disease has begun therapy with carbidopa/levodopa (Sinemet). The nurse determines that the client understands the action of the medication if the client verbalizes that results may not be apparent for: 1. 24 hours 2. 1 week 3. 2 to 3 days 4. 2 to 3 weeks

4. 2 to 3 weeks Rationale: Signs and symptoms of Parkinson's disease usually begin to resolve within 2 to 3 weeks of starting therapy, although in some clients marked improvement may not be seen for up to 6 months. Clients need to understand this concept to aid in compliance with medication therapy. Options 1, 2, and 3 are incorrect because of the short time frames.

825. The client with myasthenia gravis has become increasingly weaker. The physician prepares to identify whether the client is reacting to an overdose of the medication (cholinergic crisis) or an increasing severity of the disease (myasthenic crisis). An injection of edrophonium (Tensilon) is administered. Which of the following would indicate that the client is in cholinergic crisis? 1. No change in the condition 2. Complaints of muscle spasms 3. An improvement of the weakness 4. A temporary worsening of the condition

4. A temporary worsening of the condition

818. The nurse is caring for a client in the emergency department diagnosed with Bell's palsy. The client has been taking acetaminophen (Tylenol), and acetaminophen overdose is suspected. The nurse anticipates that the antidote to be prescribed is: 1. Pentostatin (Nipent) 2. Auranofin (Ridaura) 3. Fludarabine (Fludara) 4. Acetylcysteine (Mucomyst)

4. Acetylcysteine (Mucomyst)

181.) A client is taking ticlopidine hydrochloride (Ticlid). The nurse tells the client to avoid which of the following while taking this medication? 1. Vitamin C 2. Vitamin D 3. Acetaminophen (Tylenol) 4. Acetylsalicylic acid (aspirin)

4. Acetylsalicylic acid (aspirin) Rationale: Ticlopidine hydrochloride is a platelet aggregation inhibitor. It is used to decrease the risk of thrombotic strokes in clients with precursor symptoms. Because it is an antiplatelet agent, other medications that precipitate or aggravate bleeding should be avoided during its use. Therefore, aspirin or any aspirin-containing product should be avoided.

56.) Heparin sodium is prescribed for the client. The nurse expects that the health care provider will prescribe which of the following to monitor for a therapeutic effect of the medication? 1. Hematocrit level 2. Hemoglobin level 3. Prothrombin time (PT) 4. Activated partial thromboplastin time (aPTT)

4. Activated partial thromboplastin time (aPTT) Rationale: The PT will assess for the therapeutic effect of warfarin sodium (Coumadin) and the aPTT will assess the therapeutic effect of heparin sodium. Heparin sodium doses are determined based on these laboratory results. The hemoglobin and hematocrit values assess red blood cell concentrations.

720. A client is being treated with procainamide (Procanbid) for a cardiac dysrhythmia. Following intravenous administration of the medication, the client complains of dizziness. What intervention should the nurse take first?

4. Auscultate the client's apical pulse and obtain a blood pressure.

720. A client is being treated with procainamide (Procanbid) for a cardiac dysrhythmia. Following intravenous administration of the medication, the client complains of dizziness. What intervention should the nurse take first? 1. Administer prescribed nitroglycerin tablets. 2. Measure the heart rate on the rhythm strip. 3. Obtain a 12-lead electrocardiogram immediately. 4. Auscultate the client's apical pulse and obtain a blood pressure.

4. Auscultate the client's apical pulse and obtain a blood pressure.

679. A cromolyn sodium (Intal) inhaler is prescribed for a client with allergic asthma. A nurse provides instructions regarding the side effects of this medication. The nurse tells that client that which undesirable effect is associated with this medication? 1. Insomnia 2. Constipation 3. Hypotension 4. Bronchospasm

4. Bronchospasm

19.) Tamoxifen is prescribed for the client with metastatic breast carcinoma. The nurse understands that the primary action of this medication is to: 1. Increase DNA and RNA synthesis. 2. Promote the biosynthesis of nucleic acids. 3. Increase estrogen concentration and estrogen response. 4. Compete with estradiol for binding to estrogen in tissues containing high concentrations of receptors.

4. Compete with estradiol for binding to estrogen in tissues containing high concentrations of receptors. Rationale: Tamoxifen is an antineoplastic medication that competes with estradiol for binding to estrogen in tissues containing high concentrations of receptors. Tamoxifen is used to treat metastatic breast carcinoma in women and men. Tamoxifen is also effective in delaying the recurrence of cancer following mastectomy. Tamoxifen reduces DNA synthesis and estrogen response.

217.) A health care provider prescribes auranofin (Ridaura) for a client with rheumatoid arthritis. Which of the following would indicate to the nurse that the client is experiencing toxicity related to the medication? 1. Joint pain 2. Constipation 3. Ringing in the ears 4. Complaints of a metallic taste in the mouth

4. Complaints of a metallic taste in the mouth Rationale: Ridaura is the one gold preparation that is given orally rather than by injection. Gastrointestinal reactions including diarrhea, abdominal pain, nausea, and loss of appetite are common early in therapy, but these usually subside in the first 3 months of therapy. Early symptoms of toxicity include a rash, purple blotches, pruritus, mouth lesions, and a metallic taste in the mouth.

94.) The client with acquired immunodeficiency syndrome has begun therapy with zidovudine (Retrovir, Azidothymidine, AZT, ZDV). The nurse carefully monitors which of the following laboratory results during treatment with this medication? 1. Blood culture 2. Blood glucose level 3. Blood urea nitrogen 4. Complete blood count

4. Complete blood count Rationale: A common side effect of therapy with zidovudine is leukopenia and anemia. The nurse monitors the complete blood count results for these changes. Options 1, 2, and 3 are unrelated to the use of this medication.

124.) A client with chronic renal failure is receiving ferrous sulfate (Feosol). The nurse monitors the client for which common side effect associated with this medication? 1. Diarrhea 2. Weakness 3. Headache 4. Constipation

4. Constipation Rationale: Feosol is an iron supplement used to treat anemia. Constipation is a frequent and uncomfortable side effect associated with the administration of oral iron supplements. Stool softeners are often prescribed to prevent constipation. **Focus on the name of the medication. Recalling that oral iron can cause constipation will easily direct you to the correct option.**

A client with vascular headaches is taking ergotamine (Ergomar). The nurse would monitor the client for: 1. Constipation 2. Hypotension 3. Dependent edema 4. Cool, numb fingers and toes

4. Cool, numb fingers and toes Rationale: Ergotamine produces vasoconstriction, which suppresses vascular headaches when given at a therapeutic dose range. The nurse monitors for hypertension; cool, numb fingers and toes; muscle pain; and nausea and vomiting. **first recall that vascular headaches are caused by vasodilatation of the blood vessels in the head. Following this train of thought, you then recall that this medication must cause vasoconstriction. The only side effect consistent with vasoconstriction is option 4, the cool, numb fingers and toes.**

157.) A client receiving nitrofurantoin (Macrodantin) calls the health care provider's office complaining of side effects related to the medication. Which side effect indicates the need to stop treatment with this medication? 1. Nausea 2. Diarrhea 3. Anorexia 4. Cough and chest pain

4. Cough and chest pain Rationale: Gastrointestinal (GI) effects are the most frequent adverse reactions to this medication and can be minimized by administering the medication with milk or meals. Pulmonary reactions, manifested as dyspnea, chest pain, chills, fever, cough, and the presence of alveolar infiltrates on the x-ray, would indicate the need to stop the treatment. These symptoms resolve in 2 to 4 days following discontinuation of this medication. **Eliminate options 1, 2, and 3 because they are similar GI-related side effects. Also, use the ABCs— airway, breathing, and circulation**

132.) The client with non-Hodgkin's lymphoma is receiving daunorubicin (DaunoXome). Which of the following would indicate to the nurse that the client is experiencing a toxic effect related to the medication? 1. Fever 2. Diarrhea 3. Complaints of nausea and vomiting 4. Crackles on auscultation of the lungs

4. Crackles on auscultation of the lungs Rationale: Cardiotoxicity noted by abnormal electrocardiographic findings or cardiomyopathy manifested as congestive heart failure is a toxic effect of daunorubicin. Bone marrow depression is also a toxic effect. Nausea and vomiting are frequent side effects associated with the medication that begins a few hours after administration and lasts 24 to 48 hours. Fever is a frequent side effect, and diarrhea can occur occasionally. The other options, however, are not toxic effects. **keep in mind that the question is asking about a toxic effect and think: ABCs—airway, breathing, and circulation**

685. A nurse has given a client taking ethambutol (Myambutol) information about the medication. The nurse determines that the client understands the instructions if the client states to report immediately: 1. Impaired sense of hearing 2. Gastrointestinal side effects 3. Orange-red discoloration of body secretions 4. Difficulty in discriminating the color red from green

4. Difficulty in discriminating the color red from green

203.) A nurse is preparing to give the postcraniotomy client medication for incisional pain. The family asks the nurse why the client is receiving codeine sulfate and not "something stronger." In formulating a response, the nurse incorporates the understanding that codeine: 1. Is one of the strongest opioid analgesics available 2. Cannot lead to physical or psychological dependence 3. Does not cause gastrointestinal upset or constipation as do other opioids 4. Does not alter respirations or mask neurological signs as do other opioids

4. Does not alter respirations or mask neurological signs as do other opioids Rationale: Codeine sulfate is the opioid analgesic often used for clients after craniotomy. It is frequently combined with a nonopioid analgesic such as acetaminophen for added effect. It does not alter the respiratory rate or mask neurological signs as do other opioids. Side effects of codeine include gastrointestinal upset and constipation. The medication can lead to physical and psychological dependence with chronic use. It is not the strongest opioid analgesic available.

223.) A client with a psychotic disorder is being treated with haloperidol (Haldol). Which of the following would indicate the presence of a toxic effect of this medication? 1. Nausea 2. Hypotension 3. Blurred vision 4. Excessive salivation

4. Excessive salivation Rationale: Toxic effects include extrapyramidal symptoms (EPS) noted as marked drowsiness and lethargy, excessive salivation, and a fixed stare. Akathisia, acute dystonias, and tardive dyskinesia are also signs of toxicity. Hypotension, nausea, and blurred vision are occasional side effects.

948. A nurse is administering risperidone (Risperdal) to a client who is scheduled to be discharged. Before discharge, which of the following should the nurse teach the client? 1. Get adequate sunlight. 2. Avoid foods rich in potassium. 3. Continue driving as usual. 4. Get up slowly when changing positions.

4. Get up slowly when changing positions.

196.) A client has been prescribed amikacin (Amikin). Which of the following priority baseline functions should be monitored? 1. Apical pulse 2. Liver function 3. Blood pressure 4. Hearing acuity

4. Hearing acuity Rationale: Amikacin (Amikin) is an antibiotic. This medication can cause ototoxicity and nephrotoxicity; therefore, hearing acuity tests and kidney function studies should be performed before the initiation of therapy. Apical pulse, liver function studies, and blood pressure are not specifically related to the use of this medication.

175.) A nurse notes that a client is receiving lamivudine (Epivir). The nurse determines that this medication has been prescribed to treat which of the following? 1. Pancreatitis 2. Pharyngitis 3. Tonic-clonic seizures 4. Human immunodeficiency virus (HIV) infection

4. Human immunodeficiency virus (HIV) infection Rationale: Lamivudine is a nucleoside reverse transcriptase inhibitor and antiviral medication. It slows HIV replication and reduces the progression of HIV infection. It also is used to treat chronic hepatitis B and is used for prophylaxis in health care workers at risk of acquiring HIV after occupational exposure to the virus. **Note the letters "-vir" in the trade name for this medication**

192.) A nurse is collecting medication information from a client, and the client states that she is taking garlic as an herbal supplement. The nurse understands that the client is most likely treating which of the following conditions? 1. Eczema 2. Insomnia 3. Migraines 4. Hyperlipidemia

4. Hyperlipidemia Rationale: Garlic is an herbal supplement that is used to treat hyperlipidemia and hypertension. An herbal supplement that may be used to treat eczema is evening primrose. Insomnia has been treated with both valerian root and chamomile. Migraines have been treated with feverfew.

73.) A client with myasthenia gravis is suspected of having cholinergic crisis. Which of the following indicate that this crisis exists? 1. Ataxia 2. Mouth sores 3. Hypotension 4. Hypertension

4. Hypertension Rationale: Cholinergic crisis occurs as a result of an overdose of medication. Indications of cholinergic crisis include gastrointestinal disturbances, nausea, vomiting, diarrhea, abdominal cramps, increased salivation and tearing, miosis, hypertension, sweating, and increased bronchial secretions.

76.) Carbidopa-levodopa (Sinemet) is prescribed for a client with Parkinson's disease, and the nurse monitors the client for adverse reactions to the medication. Which of the following indicates that the client is experiencing an adverse reaction? 1. Pruritus 2. Tachycardia 3. Hypertension 4. Impaired voluntary movements

4. Impaired voluntary movements Rationale: Dyskinesia and impaired voluntary movement may occur with high levodopa dosages. Nausea, anorexia, dizziness, orthostatic hypotension, bradycardia, and akinesia (the temporary muscle weakness that lasts 1 minute to 1 hour, also known as the "on-off phenomenon") are frequent side effects of the medication.

212.) Mannitol (Osmitrol) is being administered to a client with increased intracranial pressure following a head injury. The nurse assisting in caring for the client knows that which of the following indicates the therapeutic action of this medication? 1. Prevents the filtration of sodium and water through the kidneys 2. Prevents the filtration of sodium and potassium through the kidneys 3. Decreases water loss by promoting the reabsorption of sodium and water in the loop of Henle 4. Induces diuresis by raising the osmotic pressure of glomerular filtrate, thereby inhibiting tubular reabsorption of water and solutes

4. Induces diuresis by raising the osmotic pressure of glomerular filtrate, thereby inhibiting tubular reabsorption of water and solutes Rationale: Mannitol is an osmotic diuretic that induces diuresis by raising the osmotic pressure of glomerular filtrate, thereby inhibiting tubular reabsorption of water and solutes. It is used to reduce intracranial pressure in the client with head trauma.

200.) A client is seen in the clinic for complaints of skin itchiness that has been persistent over the past several weeks. Following data collection, it has been determined that the client has scabies. Lindane is prescribed, and the nurse is asked to provide instructions to the client regarding the use of the medication. The nurse tells the client to: 1. Apply a thick layer of cream to the entire body. 2. Apply the cream as prescribed for 2 days in a row. 3. Apply to the entire body and scalp, excluding the face. 4. Leave the cream on for 8 to 12 hours and then remove by washing.

4. Leave the cream on for 8 to 12 hours and then remove by washing. Rationale: Lindane is applied in a thin layer to the entire body below the head. No more than 30 g (1 oz) should be used. The medication is removed by washing 8 to 12 hours later. Usually, only one application is required.

119.) A client with diabetes mellitus who has been controlled with daily insulin has been placed on atenolol (Tenormin) for the control of angina pectoris. Because of the effects of atenolol, the nurse determines that which of the following is the most reliable indicator of hypoglycemia? 1. Sweating 2. Tachycardia 3. Nervousness 4. Low blood glucose level

4. Low blood glucose level Rationale: β-Adrenergic blocking agents, such as atenolol, inhibit the appearance of signs and symptoms of acute hypoglycemia, which would include nervousness, increased heart rate, and sweating. Therefore, the client receiving this medication should adhere to the therapeutic regimen and monitor blood glucose levels carefully. Option 4 is the most reliable indicator of hypoglycemia.

205.) A nurse is assisting in preparing to administer acetylcysteine (Mucomyst) to a client with an overdose of acetaminophen (Tylenol). The nurse prepares to administer the medication by: 1. Administering the medication subcutaneously in the deltoid muscle 2. Administering the medication by the intramuscular route in the gluteal muscle 3. Administering the medication by the intramuscular route, mixed in 10 mL of normal saline 4. Mixing the medication in a flavored ice drink and allowing the client to drink the medication through a straw

4. Mixing the medication in a flavored ice drink and allowing the client to drink the medication through a straw Rationale: Because acetylcysteine has a pervasive odor of rotten eggs, it must be disguised in a flavored ice drink. It is consumed preferably through a straw to minimize contact with the mouth. It is not administered by the intramuscular or subcutaneous route. **Knowing that the medication is a solution that is also used for nebulization treatments will assist you to select the option that indicates an oral route**

36.) The client has a PRN prescription for ondansetron (Zofran). For which condition should this medication be administered to the postoperative client? 1. Paralytic ileus 2. Incisional pain 3. Urinary retention 4. Nausea and vomiting

4. Nausea and vomiting Rationale: Ondansetron is an antiemetic used to treat postoperative nausea and vomiting, as well as nausea and vomiting associated with chemotherapy. The other options are incorrect.

143.) A client has just taken a dose of trimethobenzamide (Tigan). The nurse plans to monitor this client for relief of: 1. Heartburn 2. Constipation 3. Abdominal pain 4. Nausea and vomiting

4. Nausea and vomiting Rationale: Trimethobenzamide is an antiemetic agent used in the treatment of nausea and vomiting. The other options are incorrect.

2.) Oral iron supplements are prescribed for a 6-year-old child with iron deficiency anemia. The nurse instructs the mother to administer the iron with which best food item? 1. Milk 2. Water 3. Apple juice 4. Orange juice

4. Orange juice Rationale: Vitamin C increases the absorption of iron by the body. The mother should be instructed to administer the medication with a citrus fruit or a juice that is high in vitamin C. Milk may affect absorption of the iron. Water will not assist in absorption. Orange juice contains a greater amount of vitamin C than apple juice.

15.) The client with small cell lung cancer is being treated with etoposide (VePesid). The nurse who is assisting in caring for the client during its administration understands that which side effect is specifically associated with this medication? 1. Alopecia 2. Chest pain 3. Pulmonary fibrosis 4. Orthostatic hypotension

4. Orthostatic hypotension Rationale: A side effect specific to etoposide is orthostatic hypotension. The client's blood pressure is monitored during the infusion. Hair loss occurs with nearly all the antineoplastic medications. Chest pain and pulmonary fibrosis are unrelated to this medication.

184.) A nurse preparing a client for surgery reviews the client's medication record. The client is to be nothing per mouth (NPO) after midnight. Which of the following medications, if noted on the client's record, should the nurse question? 1. Cyclobenzaprine (Flexeril) 2. Alendronate (Fosamax) 3. Allopurinol (Zyloprim) 4. Prednisone

4. Prednisone Rationale: Prednisone is a corticosteroid that can cause adrenal atrophy, which reduces the body's ability to withstand stress. Before and during surgery, dosages may be temporarily increased. Cyclobenzaprine is a skeletal muscle relaxant. Alendronate is a bone-resorption inhibitor. Allopurinol is an antigout medication.

13.) The client with squamous cell carcinoma of the larynx is receiving bleomycin intravenously. The nurse caring for the client anticipates that which diagnostic study will be prescribed? 1. Echocardiography 2. Electrocardiography 3. Cervical radiography 4. Pulmonary function studies

4. Pulmonary function studies Rationale: Bleomycin is an antineoplastic medication (Chemotheraputic Agents) that can cause interstitial pneumonitis, which can progress to pulmonary fibrosis. Pulmonary function studies along with hematological, hepatic, and renal function tests need to be monitored. The nurse needs to monitor lung sounds for dyspnea and crackles, which indicate pulmonary toxicity. The medication needs to be discontinued immediately if pulmonary toxicity occurs. Options 1, 2, and 3 are unrelated to the specific use of this medication.

144.) A client is taking docusate sodium (Colace). The nurse monitors which of the following to determine whether the client is having a therapeutic effect from this medication? 1. Abdominal pain 2. Reduction in steatorrhea 3. Hematest-negative stools 4. Regular bowel movements

4. Regular bowel movements Rationale: Docusate sodium is a stool softener that promotes the absorption of water into the stool, producing a softer consistency of stool. The intended effect is relief or prevention of constipation. The medication does not relieve abdominal pain, stop gastrointestinal (GI) bleeding, or decrease the amount of fat in the stools.

66.) Trimethoprim-sulfamethoxazole (TMP-SMZ) is prescribed for a client. A nurse should instruct the client to report which symptom if it developed during the course of this medication therapy? 1. Nausea 2. Diarrhea 3. Headache 4. Sore throat

4. Sore throat Rationale: Clients taking trimethoprim-sulfamethoxazole (TMP-SMZ) should be informed about early signs of blood disorders that can occur from this medication. These include sore throat, fever, and pallor, and the client should be instructed to notify the health care provider if these symptoms occur. The other options do not require health care provider notification.

44.) A client is receiving acetylcysteine (Mucomyst), 20% solution diluted in 0.9% normal saline by nebulizer. The nurse should have which item available for possible use after giving this medication? 1. Ambu bag 2. Intubation tray 3. Nasogastric tube 4. Suction equipment

4. Suction equipment Rationale: Acetylcysteine can be given orally or by nasogastric tube to treat acetaminophen overdose, or it may be given by inhalation for use as a mucolytic. The nurse administering this medication as a mucolytic should have suction equipment available in case the client cannot manage to clear the increased volume of liquefied secretions.

67.) Phenazopyridine hydrochloride (Pyridium) is prescribed for a client for symptomatic relief of pain resulting from a lower urinary tract infection. The nurse reinforces to the client: 1. To take the medication at bedtime 2. To take the medication before meals 3. To discontinue the medication if a headache occurs 4. That a reddish orange discoloration of the urine may occur

4. That a reddish orange discoloration of the urine may occur Rationale: The nurse should instruct the client that a reddish-orange discoloration of urine may occur. The nurse also should instruct the client that this discoloration can stain fabric. The medication should be taken after meals to reduce the possibility of gastrointestinal upset. A headache is an occasional side effect of the medication and does not warrant discontinuation of the medication.

232.) A client in the mental health unit is administered haloperidol (Haldol). The nurse would check which of the following to determine medication effectiveness? 1. The client's vital signs 2. The client's nutritional intake 3. The physical safety of other unit clients 4. The client's orientation and delusional status

4. The client's orientation and delusional status Rationale: Haloperidol is used to treat clients exhibiting psychotic features. Therefore, to determine medication effectiveness, the nurse would check the client's orientation and delusional status. Vital signs are routine and not specific to this situation. The physical safety of other clients is not a direct assessment of this client. Monitoring nutritional intake is not related to this situation.

160.) Meperidine hydrochloride (Demerol) is prescribed for the client with pain. Which of the following would the nurse monitor for as a side effect of this medication? 1. Diarrhea 2. Bradycardia 3. Hypertension 4. Urinary retention

4. Urinary retention Rationale: Meperidine hydrochloride (Demerol) is an opioid analgesic. Side effects of this medication include respiratory depression, orthostatic hypotension, tachycardia, drowsiness and mental clouding, constipation, and urinary retention.

140.) The client has a new prescription for metoclopramide (Reglan). On review of the chart, the nurse identifies that this medication can be safely administered with which condition? 1. Intestinal obstruction 2. Peptic ulcer with melena 3. Diverticulitis with perforation 4. Vomiting following cancer chemotherapy

4. Vomiting following cancer chemotherapy Rationale: Metoclopramide is a gastrointestinal (GI) stimulant and antiemetic. Because it is a GI stimulant, it is contraindicated with GI obstruction, hemorrhage, or perforation. It is used in the treatment of emesis after surgery, chemotherapy, and radiation.

104.) Disulfiram (Antabuse) is prescribed for a client who is seen in the psychiatric health care clinic. The nurse is collecting data on the client and is providing instructions regarding the use of this medication. Which is most important for the nurse to determine before administration of this medication? 1. A history of hyperthyroidism 2. A history of diabetes insipidus 3. When the last full meal was consumed 4. When the last alcoholic drink was consumed

4. When the last alcoholic drink was consumed Rationale: Disulfiram is used as an adjunct treatment for selected clients with chronic alcoholism who want to remain in a state of enforced sobriety. Clients must abstain from alcohol intake for at least 12 hours before the initial dose of the medication is administered. The most important data are to determine when the last alcoholic drink was consumed. The medication is used with caution in clients with diabetes mellitus, hypothyroidism, epilepsy, cerebral damage, nephritis, and hepatic disease. It is also contraindicated in severe heart disease, psychosis, or hypersensitivity related to the medication.

185.) Which of the following herbal therapies would be prescribed for its use as an antispasmodic? Select all that apply. 1.Aloe 2.Kava 3.Ginger 4.Chamomile 5.Peppermint oil

4.Chamomile 5.Peppermint oil Rationale: Chamomile has a mild sedative effect and acts as an antispasmodic and anti-inflammatory. Peppermint oil acts as an antispasmodic and is used for irritable bowel syndrome. Topical aloe promotes wound healing. Aloe taken orally acts as a laxative. Kava has an anxiolytic, sedative, and analgesic effect. Ginger is effective in relieving nausea.

1 pint to ml

480 ml

1 tsp to ml

5 ml

valproic acid (Depakene)

50-100

1 gr to mg

60 mg

the most common angle for an intramuscular injection

90 degrees

1 quart to ml

960 ml

The nurse would question the use of mannitol on which of the following patients?

A 47 year old patient with anuria

c. Hypotension

A calcium channel blocker has been ordered for a client. Which condition in the client's history is a contraindication to this medication? a. Hypokalemia b. Dysrhythmias c. Hypotension d. Increased intracranial pressure

b. Hepatic disease

A client diagnosed with hypercholesterolemia is prescribed lovastatin (Mevacor). The nurse is reviewing the client's history and would contact the health care provider about which of these conditions in the client's history? a. Chronic pulmonary disease b. Hepatic disease c. Leukemia d. Renal disease

c. Hyperlipidemia

A client has a serum cholesterol level of 265 mg/dL, triglyceride level of 235 mg/dL, and LDL of 180 mg/dL. What do these serum levels indicate? a. Hypolipidemia b. Normolipidemia c. Hyperlipidemia d. Alipidemia

b. Administer protamine sulfate.

A client has been admitted through the emergency department and requires emergency surgery. The client has been receiving heparin. What nursing intervention is essential? a. Teach the client about the phenytoin. b. Administer protamine sulfate. c. Assess the INR before surgery. d. Administer vitamin K.

c. High-ceiling (loop) diuretic

A client has heart failure and is prescribed Lasix. The nurse is aware that furosemide (Lasix) is what kind of drug? a. Thiazide diuretic b. Osmotic diuretic c. High-ceiling (loop) diuretic d. Potassium-sparing diuretic

c. Thrombolytic agent

A client is admitted to the emergency department with an acute myocardial infarction. Which drug category does the nurse expect to be given to the client early for the prevention of tissue necrosis following blood clot blockage in a coronary or cerebral artery? a. Anticoagulant agent b. Antiplatelet agent c. Thrombolytic agent d. Low-molecular-weight heparin (LMWH)

b. warfarin (Coumadin)

A client is being changed from an injectable anticoagulant to an oral anticoagulant. Which anticoagulant does the nurse realize is administered orally? a. enoxaparin sodium (Lovenox) b. warfarin (Coumadin) c. bivalirudin (Angiomax) d. lepirudin (Refludan)

b. Assess lung sounds before and after administration. c. Assess blood pressure before and after administration. d. Maintain accurate intake and output record.

A client is ordered furosemide (Lasix) to be given via intravenous push. What interventions should the nurse perform? (Select all that apply.) a. Administer at a rate no faster than 20 mg/min. b. Assess lung sounds before and after administration. c. Assess blood pressure before and after administration. d. Maintain accurate intake and output record. e. Monitor ECG continuously. f. Insert an arterial line for continuous blood pressure monitoring.

b. Respiratory assessment

A client is prescribed a noncardioselective beta1 blocker. What nursing intervention is a priority for this client? a. Assessment of blood glucose levels b. Respiratory assessment c. Orthostatic blood pressure assessment d. Teaching about potential tachycardia

d. Subcutaneously

A client is prescribed dalteparin (Fragmin). LMWH is administered via which route? a. Intravenously b. Intramuscularly c. Intradermally d. Subcutaneously

c. Muscle pain.

A client is prescribed ezetimibe (Zetia). Which assessment finding will require immediate action by the nurse? a. Headache. b. Slight nausea. c. Muscle pain. d. Fatigue.

c. "I will increase dark-green, leafy vegetables in my diet."

A client is receiving warfarin (Coumadin) for a chronic condition. Which client statement requires immediate action by the nurse? a. "I will avoid contact sports." b. "I will take my medication in the early evening each day." c. "I will increase dark-green, leafy vegetables in my diet." d. "I will contact my health care provider if I develop excessive bruising."

a. Evaluate digoxin levels.

A client is taking digoxin (Lanoxin) 0.25 mg and furosemide (Lasix) 40 mg. When the nurse enters the room, the client states, "There are yellow halos around the lights." Which action will the nurse take? a. Evaluate digoxin levels. b. Withhold the furosemide c. Administer potassium. d. Document the findings and reassess in 1 hour.

a. "I take aspirin daily for headaches."

A client is taking enoxaparin (Lovenox) daily. Which client statement requires additional monitoring? a. "I take aspirin daily for headaches." b. "I take ibuprofen (Motrin) at least once a week for joint pain." c. "Whenever I have a fever, I take acetaminophen (Tylenol)." d. "I take my medicine first thing in the morning."

b. Hypokalemia

A client is taking hydrochlorothiazide 50 mg/day and digoxin 0.25 mg/day. What type of electrolyte imbalance does the nurse expect to occur? a. Hypocalcemia b. Hypokalemia c. Hyperkalemia d. Hypermagnesemia

d. Liver enzymes

A client is taking lovastatin (Mevacor). Which serum level is most important for the nurse to monitor? a. Blood urea nitrogen b. Complete blood count c. Cardiac enzymes d. Liver enzymes

c. Fish

A client taking spironolactone (Aldactone) has been taught about the therapy. Which menu selection indicates that the client understands teaching related to this medication? a. Apricots b. Bananas c. Fish d. Strawberries

b. Administer vitamin K.

A client who has been taking warfarin (Coumadin) is admitted with coffee-ground emesis. What is the nurse's primary action? a. Administer vitamin E. b. Administer vitamin K. c. Administer protamine sulfate. d. Administer calcium gluconate.

c. Lungs clear.

A client with acute pulmonary edema receives furosemide (Lasix). What assessment finding indicates that the intervention is working? a. Potassium level decreased from 4.5 to 3.5 mEq/L. b. Crackles auscultated in the bases. c. Lungs clear. d. Output 30 mL/hr.

c. Decreased aldosterone

A client with hyperaldosteronism is prescribed spironolactone (Aldactone). What assessment finding would the nurse evaluate as a positive outcome? a. Decreased potassium level b. Decreased crackles in the lung bases c. Decreased aldosterone d. Decreased ankle edema

c. Stage 1 hypertension

A client's blood pressure (BP) is 145/90. According to the guidelines for determining hypertension, the nurse realizes that the client's BP is at which stage? a. Normal b. Prehypertension c. Stage 1 hypertension d. Stage 2 hypertension

b. It is the desired level of HDL.

A client's high-density lipoprotein (HDL) is 60 mg/dL. What does the nurse acknowledge concerning this level? a. It is lower than the desired level of HDL. b. It is the desired level of HDL. c. It is higher than the desired level of HDL. d. It is a much lower HDL level than desired.

a. Administer ordered dose of digoxin.

A client's serum digoxin level is drawn, and it is 0.4 ng/mL. What is the nurse's priority action? a. Administer ordered dose of digoxin. b. Hold future digoxin doses. c. Administer potassium. d. Call the health care provider.

decadron

A drug in the steriod family that is used to decrease swelling in the brain.

side effects of ace inhibitors

A dry, hacking cough is COMMON. Hyperkalemia, renal tubular damage, decreased B/P, dizziness, nausea & diarrhea

synthroid

A medication used to treat hypothyroidism

tigan

A medication used to treat nausea and vomiting

b. 0.5 to 2.0 ng/mL

A newly admitted client takes digoxin 0.25 mg/day. The nurse knows that which is the serum therapeutic range for digoxin? a. 0.1 to 1.5 ng/mL b. 0.5 to 2.0 ng/mL c. 1.0 to 2.5 ng/mL d. 2.0 to 4.0 ng/mL

c. The fact that Lasix has shown efficacy in treating persons with renal insufficiency.

A nurse admits a client diagnosed with pneumonia. The client has a history of chronic renal insufficiency, and the health care provider orders furosemide (Lasix) 40 mg twice a day. What is most important to include in the teaching plan for this client? a. That the medication will have to be monitored very carefully owing to the client's diagnosis of pneumonia. b. The fact that Lasix has been proven to decrease symptoms with pneumonia. c. The fact that Lasix has shown efficacy in treating persons with renal insufficiency. d. That the medication will need to be given at a higher than normal dose owing to the client's medical problems.

c. Have the client increase fluids and fiber in his diet.

A nurse is caring for a client taking cholestyramine (Questran). The client is complaining of constipation. What will the nurse do? a. Call the health care provider to change the medication. b. Tell the client to skip a dose of the medication. c. Have the client increase fluids and fiber in his diet. d. Administer an enema to the client.

c. gemfibrozil (Lopid)

A nurse is caring for a client with elevated triglyceride levels who is unresponsive to HMG-CoA reductase inhibitors. What medication will the nurse administer? a. cholestyramine (Questran) b. colestipol (Colestid) c. gemfibrozil (Lopid) d. simvastatin (Zocor)

d. Client stating that pain is 0 out of 10

A nurse is monitoring a client with angina for therapeutic effects of nitroglycerin. Which assessment finding indicates that the nitroglycerin has been effective? a. Blood pressure 120/80 mm Hg b. Heart rate 70 beats per minute c. ECG without evidence of ST changes d. Client stating that pain is 0 out of 10

c. Administer the medication into subcutaneous tissue.

A nurse is preparing to administer enoxaparin sodium (Lovenox) to a client for prevention of deep vein thrombosis. What is an essential nursing intervention? a. Draw up the medication in a syringe with a 22-gauge, 1-½ inch needle. b. Utilize the Z-track method to inject the medication. c. Administer the medication into subcutaneous tissue. d. Rub the administration site after injecting.

d. Serum glucose (sugar)

A nurse teaching a client who has diabetes mellitus and is taking hydrochlorothiazide 50 mg/day. The teaching should include the importance of monitoring which levels? a. Hemoglobin and hematocrit b. Blood urea nitrogen (BUN) c. Arterial blood gases d. Serum glucose (sugar)

Morphine

A respiratory depressant. It should be withheld if the respirations are below 10

-pril

ACE inhibitor

ace inhibitors

ACTION: prevent the conversion of angiotensin I to angiotensin II in the lungs USES: CHF, HTN , usually end in PRIL

-sartan

ARB angiotensin II receptor antagonist, antihypertensive * losartan

The nurse would assess which laboratory value to determine the effectiveness of intravenous heparin?

Activated partial thromboplastin time (aPTT)

The nurse plans which of the following interventions to decrease the flushing reaction of niacin?

Administer aspirin 30 minutes before nicotinic acid

Antabuse

Alcohol Abuse Therapy Adjunct

Benadryl

Allergy & Cough & Cold

Codeine

Allergy, Cough & Cold, Antitussive & Opioid Analgesic

Gentamicin

Aminoglycoside

Neomycin

Aminoglycoside

Streptomycin

Aminoglycoside

Tobramycin

Aminoglycoside

Neurontin

Analgesic Adjunct, Anticonvulsant & mood stabilizer for chronic pain or bipolar

Lidocaine

Anesthetic

-pril

Angio-tensin Converting Enzyme (ACE) inhibitors (Vasodilator; decreases resistance by decreasing Angiotensin II levels)

Aluminium hydroxide and Magnesium hydroxide (Maalox)

Antacid Medication

Aluminium hydroxide and magnesium trisilicate

Antacid Medication

Steroids (Prednisone)

Anti-asthmatic & Corticosteroid

Ampicillin

Anti-infective

Ciprofloxacin

Anti-infective

Coumadin

Anticoagulant

Heparin

Anticoagulant

Coumadin

Anticoagulant Medication

Heparin

Anticoagulant Medication

Lovenox

Anticoagulant Medication

Cerebyx

Anticonvulsant Medication

Glucophage

Antidiabetic

Insulin

Antidiabetic Hormone

Imodium

Antidiarrheal Medication

Kaopectate, Pepto-Bismol

Antidiarrheal Medication

ACE Inhibitors

Antihypertensive

Beta Blockers

Antihypertensive

Calcium Channel Blocker

Antihypertensive

Captopril

Antihypertensive

Rifampin

Antitubercular

Benylin DM, Pertissis

Antitussive/Expectorant Medication

ES, Vicks Formula 44

Antitussive/Expectorant Medication

Robitussin

Antitussive/Expectorant Medication

Antacids

Antiulcer

ANTIHYPERTENSIVES Beta Adrenergic Blockers

B is for beats works on heart itself decreases excitability of heart decreases O2 consumption Good for post MI) uses: HTN, angina, supraventricular tachycardia, decreases release of renin meds: tenormin (atenolol), corgard (nadalol), inderal (propanalol) OLOL OLOL side effect: hypotension BETA BLOCKERS CAN CAUSE BRONCHOSPAMS AND CHANGES BEATS OF THE HEART CAN MASK SIGNS OF SHOCK AND HYPOGLYCEMIA UNIQUE: take WITH meals DO NOT DISC RAPIDLY ( 2weeks) Not as effective in african americans

Atropine: What checks do you do before giving this drug ?

BP

3. Heart failure Rationale: Calcium channel blockers such as verapamil (Calan) are used cautiously or are contraindicated in patients with heart failure because of the negative inotropic effects on cardiac muscle which may precipitate or worsen heart failure. Verapamil and calcium channel blockers are often prescribed to treat these conditions.

Because of its effect on the heart, verapamil (Calan,Covera- HS, Isoptin SR, Verelan) should be used with extra caution or is contraindicated in patients with: 1.Hypertension 2.Tachycardia 3.Heart failure 4.Angina

ending in: olol or lol

Beta blocker. Antihypertensive. Contraindicated in symptomatic bradycardia, greater than first degree heart block, class IV heart failure & asthma. Side effects include insomnia, fatigue, dizziness, nervousness, edema, increased airway resistance, muscle/joint pain, bradycardia, hyptotension, heart failure & bronchospasm. Nursing considerations: Monitor apical pulse for increased risk of bradycardia. DO NOT STOP TAKING ABRUPTLY. Interact with antihypertensives, insulin, & oral antidiabetic agents.

Evista

Bone Reabsorbtion Inhibitor

Actonel

Bone-Reabsorption Inhibitor

Boniva

Bone-Reabsorption Inhibitor

Fosamax

Bone-Reabsorption Inhibitor

Adrenalin, Primatene Mist

Bronchodilator

Albuterol

Bronchodilator

Aminophylline

Bronchodilator

Atrovent

Bronchodilator

Brethaire

Bronchodilator

Intal

Bronchodilator

Mucomyst

Bronchodilator

Proventil

Bronchodilator

Serevent

Bronchodilator

Ritalin

CNS Stimulant

-dipine

Ca channel blocker (prevents Ca+ from enering heart & blood vessels ==> decreases heart pump strength, & relaxes blood vessels) tx: Hypertension, Angina, Arrhythmias

Which classification of drugs is used to treat both hypertension and dysrhythmias?

Calcium channel blockers

d. Constant, irritating cough

Captopril (Capoten) has been ordered for a client. The nurse teaches the client that ACE inhibitors have which common side effects? a. Nausea and vomiting b. Dizziness and headaches c. Upset stomach d. Constant, irritating cough

Diamox

Carbonic Anhydrase Inhibitor

Lanoxin (Digoxin)

Cardiac Glycosides

Cephalexin (Keflex)

Cephalosporin 1st generation

Cefaclor (Ceclor)

Cephalosporin 2nd generation

Ceftriaxone (Rocephin)

Cephalosporin 3rd generation

d. To suppress platelet aggregation

Cilostazol (Pletal) is being prescribed for a client with coronary artery disease. The nurse knows that which is the major purpose for antiplatelet drug therapy? a. To dissolve the blood clot b. To decrease tissue necrosis c. To inhibit hepatic synthesis of vitamin K d. To suppress platelet aggregation

Sodium channel blockers are considered which class of antiarrhythmic drugs?

Class I

ANTI-PARKINSONIAN

Cogentin: Sinemet:

The nurse would question an order for colesevelam (Welchol) in a patient with which conditon?

Complete bowel obstruction

Calcium channel blockers have which pharmacodynamic effect?

Coronary vasodilation

Carafate

Cytoprotective Agents

CATHETER EMBOLISM

DECREASED BLOOD PRESSURE PAIN ALONG THE VEIN WEAK, RAPID PULSE CYANOSIS OF THE NAIL BEDS LOSS OF CONSCIOUSNESS

The nurse monitors a patient prescribed dicyclomine (Bentyl) for which therapeutic effect?

Decrease in gastrointestinal motility

A patient receiving intravenous nitroglycerin at 20 mcg/min complains of dizziness. Nursing assessment reveals a blood pressure of 85/40 mm Hg. heart rate of 110 beats/min, and respiratory rate of 16 breaths/min. What is the nurse's best action?

Decrease the intravenous nitroglycerin by 10 mcg/min

Which is an expected outcome associated with the administration of digoxin?

Decreased heart rate

The nurse monitors a patient taking tolterodine (Detrol) for which thereapeutic effect?

Decreased in urinary frequency

During patient teaching, the nurse explains that nitroglycerin relieves chest pain primarily by which action?

Decreasing the workload of the heart

Lanoxin

Digitalis Preparations: Digoxin/Digitoxin (increases force of heart contractions. Benefecial in heart failue & iregular heart beats) tx: Heart failure symptoms if NOT responding to Ace inhibitors & Diuretics. Arrhthymias especially Atrial Fibrillation.

Lasix

Diurectic

Aldactone

Diuretic

Mannitol

Diuretic

Aldactone

Diuretics

Bumex

Diuretics

Diuril

Diuretics

Edecrin

Diuretics

Hydrodiuril

Diuretics

Hygroton, Thalitone

Diuretics

Lasix

Diuretics

Mannitol

Diuretics

Sinemet:

Drug is effective when tremors are not observed

ANALGESICS

Drugs used to relieve or eliminate pain: Aspirin NSAID's e.g. Ibuprofen— Morphine

Carvedilol (Coreg) is classified as a

Dual action alpha, and beta-receptor blocker

HEMATOMA

ECCHYMOSIS, IMMEDIATE SWELLING & LEAKAGE OF BLOOD AT THE SITE, HARD & PAINFUL LUMPS AT THE SITE.

INFILTRATION

EDEMA, PAIN,& COOLNESS AT THE SITE, MAY OR MAY NOT HAVE BLOOD RETURN.

ANTIDEPRESSANT Tricyclics

Elavil, Tofranil, Norpramin depression, sleep apnea side effects: sedation, anticholinergic effects, confusion, POSTURAL HYPOTENSION, urinary retention Monitor for suicide Take 2-6 weeks to work monitor VS take PM wean off over time IF rapidly discontinued: headache, vertigo rapid weight change, no alcohol, monitor for sleep inducing drugs, avoid sun

Cialis

Erectile Dysfunction Agent

It is most important to instruct a patient prescribed nitroglycerin to avoid which substances?

Erectile dysfunction medications

A patient is taking digoxin 0.25 mg and furosemide (Lasix) 40 mg. When the nurse enters the room, the patient states, "There are nice yellow halos around the lights." which action will the nurse take?

Evaluate the patient for other symptoms of digoxin toxicity

Alcaine

Eye Medication

Betagan

Eye Medication

Garamycin

Eye Medication

Regular, Humulin R

FAST ACTING Onset 0.5 - 1 hr Peak 2-4 hr Duration 6-8 hr if taken with break fast, mid morning S&S of hypoglycemia given SQ REGULAR INSULIN is the ONLY insulin given by IV or by implanted pump.

ending in: floxacin

Flouroquionlone. Antibiotic. Contraindicated in renal impairment or if hx of seizures. Side effects include dizziness, fatigue/lethargy, insomnia, depression, restlessness, conflusion, convulsions, oral candidiasis, dysphagia, pseudomembranous colitis, increased liver labs, rash, pruritis, urticaria, photosensitivity, flushing, fever, chills, piloerection, tinnitus, blurred vision. Nursing considerations: Monitor PT & INR, check for increased bleeding. Eat small frequent meals with increased protein. Report dyspnea, severe h/a, dizziness or weakness ASAP. Interact with oral antacids, iron/zinc/sucralfate supplements, glucocorticosteroids, alkaline foods (dairy, veggies, legumes).

Ciprofloxacin (Cipro)

Fluoroquinolone

-prazole

GI proton pump inhibitor, antiulcer * omeprazole, pantoprazole

what are 6 conditions for a patient to recieve TPN

GI, trauma, burn, malnutrtion, cancer, AIDS

Nitrofurantoin (Furadantin)

Genitourinary Medication

Phenazopyridine (Pyridium)

Genitourinary Medication

Sildenafil (Viagra)

Genitourinary Medication, Anti-impotence

Verdenafil (Levitra)

Genitourinary Medication, Anti-impotence

Derifenacin (Enablex)

Genitourinary Medication, Anticholinergic

Hyoscynamine (Anaspaz)

Genitourinary Medication, Anticholinergic

Oxybutynin (Ditropan)

Genitourinary Medication, Anticholinergic

Hydrocortisone (Solu-Cortef)

Glucocorticoid

Prednisone (Deltasone)

Glucocorticoid

Vancomycin

Glycopeptide

MAOI inhibitors:

Have dangerous food-drug interactions. Food with Tyramine should be avoided. For example: aged cheese, wine etc.

ANTI-COAGULANTS

Heparin: Coumadin:

Kava

Herbal Antianxiety Agent

Synthroid

Hormone for Thyroid

Pitocin

Hormone, induces labor

DIURETICS

Hydrochlothiazide Lasix Aldactone

When assessing a patient on triamterene (Dyrenium), the nurse would specifically monitor for which adverse effect?

Hyperkalemia

Which laboratory value depics a known side effect of furosemide (Lasix)?

Hypokalemia

Kayexalate

Hypokalemic & Electrolyte Modifiers

The nurse would question an order for a calcium channel blocking drug in a patient with which condition?

Hypotension

besides vitals what other signs should be noted during TPN

I and O

CIRCULATORY OVERLOAD

INCREASED BLOOD PRESSURE DISTENDED JUGULAR VEINS RAPID BREATHING DYSPNEA MOIST COUGH & CRACKLES

NPH, Humulin N

INTERMEDIATE ACTING insulin onset 2 hrs peak 6-12 hrs duration 18-26hr if given in the morning , peak time early evening

Cyclosporine

Immunosuppressant

Cyclosporine

Immunosuppressant & Antirheumatic

HMG-CoA reductase inhibitors (statins) are generally administered at which time?

In the evening

Cholinergic (parasympathomimetic) drugs have which therapeutic effect?

Increased gastroinstestinal motility

Epinephrine, as an adrenergic (sympathomimetic) drug, produces which of the following therapeutic effects?

Increased heart rate and contractility

The nurse will assess a patient receiving gemfibrozil (Lopid) and warfarin (Coumadin) for which adverse effect?

Increased risk of bleeding

The priority nursing diagnosis for a patient taking metoprolol (Lopressor) would be

Ineffective tissue perfusion (cerebral and cardiovascular) related to effects of medication

Which is a priority nursing diagnosis for a patient taking an antihypertensive medication?

Ineffective tissue perfusion related to disease process and/or medication

Atorvastatin (Lipitor) decreases lipid levels by

Inhibiting HMG-CoA reductase, the enzyme responsible for the biosynthesis of cholesterol in the liver.

Lente

Intermediate-Acting Insulin

NPH

Intermediate-Acting Insulin

FeS04, Feosol

Iron Preparation

Imferon

Iron Preparation

Liver metabolizes MOST drugs

Kidneys excrete MOST drugs

Lithium Carbonate:.

Know therapeutic range (0.8 to 1.2mEq). Also know symptoms of toxicity. Adequate fluid and salt intake is important

INFECTION

LOCAL- REDNESS, SWELLING & DRAINAGE AT THE SITE SYSTEMIC- CHILLS. FEVER, MALAISE, HEADACHE, NAUSEA, VOMITING, BACKACHE, TACHYCARDIA

Bisacodyl (Dulcolax)

Laxatives and Stool Softeners

Cascara

Laxatives and Stool Softeners

Docusate (Colace)

Laxatives and Stool Softeners

Metamucil

Laxatives and Stool Softeners

Lactulose

Laxitive

Lipitor

Lipid Lowering Agent

Questran

Lipid Lowering Agent

PSYCHOTROPICS

Lithium Carbonate:. MAOI inhibitors: Disulfiram (Antabuse)

Bipolar meds

Lithium, Tegretol, Depakote uses: mostly used during manic episodes DO NOT TAKE CARBONATED BEVERAGES WITH THESE DRUGS They are also anticonvulsants sub classes: Lithium

Which test will the nurse use to assess for adverse reactions to HMG-CoA reductase inhibitors?

Liver functin tests

Ultralente

Long-Acting Insulin

Enoxaparin sodium (Lovenox) is an anticoagulant used to prevent and treat deep vein thrombosis and pulmonary embolism. This drug is in which drug group?

Low molecular weight heparin

Cholinergic (parasympathomimetic) drugs are indicated for which situation

Lowering intraocular pressure in patients with glaucoma

Clindamycin HCL Phosphate (Cleocin)

Lycosamide

A hypertensive crisis may occur if adrenergic (sympathomimetic) drugs are given along with

MAO inhibitors

Nardil

MAO inhibitors

mARplan, pARnate, nARdil (trade names)

MAOI antidepressant-4 wk to effectiveness adfx: Hypertensive crisis with tyramine foods: aged cheese, liver, yogurt, beer, wine, pickled foods, herring, bologna, salami, bananas. potentiate alcohol, interactions with CNS stimulants or cold meds

ANTIDEPRESSANTS

MAOIs, SSRIs, Tricyclics actions increase concentration of neurotransmitters used in depression and chronic pain

Azethromycin (Zithromax)

Macrolide

Erythromycin (Erythrocin)

Macrolide

ANTIHYPERTENSIVE (PRE-ECLAMPSIA)

Magnesium Sulfate: Monitor for deep tendon reflex and respiratory depression

While observing a patient self-administer enoxaparin (Lovenox), the nurse identifies the need for further teaching when the patient

Massages the site after administration of the medication

AcipHex

Medication for Ulcer

Axid

Medication for Ulcer

Carafate

Medication for Ulcer

Cytotec

Medication for Ulcer

Nexium

Medication for Ulcer

Pepcid

Medication for Ulcer

Prevacid

Medication for Ulcer

Prilosec

Medication for Ulcer

Protonix

Medication for Ulcer

Calcium

Mineral

Potassium

Mineral

Sodium fluoride

Mineral

Hydrochlothiazide

Monitor potassium levels

Lasix

Monitor potassium levels

Heparin:

Monitor pt's lab work-PTT. Antidote is protamine sulfate

Coumadin:

Monitor pt's lab work—PT. Antidote is Vitamin K

Lithium

Mood Stabilizer

Discharge teaching for a patient receiving simvastatin (Zocor) should include the importance of reporting which symptoms that might indicate a serious adverse reaction to the medication?

Muscle pain

Alendronate sodium (Fosamax)

Musculoskeletal Medications

Edrophonium (Tensilon)

Musculoskeletal Medications

Celecoxib, Celebrex

NSAID

Ibuprofen, Motrin

NSAID

Indomethacin, lndocin

NSAID

Ketorolac

NSAID

Naproxen, Naprosyn

NSAID

ANTIDOTES

Narcan Calcium Gluconate Vitamin K

Codeine

Narcotic (Opioid Anagesic)

Morphine sulfate

Narcotic (Opioid Anagesic)

Oxycodone (Percodan)

Narcotic (Opioid Anagesic)

Which effects would alert the nurse to suspect early digitalis toxicity?

Nausea and vomiting

Isosorbide, lsordil,Sorbitrate

Nitrate/Antianginal

Nitroglycerin, Nitro-Bid, Nitrostat

Nitrate/Antianginal

Aspirin

Non Opiod Analgesic & Antipyretic

Acetaminophen

Non Opioid Analgesic & Antipyretic

ANTI DIABETIC MEDICATIONS

ONSET AND PEAK very important in relation to food requirements

Fentanyl

Opioid Analgesic

Morphine

Opioid Analgesic

Glyset

Oral Hypoglycemic Agents, Alpha Glucoside Inhibitors

Precose

Oral Hypoglycemic Agents, Alpha Glucoside Inhibitors

Glucophage

Oral Hypoglycemic Agents, Biguanides

Prandin

Oral Hypoglycemic Agents, Meglitinides

Amaryl

Oral Hypoglycemic Agents, Sulfonylureas

Glucotrol

Oral Hypoglycemic Agents, Sulfonylureas

Micronase

Oral Hypoglycemic Agents, Sulfonylureas

Actus

Oral Hypoglycemic Agents, Thiazolidinediones

Avandia

Oral Hypoglycemic Agents, Thiazolidinediones

MATERNITY DRUGS

Oxytocin: Assess uterus frequently for tetanic contraction.

peripherally inserted central catheter aka this, used in basilic or cephalic vein when TPN is needed for up to 3 months

PICC line

if TPN is dislodged or pulled out what is to be in replace until another TPN is placed

PPN of less than 10% Dextrose

Calcitonin

Paget's Disease Medication

Etidronate disodium

Paget's Disease Medication

Mithramycin

Paget's Disease Medication

Adenosine is used to treat which condition?

Paroxysmal supraventricular tachycardia (PSVT)

Amoxicillin

Penicillin

Ampicillin

Penicillin

Methicilin

Penicillin

Penicillin G

Penicillin

Penicillin V

Penicillin

Before administering alteplase (Activase), it is important for the nurse to perform which action?

Perform all necessary venipunctures

ending in: zosin

Peripherally acting alpha-1 blocker. Antihypertensive. Side effects include first dose syncope, h/a, drowsiness, hypotension, palpitations, impotence, nasal congestion, tachycardia, anaphylaxis & arrhythmia. Nursing considerations: May cause syncope within 30-60 min of first dose. DO NOT STOP TAKING ABRUPTLY. Monitor VS, especially BP & pulse. Monitor for CHF/edema. Monitor diabetic clients for hypoglycemia. Abrupt d/c may worsen angina or precipitate MI. Change position slowly to prevent dizziness/falls. Weigh daily, report weight gain >5lbs/week. May take 3-4 weeks for tx response. Interact with oral antidiabetic agents, digoxin, & beta blockers.

Which are therapeutic effects of digoxin?

Positive inotropic, negative chronotropic, and negative dromotropic

Kcl

Potassium chloride, it is often given to cardiac patients whose potassium is depleted by diuretic medications, such as Lasix.

Aldactone

Potassium sparing

Aldactone is often prescribed for children with CHF because

Potassium sparing diuretic

ANTIINFLAMMATORY

Predisone: Causes Cushing like symptoms. Common side effects are immunosupression(monitor client for infection), hyperglycemia

ending in: prazole

Proton pump inhibitor (PPI). Antiulcer. Contraindicated in children & nursing mothers. Side effects include h/a, diarrhea, constipation, flatulence, rash, hyperglycemia, dizziness, pruritis, dry mouth, pancreatitis, liver necrosis, hepatic failure, toxic epidermal necrolysis, Stevens-Johnson syndrome, MI, shock, CVA, & GI hemorrhage. Nursing considerations: May be given with antacids. Dosage should be reduced in severe liver disease. Monitor hepatic & renal labs. Increase water intake to 8-10 glasses per day to prevent constipation. Report diarrhea. Interact with digoxin, iron preparations, ampicillin, theophylline, diazepam, pheytoin, warfarin, and benzodiazepines.

Amiodarone toxicity is evidenced in which body system?

Pulmonary

Humalog

Rapid-Acting Insulin

NovoLog

Rapid-Acting Insulin

A client with narcolepsy has been prescribed dextroamphetamine (Dexedrine). The client complains to the nurse that he cannot sleep well anymore at night and does not want to take the medication any longer. Before making any specific comment, the nurse plans to investigate whether the client takes the medication at which of the following proper time schedules? 1. After dinner each day 2. Just before going to bed 3. Two hours before bedtime 4. At least 6 hours before bedtime

Rationale: Dextroamphetamine is a central nervous system (CNS) stimulant, which acts by releasing norepinephrine from nerve endings. The client should take the medication at least 6 hours before going to bed at night to prevent disturbances with sleep. Taking the medication at the time frames indicated in options 1, 2, and 3 will prevent the client from sleeping because of the stimulant properties of the medication.

134.) A nurse reinforces instructions to a client who is taking levothyroxine (Synthroid). The nurse tells the client to take the medication: 1. With food 2. At lunchtime 3. On an empty stomach 4. At bedtime with a snack

Rationale: Oral doses of levothyroxine (Synthroid) should be taken on an empty stomach to enhance absorption. Dosing should be done in the morning before breakfast. **Note that options 1, 2, and 4 are comparable or alike in that these options address administering the medication with food.**

206.) A client is receiving baclofen (Lioresal) for muscle spasms caused by a spinal cord injury. The nurse monitors the client, knowing that which of the following is a side effect of this medication? 1. Muscle pain 2. Hypertension 3. Slurred speech 4. Photosensitivity

Rationale: Side effects of baclofen include drowsiness, dizziness, weakness, and nausea. Occasional side effects include headache, paresthesia of the hands and feet, constipation or diarrhea, anorexia, hypotension, confusion, and nasal congestion. Paradoxical central nervous system excitement and restlessness can occur, along with slurred speech, tremor, dry mouth, nocturia, and impotence. **Option 3 is most closely associated with a neurological disorder**

What is the best way to prevent tolerance to nitrates when using transdermal patches?

Remove the patch at night for 8 hours, and then apply a new patch in the morning

Narcan:

Reverses the effects of narcotics

-pram, -ine

SSRI

-etine, -aline

SSRI antidepressant- 4 weeks for effect adfx: anxiety, urinary/bowel retention, insomnia at hs, pink urine suicide risk on upswing

Celexa

SSRIs

Lexapro

SSRIs

Luvox

SSRIs

Paxil

SSRIs

Prozac

SSRIs

Zoloft

SSRIs

FLUOROQUINOLONES (antibiotics)

Sample: Cipro (ciprofloxin) Action: interferes with DNA of gram negative Uses: E.Coli, pseudodomonoas, S aureus, chronic bacterial prostatitis, post inhalation anthrax Side effects: diarrhea decreased WBC & hematocrit, elevated liver enzymes AST and ALT, elevated alakine phosphotase TAKE FLUIDS 3000 ml/day normal 1500/day DO NOT TAKE WITH FOOD, DO NOT TAKE WITH ANTIACIDS DO NOT TAKE WITH IRON PREPARATIONS MAY BE GIVEN WITH PROBENECID (GOUT MED), WHICH HELPS CIPRO ACT BETTER (less cipro, less side effects with more action) nursing considerations: Teratogenic antibiotic that causes cartilage damage

Restoril

Sedative & Hypnotic

Assessment of a patient is receiving a positive inotropic drug would include reviewing which values?

Serum electrolytes

Sulfasalazine (Azulfidine)

Sulfonamide

Sulfisoxazole (Gentrisin)

Sulfonamide

AIR EMBOLISM

TACHYCARDIA DYSPNEA HYPOTENSION CYANOSIS DECREASED LEVEL OF CONSCIOUSNESS

A patient who is taking an anticoagulant requests as aspirin for headache relief. What is the nurse's bes action?

Take advantage of a teachable moment to inform the patient of potential drug interactions with anticoagulants

When providing teaching to a patient diagnosed with myasthenia gravis, which instruction regarding the administration of physostigmine (Antilirium) is most appropriate?

Take the medication 30 minutes before meals

NSAID's e.g. Ibuprofen

Take with food; contraindicated for people with GI ulcers

Doxycycline (Vibramycin)

Tetracycline

Minocycline (Minocin)

Tetracycline

Tetracycline HCL (Panmycin)

Tetracycline

a. It is in the high (elevated) range.

The client's serum digoxin level is 3.0 ng/mL. What does the nurse know about this serum digoxin level? a. It is in the high (elevated) range. b. It is in the low (decreased) range. c. It is within the normal range. d. It is in the low average range.

c. Decrease heart rate and decrease myocardial contractility.

The nurse acknowledges that beta blockers are as effective as antianginals because they do what? a. Increase oxygen to the systemic circulation. b. Maintain heart rate and blood pressure. c. Decrease heart rate and decrease myocardial contractility. d. Decrease heart rate and increase myocardial contractility.

d. "I should use a soft toothbrush for dental hygiene."

The nurse evaluates that the client understood discharge teaching regarding warfarin (Coumadin) based on which statement? a. "I will double my dose if I forget to take it the day before." b. "I should keep taking ibuprofen for my arthritis." c. "I should decrease the dose if I start bruising easily." d. "I should use a soft toothbrush for dental hygiene."

a. Beta1 blocker

The nurse explains that which beta blocker category is preferred for treating hypertension? a. Beta1 blocker b. Beta2 blocker c. Beta1 and beta2 blockers d. Beta2 and beta3 blockers

c. Administer 2 mEq potassium chloride per kilogram per day IV.

The nurse is assessing a client who is taking furosemide (Lasix). The client's potassium level is 3.4 mEq/L, chloride is 90 mmol/L, and sodium is 140 mEq/L. What is the nurse's primary intervention? a. Mix 40 mEq of potassium in 250 mL D5W and infuse rapidly. b. Administer Kayexalate. c. Administer 2 mEq potassium chloride per kilogram per day IV. d. Administer PhosLo, two tablets three times per day.

c. Beta blockers and ACE inhibitors

The nurse is aware that which group(s) of antihypertensive drugs are less effective in African-American clients? a. Diuretics b. Calcium channel blockers and vasodilators c. Beta blockers and ACE inhibitors d. Alpha blockers

d. Chest pain

The nurse is monitoring a client during IV nitroglycerin infusion. Which assessment finding will cause the nurse to take action? a. Blood pressure 110/90 mm Hg b. Flushing c. Headache d. Chest pain

b. Heart rate 58 beats per minute

The nurse is monitoring a client taking digoxin (Lanoxin) for treatment of heart failure. Which assessment finding indicates a therapeutic effect of the drug? a. Heart rate 110 beats per minute b. Heart rate 58 beats per minute c. Urinary output 40 mL/hr d. Blood pressure 90/50 mm Hg

d. spironolactone (Aldactone)

The nurse is reviewing a medication history on a client taking an ACE inhibitor. The nurse plans to contact the health care provider if the client is also taking which medication? a. docusate sodium (Colace) b. furosemide (Lasix) c. morphine sulfate d. spironolactone (Aldactone)

a. 150 to 200 mg/dL

The nurse knows that the client's cholesterol level should be within which range? a. 150 to 200 mg/dL b. 200 to 225 mg/dL c. 225 to 250 mg/dL d. Greater than 250 mg/dL

b. hydrochlorothiazide

The nurse knows that which diuretic is most frequently combined with an antihypertensive drug? a. chlorthalidone b. hydrochlorothiazide c. bendroflumethiazide d. potassium-sparing diuretic

b. Administer aspirin 30 minutes before nicotinic acid.

The nurse plans which intervention to decrease the flushing reaction of niacin? a. Administer niacin with an antacid. b. Administer aspirin 30 minutes before nicotinic acid. c. Administer diphenhydramine hydrochloride (Benadryl) with niacin. d. Apply cold compresses to the head and neck.

b. To administer digoxin immune FAB (antidote)

The nurse reviews a client's laboratory values and finds a digoxin level of 10 ng/mL and a serum potassium level of 5.9 mEq/L. What is the nurse's primary intervention? a. To administer atropine b. To administer digoxin immune FAB c. To administer epinephrine d. To administer Kayexalate

RESPIRATORY

Theophylline/Aminophylline: Side effects--Tachycardia

The nurse would question the health care provider if which type of diuretics is prescribed to a patient with gout?

Thiazide diuretics

ending in: ase or plase

Thrombolytic. Clot buster. Side effects include hemorrhage, hypotension & cardiac dysrhythmias. Nursing considerations: DO NOT GIVE IM. Interact with anticoagulant & antiplatelet meds.

Activase

Thrombolytics

Eminase

Thrombolytics

Retavase

Thrombolytics

Streptokinase

Thrombolytics

Tissue plasminogen activator (tPA)

Thrombolytics

Bacitracin ointment

Topical Antibacterial

Neosporin cream

Topical Antibacterial

Which nitrate preparation or dosage form has the longest duration of action?

Transdermal nitroglycerin patch

Elavil

Tricyclics

Norpramin

Tricyclics

ANTIANXIETY AGENTS

USES: manic episodes, anxiety disorders and panic attacks Action: affect neuro transmitters. Examples: Librium (Chlordiazepoxide), Diazepam (Valium), alprazolam (xanax), lorazepam (ativan), versed (midazalam), vistaril (hydroxyzine), equanil Subclasses: benzos, nonbenzos such as melatonin and herbals such as Kava Kava remember most benzos end in: "pam and am" side effects (central nervous depression): sedation, confusion, hepatic dysfunction, depression nursing considerations: potential for addiction/overdose avoid alcohol monitor liver function stay in touch with client respiratory depression (labs: AST & ALT values)

Epogen:

Used in treating anemia because it increases RBC production.

Cogentin:

Used to treat EPS

Diphenhydramine HCL (Benedryl)

Used to treat shock, cardiac arrest and anaphylaxis

Dopamine

Used to treat shock, cardiac arrest and anaphylaxis

ANTIACIDS

Uses: peptic ulcer, indigestion, reflex esophagitis neutralize gastric acid: Antiacids: Ampholjel (aluminuim hydroxide), mik of magnesia, Maalox (both). Central nervous depressant THESE MEDS DO NOT COAT STOMACH side effects: constipation or diarrhea or acid rebound effect nursing considerations: Interferes with absorption of antibiotics, iron preps, INH, ORAL CONTRACEPTIVES (antibiotics also effects contraceptives!!!) Monitor bowel function may cause bowel addiction timing: 1-2 hr AFTER eating and other medications Fluid and electrolyte balances extremely important monitor S&S of labs shifting

the antidote for anticoagulants

VITAMIN K

Imitrex

Vascular Headache Suppressant

Lantus

Very Long-Acting Insulin

vin-

Vinca-alkyloid antineoplastic. derived from periwinkle. interferes with cell division ex: vinblastine use: CA- affects every rapid producing cell in body adfx: CNA Depression, stomatitis, alopecia, bone marrow suppression check reflexes, give with Zyloprim to reduce uric acid and adfx risk of extravasation. check IV

heterocyclic antidepressant

Wellbutrin, trazadone used also for smoking cessation adfx: wellbutrin- agitation and insomnia. trazadone- sedation wean slowly, no alcohol

b. Apply the ointment to a nonhairy part of the upper torso.

What instruction should the nurse provide to the client who needs to apply nitroglycerin ointment? a. Use the fingers to spread the ointment evenly over a 3-inch area. b. Apply the ointment to a nonhairy part of the upper torso. c. Massage the ointment into the skin. d. Cover the application paper with ointment before use.

b. "This combination promotes diuresis but decreases the risk of hypokalemia."

What is the best information for the nurse to provide to the client who is receiving spironolactone (Aldactone) and furosemide (Lasix) therapy? a. "Moderate doses of two different diuretics are more effective than a large dose of one." b. "This combination promotes diuresis but decreases the risk of hypokalemia." c. "This combination prevents dehydration and hypovolemia." d. "Using two drugs increases the osmolality of plasma and the glomerular filtration rate."

b. Continuous blood pressures d. Presence of chest pain

What must the nurse monitor when titrating intravenous nitroglycerin for a client? (Select all that apply.) a. Continuous oxygen saturation b. Continuous blood pressures c. Hourly ECGs d. Presence of chest pain e. Serum nitroglycerin levels f. Visual acuity

b. Elevated liver function tests

Which assessment finding in a client taking an HMG-CoA reductase inhibitor will the nurse act on immediately? a. Decreased hemoglobin b. Elevated liver function tests c. Elevated HDL d. Elevated LDL

a. Decreased intracranial pressure

Which assessment indicates a therapeutic effect of mannitol (Osmitrol)? a. Decreased intracranial pressure b. Decreased potassium c. Increased urine osmolality d. Decreased serum osmolality

a. Alteration in cardiac output related to effects on the sympathetic nervous system

Which is a priority nursing diagnosis for a client taking an antihypertensive medication? a. Alteration in cardiac output related to effects on the sympathetic nervous system b. Knowledge deficit related to medication regimen c. Fatigue related to side effects of medication d. Alteration in comfort related to nonproductive cough

b. Fasting blood glucose level of 140 mg/dL

Which laboratory value will the nurse report to the health care provider as a potential adverse response to hydrochlorothiazide (HydroDIURIL)? a. Sodium level of 140 mEq/L b. Fasting blood glucose level of 140 mg/dL c. Calcium level of 9 mg/dL d. Chloride level of 100 mEq/L

c. Risk for injury

Which nursing diagnosis would be possible for a client receiving intravenous heparin therapy? a. Potential for fluid volume excess b. Potential for pain c. Risk for injury d. Potential for body image disturbance

a. "I will check my blood pressure daily and take my medication when it is over 140/90."

Which statement indicates that the client needs additional instruction about antihypertensive treatment? a. "I will check my blood pressure daily and take my medication when it is over 140/90." b. "I will include rest periods during the day to help me tolerate the fatigue my medicine may cause." c. "I will change my position slowly to prevent feeling dizzy." d. "I will not mow my lawn until I see how this medication makes me feel."

d. "I can take up to five tablets at 3-minute intervals for chest pain if necessary."

Which statement made by the client demonstrates a need for further instruction regarding the use of nitroglycerin? a. "If I get a headache, I should keep taking nitroglycerin and use Tylenol for pain relief." b. "I should keep my nitroglycerin in a cool, dry place." c. "I should change positions slowly to avoid getting dizzy." d. "I can take up to five tablets at 3-minute intervals for chest pain if necessary."

Estradiol

Women's Health Medication

Ethinyl

Women's Health Medication

Norplant

Women's Health Medication

theophylline

a bronchodilator used to treat asthma and bronchitis and emphysema

An annoying side effect of ACE inhibitors that may be minimized by switching to an angiotensin receptor blocking drug includes

a dry, nonproductive cough

clomid

a fertility drug that is used to stimulate ovulation and that has been associated with multiple births

cbc

a test ordered for suspected bleeding disorders

3.A client is prescribed metoprolol (Lopressor) to treat hypertension. It is important for the nurse to monitor the client for which condition? a. Bradycardia b. Hypertension c. Ankle edema d. Decreased respirations

a. Bradycardia

2. The nurse understands that antipsychotics act in which way? a. By blocking actions of dopamine b. By blocking actions of epinephrine c. By promoting prostaglandin synthesis d. By enhancing the action of gamma-aminobutyric acid

a. By blocking actions of dopamine

The nurse is doing preconception counseling with the client. Folic acid is included in the health teaching pan because it is known to prevent CNS anomalies and may offer protection from which disorder? a. Colorectal cancer b. Diabetes mellitus c. Celiac disease d. Migraine headaches

a. Colorectal cancer

7. When a client is taking ezetimibe (Zetia), she asks the nurse how it works. The nurse should explain that Zetia does what?

a. Inhibits absorption of dietary cholesterol in the intestines.

6. When a client is taking an antidepressant, what should the nurse do? (Select all that apply.) a. Monitor the client for suicidal tendencies. b. Observe the client for orthostatic hypotension. c. Teach the client to take the drug with food if GI distress occurs. d. Tell the client that the drug may not have full effectiveness for 1 to 2 weeks. e. Advise the client to maintain adequate fluid intake of 2 L/day.

a. Monitor the client for suicidal tendencies. b. Observe the client for orthostatic hypotension. c. Teach the client to take the drug with food if GI distress occurs. d. Tell the client that the drug may not have full effectiveness for 1 to 2 weeks.

It is expected that the nurse will question the health care provider if a drug with a t ½ of >24 hours is ordered to be given more than how often? (select all that apply) a. Once daily b. Twice daily c. Twice weekly d. Once weekly

a. Once daily b. Twice daily

4. The client is also taking a diuretic that decreases her potassium level. The nurse expects that a low potassium level (hypokalemia) could have what effect on the digoxin?

a. The beta blocker should be abruptly stopped when another cardiac drug is prescribed.

8. The nurse should know that which drugs are used to treat migraine attacks? a. Triptans b. Anticonvulsants c. Tricyclic antidepressants d. Beta-adrenergic blockers

a. Triptans

7. When the client has a cholinergic overdose, the nurse anticipates administration of which drug as the antidote? a. atropine b. bethanechol c. ambenonium d. metoclopramide

a. atropine

3. A selective serotonin reuptake inhibitor (SSRI) is prescribed for a client. The nurse knows that which drug is an SSRI? a. paroxetine (Paxil) b. amitriptyline (Elavil) c. divalproex sodium (Depakote) d. bupropion HCl (Wellbutrin)

a. paroxetine (Paxil)

2. The nurse knows that the client's cholesterol level should be within which range?

a.150 to 200 mg/dL

3. A client is prescribed enoxaparin (Lovenox). The nurse knows that low-molecular-weight heparin (LMWH) has what kind of half-life?

a.A longer half-life than heparin

5. The nurse acknowledges that which condition could occur when taking furosemide?

a.Hypokalemia

-terol

adrenergic type of bronchodilator * albuterol, salmeterol

when should coumadin be administered

afternoon

Parkinson-like Effects Assessment

akinesia tremors drooling changes in gait

-mustine

alkylating agent (antineoplastic)

antibiotics general

allergic reaction (after 1st time), superinfection, liver kidney toxic do C&S, encourage fluids, toxic when expired peak level about an hour after IV, trough just after it's hung

classes of antacids

alminum, MG, NA, and Ca compounds

-zosin

alpha blocker

histamine

amine formed from histidine that stimulates gastric secretions and dilates blood vessels, a regulating body substance released in excess during allergic reactions causing swelling and inflammation of tissues

what are two main products of TPN

amino acids and dextrose

-mycin

aminoglycoside

-micin; -mycin

aminoglycoside, antibiotic * gentamycin, tobramycin

acyclovir

an oral antiviral drug (trade name Zovirax) used to treat genital herpes

zocor

an oral lipid-lowering medicine administered to reduce blood cholesterol levels

-caine

anesthetics

-sartan

angiotensin II receptor blocker; ARB

-parin

anticoagulant, heparin or heparinoid * enoxaparin, heparin

-ase

antidiabetic (also Dymelor) for type II only adfx: skin reaction, hypoglycemia take prior to eating- EAT. alcohol, aspirin, sulfonamides, oral birth control, maois affect action

Phenothiazine

antipsych

-azine

antipsychotic-block dopamine receptors adfx: akasthesia, dyskinesia, dystonia, parkinsonian, tardive dyskinesia, leukopenia, can stunt growth in kids avoid etoh and caffeine

-tidine

antiulcer

lipitor is best given

at night this is when the body makes the most cholesterol

An example of a cardioselective beta-blocker includes

atenolol (Tenormin)

what medication would be given fo an overdose of a cholinergic drug

atropine anticholenergic

208.) A client with myasthenia gravis verbalizes complaints of feeling much weaker than normal. The health care provider plans to implement a diagnostic test to determine if the client is experiencing a myasthenic crisis and administers edrophonium (Enlon). Which of the following would indicate that the client is experiencing a myasthenic crisis? 1. Increasing weakness 2. No change in the condition 3. An increase in muscle spasms 4. A temporary improvement in the condition

auto-define "A client with myasthen..." Rationale: Edrophonium (Enlon) is administered to determine whether the client is reacting to an overdose of a medication (cholinergic crisis) or to an increasing severity of the disease (myasthenic crisis). When the edrophonium (Enlon) injection is given and the condition improves temporarily, the client is in myasthenic crisis. This is known as a positive test. Increasing weakness would occur in cholinergic crisis. Options 2 and 3 would not occur in either crisis.

To prevent the occurrence of cinchonism in a patient prescribed quinidine, the nurse emphasizes the importance of

avoiding ingestion of antacids

The nurse determines the client's gastric residual before administering an enteral feeding; the last feeding was 240mL. the client will be discharged on enteral feedings. It is important to include in the health teaching plan that a residual of more than which amount would indicate a delayed gastric emptying (based on last feeding)? a. 100 mL b. 125 mL c. 150 mL d. 175 mL

b. 125 mL

Which of these statements is correct? a. A drug bound to a protein is an active drug b. A drug non bound to a protein is an active drug c. Most receptors are found under cell membrane d. Toxic effects can result if the trough level is low

b. A drug non bound to a protein is an active drug

The nurse notices that one of the client's drugs has a low therapeutic index. What is the most important nursing implication of this? a. A wide margin of safety b. A narrow margin of safety c. Measured 1 hour after administration d. Measured 10 minutes after administration

b. A narrow margin of safety

The visiting nurse has a caseload of adult and pediatric clients receiving TPN at home. The nurse carefully checks all orders for TPN solutions. Which orders (all have appropriate amounts of essential fatty acids, vitamins, and minerals) requires the nurse to contact the health care provider? a. Adults: water 32 mL/kg/d; energy 32 kcal/kg/d; amino acids 1.2g/kg/d b. Adults: water 34 mL/kg/d; amino acids 2g/kg/d c. Children: water 32 mL/kg/d; energy 120 kcal/kg/d; amino acids 2.5g/kg/d d. Children: water 38 mL/kg/d; energy 58 kcal/kg/d; amino acids 2g/kg/d

b. Adults: water 34 mL/kg/d; amino acids 2g/kg/d

The client is receiving TPN. Health teaching for this client includes the Valsalva maneuver, which is done to prevent which condition? a. Infection b. Air embolism c. Dehydration d. Fat embolism

b. Air embolism

7.The nurse is administering atenolol (Tenormin) to a client. Which concurrent drug does the nurse expect to most likely cause an interaction? a. ginseng herb b. An NSAID, such as aspirin c. methyldopa (Aldomet) d. haloperidol (Haldol)

b. An NSAID, such as aspirin

The alcoholic client has questions about his medications. The nurse correctly explains that alcoholism can be associated with deficiency of which vitamins? a. A b. B12 c. D d. K

b. B12

8. During an admission assessment, the client states that she takes amlodipine (Norvasc). The nurse wishes to determine whether or not the client has any common side effects of a calcium channel blocker. The nurse asks the client if she has which signs and symptoms? (Select all that apply.)

b. Dizziness c. Headache e. Ankle edema

Chapter 23 1. A client with parkinsonism asks the nurse to explain what causes this condition. The most accurate response by the nurse is that parkinsonism is caused by the degeneration of which? a. Cholinergic neurons b. Dopaminergic neurons c. Acetylcholine neurotransmitters d. Monamine oxidase-B neurotransmitters

b. Dopaminergic neurons

3. A family member of a client with Alzheimer's disease asks the nurse what causes this disorder. What does the nurse explain is the cause of Alzheimer's disease? a. An excess of acetylcholine b. Neurofibrillary tangles inside neurons c. Degeneration of dopaminergic neurons d. Neuritic plaques that form inside neurons in the cerebellum

b. Neurofibrillary tangles inside neurons

6. The nurse should monitor the client receiving phenytoin (Dilantin) for which adverse effect? a. Psychosis b. Nosebleeds c. Hypertension d. Gum erosion

b. Nosebleeds

7. The health care provider is planning to discontinue a client's beta blocker. What instruction should the nurse give the client regarding the beta blocker?

b. The beta blocker should NOT be abruptly stopped; the dose should be tapered down.

The nurse prepares to present the Be A.L.E.R.T campaign to colleagues. Which instructions are important to include? (select all that apply) a. Elevate head of bed to 90 degrees b. Wear gloves when handling feeding tube c. Label enteral equipment d. Verify that enteral tubing connects formula to feeding tube

b. Wear gloves when handling feeding tube c. Label enteral equipment d. Verify that enteral tubing connects formula to feeding tube

The client complains of night blindness. The nurse correctly recommends which food? a. Skim milk and peas b. Whole milk and eggs c. Nuts and yeast d. Enriched bread and cereal

b. Whole milk and eggs

1. A newly admitted client takes digoxin 0.25 mg/day. The nurse knows that which is the serum therapeutic range for digoxin?

b.0.5 to 2.0 ng/mL

7. A client is prescribed losartan (Cozaar). The nurse teaches the client that an angiotensin II receptor blocker (ARB) acts by doing what?

b.Blocking angiotensin II from AT1 receptors

-bital

barbiturate (sedative)

-lol

beta blocker adfx: bronchospasm

-ine

beta-adrenergic agonists (bronchodilate, increase vasc resist) ex: norepinephrine (Levophed), dopamine (Inotropin) used cardiac arrest, COPD adfx: anticholinergic, dysrhythmias

ANTIHYPERTENSIVES Alpha -1 Adrenergic Blockers

blocks alpha 1 receptors in the peripheral blod vessels (heart itself is NOT affected) uses: HTN, BPH (benign prostatic hypertrophy), side effects: reflex tachy cardia, orthostatic hypotension, UNIQUE: nasal congestion admin first dose at bedtime to avoid fainting !!! monitor BUN , weight and edema (may cause peripheral edema) monitor weight shift changes change positions slowly

-phylline

bronchodilator

A prenatal client tells the nurse that she is not taking vitamins because she heard that "vitamins may cause damage to my baby." What is the best response by the nurse? a. "vitamins can only help you and your baby" b. "take extra vitamins now to make up for missed doses" c. "megadoses of vitamins can be harmful in the first trimester" d. "taking above the RDA of any vitamin is not recommended"

c. "megadoses of vitamins can be harmful in the first trimester"

The client gained 10 pounds in 2 days. It is determined that the weight gain is caused by fluid retention. The nurse correctly estimates that the weight gain may be equivalent to how many liters of fluid? a. 2 b. 3 c. 4 d. 5

c. 4

2. A client states during a medical history that he takes several acetaminophen tablets throughout the day. The nurse teaches the client that the dosage should not exceed which amount? a. 1 g/day b. 2 g/day c. 4 g/day d. 6 g/day

c. 4 g/day

A pediatric client is ordered to receive 3 mg/kg of a medication. The client weighs 88 pounds. The medication is available in a 15 mg/mL elixir. How much medication should the client receive? a. 2 mL b. 4 mL c. 8 mL d. 16 mL

c. 8 mL

4. A client received spinal anesthesia. Which is most important for the nurse to monitor? a. Loss of consciousness b. Hangover effects and dependence c. Hypotension and headaches d. Excitement or delirium

c. Hypotension and headaches

5. A client is taking carisoprodol (Soma). Which statement would the nurse include in teaching the client about this drug? a. It may cause hypertension. b. It may lead to bradycardia. c. It blocks interneuronal activity. d. Its action is decreased by antihistamines.

c. It blocks interneuronal activity.

5. The nurse teaches a client about which common side effect of analeptics? a. Bradycardia b. Constipation c. Nervousness d. Urinary retention

c. Nervousness

2. The nurse realizes that some herbs interact with selective serotonin reuptake inhibitors (SSRIs). Which herb interaction may cause serotonin syndrome? a. feverfew b. ma-huang c. St. John's wort d. gingko biloba

c. St. John's wort

4. A client is taking rivastigmine (Exelon). The nurse should teach the client and family which information about rivastigmine? a. That hepatotoxicity may occur b. That the initial dose is 6 mg t.i.d c. That GI distress is a common side effect d. That weight gain may be a side effect

c. That GI distress is a common side effect

10. A client is admitted to the emergency department with an acute myocardial infarction. Which drug category does the nurse expect to be given to the client early for the prevention of tissue necrosis following blood clot blockage in a coronary or cerebral artery?

c. Thrombolytic agent

6. The nurse acknowledges that beta blockers are as effective as antianginals because they do what?

c.Decrease heart rate and decrease myocardial contractility

4. A client has heart failure and is prescribed Lasix. The nurse is aware that furosemide (Lasix) is what kind of drug?

c.High-ceiling (loop) diuretic

2. What would cause the same client's electrolyte imbalance?

c.Hydrochlorothiazide

-dipine

calcium channel blocker

The nurse is admitting a patient with a history of angina and hypertension who is currently experiencing moderate heart failure. The patient's current medication regimen includes digoxin, furosemide, and accupril. Which medication would be most beneficial to add to this patient's treatment plan?

carvedilol (Coreg)

Anticholinergics

cause the mucous membranes to become dry; therefore, oral secretions are decreased

what type of line is used for TPN

central line for 4 weeks

cef-, ceph-

cephalosporin antibiotic

cef-, ceph-

cephalosporin antibiotics (check allergy to this class and penicillins) * cefazolin, cefotetan, ceftriazone

ANTIGOUT AGENTS

cholchicine (colsalide)ONLY IN ACUTE , probenecid (benemid) CHRONIC, Allopurinol (Zyloprim) decreases production and action of uric acid RENAL CULCULI (KIDNEY STONES) AGRANULOCYTOSIS GI UPSET FLUIDS! UNIQUE: Take With food and/or milk (most antacids take WITHOUT FOOD, hence this med is unique)

anti-gout

cholsalide (Colchicine), benemid (Probenecid), Zyloprim (Allopurinol) decreases uric acid Probenecid increases action of Cipro Colchicine used only in acute gout syndrome Probenecid for chronic adfx: agranulocytosis, renal calculi-flush kidneys at treatment may take with food/milk/antacids

tetracycline

comonly prescribed for acne vuglgaris • Dental staining in children

CNS effects w/ Cinetidine (Tagament) occur in elderly; signs?

confusion

CNS impairment Assessment

confusion, delirium insomnia drowsiness halluncinations

zoloft

considered the first line of treatment in panic-anxiety disorders and mild to moderate depresssion

Miotics

constrict the pupil and are used in the treatment of clients with glaucoma

sal-; -sal-

contains salicylate (check allergies to salicylates or aspirin) * salsalate (nonopioid analgesic), sulfasalazine (GI anti-inflammatory)

Ibutilide (Corvert) is a class III antiarrythmnic drug that is used for

conversion of recent-onset of atrial fibrillation and flutter

-asone

corticosteroid

-asone, -solone

corticosteroid

-onide

corticosteroid

cort

corticosteroid

pred; pred-

corticosteroid

-sone; -lone; pred-

corticosteroid * betamethasone, cortisone, dexamethasone, prednisone

-olone

corticosteroid; anabolic steroid

what medication will increase BS tests and create a false reading of TPN therapy

corticosteroids

5. Which statement is true concerning lithium? a. The maximum dose is 3.4 g/day. b. The therapeutic drug range is 2.5 to 3.5 mEq/L. c. Lithium increases receptor sensitivity to GABA. d. Concurrent NSAIDs may increase lithium levels.

d. Concurrent NSAIDs may increase lithium levels.

Chapter 20 1. When a 12-year-old child is prescribed methylphenidate, which is most important for the nurse to monitor? a. The child's temperature b. The child's respirations c. The child's intake and output d. The child's height and weight

d. The child's height and weight

The nurse is reviewing the client's laboratory results and current medications. The nurse notes that the client's prothrombin time is prolonged. What vitamin might be contributing to this? a. Vitamin A b. Vitamin B c. Vitamin D d. Vitamin E

d. Vitamin E

decongestant

decrease the amount of mucus secreation from the bronchi ,reduces congestion and swelling of membranes, such as those of the nose and eustachian tube in an infection

Diuretics

decrease water and sodium absorbed from the loop of Henle (loop diuretics) or inhibit antidiuretic hormone (potassium-sparing diuretics)

Pancrelipase is a pancreatic enzyme used w/ patients w/ pancreatitis as a

digestive aid to reduce fatty stool & improve nutritional status

The nurse reviews a patient's laboratory values and finds a digoxin level of 10 and a serum potassium level of 6.2. The nurse would notify the health care provider and anticipate administering

digoxin immune FAB

sublingual nitroglycerine

dilate blood vessels and increase circulation

Mydriatics

dilate the pupils and are used in the treatment of clients with cataracts

if TPN solutions are behind schedule what is to be done

do not catch up set at normal rate

adverse reactions to skeletal muscle relaxants

drowsiness, incoordination, GI upset

expectorant

drug that breaks up mucus and promotes coughing

cromolyn

drug used to prevent asthma attacks or decrease in allergic response

-afil

erectile dysfunction

assess temperature every __ hours for presence of infection

every 4 hours

how often should BS levels be monitored for hyperglycemia while on TPN

every 6 hours

side effects would be expected with elderly patients taking barbiturates

excitement confusion depression

Superinfection Nursing Care

frequent mouth care good skin care antifungal meds

Stomatitis nursing considerations

frequent mouth care with non alcoholic solution frequent, small feedings-non spicy foods antifungal meds

glucose laboratory test measures?

glucose

how much dextrose may be used for a TPN

greater than 10%

sign of digoxin toxicity

green yellow vision

once TPN is completed and new bag is not available what should be hung instead to prevent hypoglycemia

hang a 10% dextrose bag

Adverse reactions to bethanechol include

headache

adverse reactions to hypothroid

headache and insominia

adverse reactions of uterine relaxants

heart palpitations, nausea, vomiting, headache

lasix

helps decrease edema in the body

-tidine

histamine H2 antagonist (GI), antiulcer * cimetidine, famotidine, ramitidine

-tidine

histamine H2 antagonist (GI), antiulcer * cimetidine, famotidine, ranitidine

anaphylaxis Assessment

hives, rash difficulty breathing dilated pupils decreased BP, increased pulse, increased respirations

Dermatological Reactions Assessment

hives, rashes, lesions Exfoliate dermatitis (rash, enlarged lymph nodes, enlarged lives) Stevens-Johnson Syndrome( dark red papules that dont itch or hurt)

epinepherine

hormone that speeds up heart

Aluminum hydroxide is an antacid that treats

hyperphosphatemia

INSULIN side effects

hypoglycemia monitor BGS short term -prick long term - HA1C

Aluminum hydroxide is an antacid that causes

hypophosphatemia

Antianemics

increase factors necessary for red blood cell production

ANTICHOLINERGIC

inhibit action of acetylcholine, blocks parasympathetic nervous system action: dilates pupil, causes bronchodilation and decreases secretions (often used preoperatively), decreased mobility and GI secretions Causes some hypotension, urinary retention, changes in heart rate, dry mouth, blurred vision. MONITOR OUTPUT CONTRAINDICATED IN GLAUCOMA (because of pupil dilation interferes with othalmic fluid drainage) GIVE 30min bac meals hs, or 2hr pc CONTRAINDICATED WITH: PARALYTIC ILLEUS (would decrease motility even more) BENIGN PROSTHETIC HYPERTROPHY (urinary retention issue)

Histamine 2 antagonist

inhibit gastric acid

like blood products being infused what type of tubing should be used for TPN

inline filter

what two meds may be added to TPN solution to prevent problems

insulin and or heparin

Lithium

is a salt and has a lot of rules. Dangerous drug to use, controls manic phase and must assess client as to where client is at in the cycle of bipolar. dosages change all the time very tight therapeutic range blood drawn q 2-3 weeks, inspect pt q 2weeks FLUIDS IMPORTANT MUST TAKE 3000 ml /day Dehydration cashes severe response

benadryl

is an anti-hystemine that decreases itching

apresoline

is an antihypertensive medication

A patient is admitted to the hospital with pneumonia and has a history of chronic renal insufficiency. The physician orders furosemide (Lasix) 40 mg twice a day because it

is effective in treating patients with renal insufficiency

gantrisin

is the most common anti-biotic to treat UTI's

imodium

is used to treat diarrhea

Questron therapy is use not compliant due to

its terrible taste; encourage to drink w/ fruit juice

what should the BS maintained while on TPN

less than 200 mg/dL

what is the only solution to be able to piggyback a TPN

lipids

grapefruit juice interaction

lipitor, viagra, ...

meds for bipolar disorder

lithium, Tegretol, Depakote block catecholamines Lithium salt is dangerous: tight therapeutic range. blood drawn 2-3 weeks. 3000 mL water/day. evaluated q2-3 wk Tegretol, Depakote: antipsychotic/convulsant watch kidney and liver adfx: GI upset, Tremors, polydipsia, polyuria take with food.

-caine

local anesthetic

Antihypertensives

lower blood pressure and increase blood flow th the myocardium

admininster eye drops by dropping

lower conjuctiva

Cholesterol-lowering

lowers cholesterol and triglyceride levels

these two blood tests if abnormally high may indicate problems with glucose or protein metabolism or excess lipids

lymphocyte, liver function

streptase

lysis of thrombi in acute myocardial infarction

-omycin

macrolide, antibiotic * arithromycin, azithromycin

specificgravity

measures a patients hydration status

electrolyte laboratory test measures?

measures potassium

cipro

medication given for UTI, bone, joint, skin infectons

cogentin

medication used to treat parkinson's disease

direct acting vasodilators

minoxidil, hydralazine (Apresoline)-relax smooth muscle uses: HTN minoxidil topical is Rogaine adfx: ortho hypotn, tachycardia (may use with B-blocker to prevent), increased hair check pulse when taking med

Bone Marrow Depression Nursing Considerations

monitor CBC rest protection from infections avoid activities that may cause injury

Ocular Impairment Nursing Considerations

monitor vision carefully monitor lighting and exposure to light

-nacin

muscarinic antagonist (anticholinergic)

The nurse would plan to administer which calcium channel blocking drug to a patient with cerebral arterial spasms following a subarachnoid hemorrhage?

nimodipine (Nimotop)

what is not to be added to TPN solutions

no blood or meds

Antivirals

no virus

Propranolol (Inderal) is an effective

nonselective beta-adrenergic antagonist

-fenac

nonsteroidal anti-inflammatory drug (NSAID)

-profen

nonsteroidal anti-inflammatory drug (NSAID)

lovenox is contraindicated in clients

on a kosher diet due to the pork content

protease inhibitor is best taken

on an empty stomach 1hr b4 or 2hrs. after a meal

Humulin 70/30

onset 0.5 hr peak 2-12 hrs duration 24 hrs used for client not needed to mix Looks at hemoglobin A1C (2-3 month sugar review)

arterial blood gases lab test measures what?

oxygen and pH levels

the desired effect of morphine is

pain relief

-cillin

penicillin antibiotic

-cillin

penicillin, antibiotic (check allergy) * amoxicillin, piperacillin, oxacillin

what type of line is used for PPN

peripheral line for ten days

-zosin

peripherally acting alpha-1 blocker, antihypertensive * terazosin

During assessment of a patient diagnosed with pheochromocytoma, the nurse auscultates a blood pressure of 210/110 mm Hg. The nurse would expect to administer which medication?

phentolamine (Regitine)

-afil

phosphodiesterase (PDE) inhibitor

Which would the nurse question if ordered concurrently with ACE inhibitors?

potassium

patient on lasix may need

potassium supplements, to weigh self each wk. and change positions slowly to prevent dizziness, orthrostatic hypotension is a side effect of diuretic

Calcium channel blocers reduce myocardial oxygen demand by reducing afterload, which is

pressure against which the heart must pump

heparin

prevents blood clotting

Laxatives

promote the passage of stool

Before emergency surgery, the nurse would anticipate administering which medication to a patient receiving heparin?

protamine sulfate

the antidote for heparin

protamine sulfate

-prazole

proton pump inhibitor

-eprazole

proton pump inhibitor (PPI)

-oprazole

proton pump inhibitor (PPI)

tretin-; tretin; -tretin

retinoid; dermatologic agent; form of vitamin A

narcoan is given to overcome a narcotic overdose by

reverse CNS and respiratory depression

Parkinson-like Nursing Considerations

safety measure for gait changes anticholinergic/antiparkinson meds

CNS impairment Nursing Considerations

safety measures to prevent injury avoid activities that require alertness(driving a car) orient to surrounding frequently

General side effects and nursing consideration of bipolar meds

side effects: GI upset, tremors, polydispsia, polyuria monitor serum levels: Lithium salt most important GIVE WITH MEALS increase fluid intake Fluid fluid fluid especially with lithium

Delayed Allergic Reaction Nursing Considerations

skin care antihistamines, topical corticosteroids

The nurse would expect to administer which diuretic to treat a patient diagnosed with primary hyperaldosteronism?

spironolactone (Aldactone)

ANTIHYPERTENSIVES Centrally acting alpha-adrenergics

stimulates receptors in the medulla decrease rate and force of heart uses HTN drugs: Aldomet, Catapres side effects: hypotension UNIQUE: sedation fluid retention check BO and pulses

neupogen

stimulates the production of neutraphils

oxytocin

stimulates uterine contraction

how is TPN solution stored

store in fridge

Proton pump inhibitors

suppress gastric secretions

Glucocorticoid

suppress immune system, anti-inflammatory, anti-allergy, anti-stress

Nonselective beta-blockers may be used to treat hypertension and

supraventricular arrhythmias

Stomatitis Assessment

swollen gums swollen red tounge bad breath difficulty swallowing

The nurse would question the use of milrinone (Primacor) in a patient with which disorder?

systolic heart failure

an adverse reaction to atropine sulfate

tachycardia

carafate

the action of this drrug is to line and protect the stomach, it is better able to do so if the medication can come in contact with the stomach

Herbal Consideration with anticoagulants

the big three G's Garlic Ginger Ginko all will INCREASE the effect of the meds

tolerance

the diminishing effect with regular use of the same dose of a drug, requiring the user to take larger and larger doses before experiencing the drug's effect

cumulative

the drug is not completly metabolized and is excreted before the next dose is given

inderal

the first beta blocker (trade name Inderal) used in treating hypertension and angina pectoris and essential tremor

When teaching a patient why spironolactone (Aldactone and furosemide (Lasix) are prescribed together, the nurse's statements are based on knowledge that

this combingation promotes diuresis but decreases the risk of hypokalemia

urecholine

this drug produces smooth muscle contraction(bladder tone) and is used for abdominal and urinary retention

brevicon

this medication is a contraceptive agent

depo-provera

this medication suppresses endometrial bleeding

troche

throat lozenger

-ase

thrombolytic

INSULIN

type 1 insulin dependent client (can be used with type 2 sometimes) Pancreas does not produce any Insulin If used with type 2, then used to augment not enough production of insulin by pancreas.

avandia

type 2 diabetes mellitus , oral diabetic drug doesn't produce more insulin, only gets glucose level to norm

Misoprostol is a gastric protectant and is given to prevent

ulcers w/ NSAID and aspirin use

Anti-infectives

used for the treatment of infections:

Phosphodiesterase inhibitors have an added advantage in treating hear failure because they cause a positive inotropic effect and

vasodilation

the antidote for coumadin

vit k

A patient who has been anticoagulated with warfarin (Coumadin) is admitted with gastrointestinal bleeding. The nurse will anticipate administering which substance?

vitamin K

an important nursing interverntion while taking lasix

weights and vital signs. baseline is needed to determine the effectiveness of therapy

ANTIDEPRESSANTS: heterocyclics

wellbutrin, desyrel uses: depression and smoking cessation UNIQUE side effects: wellbutrin: agitation and insomnia desyrel: sedation avoid alcohol and CNS depressants, wean off slowly

ANTIHYPERTENSIVES (several groups)

will affect BP (HTN) side effect hypo tension

decadron is best given

with a glass of milk to decrease gastric distress

corticosteriods is taken

with or after meals

-phylline

xanthine derivative (bronchodilator)

-phylline

xanthine type of bronchodilator * theophylline

how is TPN tube placement verified

xray

ending in: sone or lone beginning with: pred

Corticosteriod. Contraindicated in systemic infections, UC, diverticulitis, active or latent PUD, and pregnancy/lactation (safety not established). Side effects include euphoria, h/a, insomnia, psychosis, edema, muscle weakness, delayed wound healing, osteoporosis, spontaneous fractures, & hyperglycemia. Nursing considerations: Oral forms should be given with meals. IV form should be given slowly IVPB to prevent vaginal & anal burning. Establish baseline & montior BP, I & O, weight & sleep patterns. Measure 2h post-meal blood glucose, potassium & calcium prior to tx & regularly thereafter. Watch for changes in mood, emotional stability, & sleep. Interact with phenytoin, rifampin, amphotericin-B, diuretics, vaccines & toxoids.

b. Dizziness c. Headache e. Ankle edema

During an admission assessment, the client states that she takes amlodipine (Norvasc). The nurse wishes to determine whether or not the client has any common side effects of a calcium channel blocker. The nurse asks the client if she has which signs and symptoms? (Select all that apply.) a. Insomnia b. Dizziness c. Headache d. Angioedema e. Ankle edema f. Hacking cough

ANTIDEPRESSANTS (MAOI's)

Marplan, Parnate, Nardil side effects: HYPERTENSIVE CRISIS when taken with tyramine containing foods photo sensitivity potentiate alcohol AVOID wine, pickled products, herring, aged cheeses, bologna, pepperoni, salami, bananas, raisins, beer, sour cream What if patient is on lasixs as an antidiuretic who is told to eat bananas=conflict=monitor closely Take a GOOD assessment monitor output takes 4 weeks to work TEACHING with client that it takes time to work Do not take with CNS stimulants or cold medicines (teach: list of correct cold meds) MAOI's with thyramine foods hypertensive crisis: severe headache , palpitations, stiff neck, diaphoretic intracranial hemorrhage

Avodart

Men's Health Medications

Cialis

Men's Health Medications

Sodium Nitroprusside (Nitropress)

Used to treat shock, cardiac arrest and anaphylaxis

The older adult client has questions about oral drug metabolism. Which information should be included in this client's teaching plan? a. First-pass effect b. Enzyme function c. Glomerular filtration rate d. motility

a. First-pass effect

Nursing responsibilities in the assessment phase of the nursing process include which responsibilities? (Select all that apply) a. Identify side effects of drugs that are nonspecific b. Check peak and trough levels of drugs c. Advise client to avoid fatty foods prior to ingesting an enteric coated tablet d. Evaluate client's reaction to drug

a. Identify side effects of drugs that are nonspecific b. Check peak and trough levels of drugs

A medication health teaching plan is tailored to a specific client. Common topics for health teaching include which? (select all that apply) a. Importance of adherence to the prescribed regimen b. How to administer medication(s) c. What side/adverse effects to report to the health care provider d. Instruction of the client on which foods should be eaten

a. Importance of adherence to the prescribed regimen b. How to administer medication(s) c. What side/adverse effects to report to the health care provider

The client is receiving fluid replacement. The nurse's health teaching with this client includes which suggestions? (select all that apply) a. Measure weight daily b. Know that thirst means a mild fluid deficit c. Monitor fluid intake d. Avoid the use of calcium supplements

a. Measure weight daily b. Know that thirst means a mild fluid deficit c. Monitor fluid intake

7. A client is taking lithium. The nurse should be aware of the importance of which nursing intervention(s)? (Select all that apply.) a. Observe the client for motor tremors. b. Monitor the client for orthostatic hypotension. c. Draw lithium blood levels immediately after a dose. d. Advise the client to drink 750 mL/day of fluid in hot weather. e. Advise the client to avoid caffeinated foods and beverages, such as coffee, tea, colas, and chocolate. f. Teach the client to take lithium with meals to decrease gastric irritation.

a. Observe the client for motor tremors. b. Monitor the client for orthostatic hypotension. e. Advise the client to avoid caffeinated foods and beverages, such as coffee, tea, colas, and chocolate. f. Teach the client to take lithium with meals to decrease gastric irritation.

7. A client is taking tacrine (Cognex) to improve cognitive function. What should the nurse teach the client? (Select all that apply.) a. That the client should rise slowly to avoid dizziness b. That obstacles should be removed from pathways to avoid injury c. That the drug dosing schedule should be followed closely d. That the client should be checked frequently for hypertension e. That the client should receive regular liver function tests

a. That the client should rise slowly to avoid dizziness b. That obstacles should be removed from pathways to avoid injury c. That the drug dosing schedule should be followed closely e. That the client should receive regular liver function tests

6. A client who is prescribed pyridostigmine bromide (Mestinon) is being taught about the drug. Which statements should the nurse include in the teaching? (Select all that apply.) a. The drug must be taken on time. b. The drug must be taken two times per day. c. Underdosing can result in cholinergic crisis. d. Overdosing can result in cholinergic crisis. e. The client should report the adverse effects of tachycardia to the health care provider.

a. The drug must be taken on time. c. Underdosing can result in cholinergic crisis. d. Overdosing can result in cholinergic crisis.

6. What should the client who is taking anticholinergic therapy for parkinsonism be taught? (Select all that apply.) a. To avoid alcohol, cigarettes, and caffeine b. To relieve dry mouth with hard candy or ice chips c. To use sunglasses to reduce photophobia d. To urinate 2 hours after taking the drug e. To receive routine eye examinations

a. To avoid alcohol, cigarettes, and caffeine b. To relieve dry mouth with hard candy or ice chips c. To use sunglasses to reduce photophobia e. To receive routine eye examinations

The client comes to the office with chief complaint of hair loss and peeling skin. The nurse notes that many vitamins are on the list of medications that the client reports using to treat liver disease. The client's complaint may be caused by excess of what vitamin? a. Vitamin A b. Vitamin B c. Vitamin C d. Vitamin D

a. Vitamin A

4. An atypical antipsychotic is prescribed for a client with psychosis. The nurse understands that this category of medications includes which drug? a. clozapine (Clozaril) b. loxapine (Loxitane) c. haloperidol (Haldol) d. thiothixene (Navane)

a. clozapine (Clozaril)

5. A client is admitted to the emergency department with status epilepticus. Which drug should the nurse most likely prepare to administer to this client? (Select all that apply.) a. diazepam (Valium) b. midazolam (Versed) c. gabapentin (Neurontin) d. levetiracetam (Keppra)

a. diazepam (Valium) b. midazolam (Versed)

6. The nurse who is teaching the client to self-administer medications explains to the client that which drug treats narcolepsy? a. modafinil b. atomoxetine c. lisdexamfetamine d. methylphenidate

a. modafinil

5. Assessing a client following IV morphine administration, the nurse notes cold, clammy skin; a pulse of 40 beats/min; respirations of 10 breaths/min; and constricted pupils. Which medication will the client likely need next? a. naloxone (Narcan) b. meloxicam (Mobic) c. pentazocine (Talwin) d. propoxyphene (Darvon)

a. naloxone (Narcan)

8. A client who has angina is prescribed nitroglycerin. The nurse reviews which appropriate nursing interventions for nitroglycerin? (Select all that apply.)

a.Have client lie down when taking a nitroglycerin sublingual tablet. b.Teach client to repeat taking a tablet in 5 minutes if chest pain persists. e.Warn client against ingesting alcohol while taking nitroglycerin.

anticonvulsant

adfx: gingival hypertrophy, resp depression, aplastic anemia turn urine pink or reddish brown Dilantin: can cause cardiac arrest if pushed too fast CANNOT BE MIXED WITH ANOTHER MED no carbonation with depakote

glucophage

an antidiabetic drug prescribed to treat type II diabetes the action: decreases cellular resistance to insulin

4. A client is taking tranylcypromine sulfate (Parnate) for depression. What advice should the nurse include in the teaching plan for this medication? a. Warn of severe hypotension. b. Avoid beer and cheddar cheese. c. Encourage ginseng and ephedra. d. Encourage fruit such as bananas.

b. Avoid beer and cheddar cheese.

Chapter 24 1. When the nurse explains the pathophysiology of myasthenia gravis to a client, which is the best explanation? a. Degeneration of cholinergic neurons and a deficit in acetylcholine leads to neuritic plaques and neurofibrillary tangles. b. Decreased amount of acetylcholine to cholinergic receptors produces weak muscles and reduced nerve impulses. c. Myelin sheaths of nerve fibers in brain and spinal cord develop lesions or plaques. d. Imbalance of dopamine and acetylcholine leads to degeneration of neurons in midbrain and extrapyramidal motor tracts.

b. Decreased amount of acetylcholine to cholinergic receptors produces weak muscles and reduced nerve impulses.

During a medication review session, a client comments, "I just do not know why I am taking all of these pills." This comment suggests which nursing diagnosis? a. Risk for injury b. Deficient knowledge c. Risk for aspiration d. anxiety

b. Deficient knowledge

7. A client is taking warfarin 5 mg/day for atrial fibrillation. The client's international normalized ratio (INR) is 3.8. The nurse would consider the INR to be what?

b. Elevated INR range

The nurse is aware of the many factors related to effective health teaching about the medication. The most essential component of the teaching plan is to do which? a. Provide written instructions b. Establish a trust relationship c. Use colorful charts d. Review community resources

b. Establish a trust relationship

3. A client is taking zolpidem (Ambien) for insomnia. The nurse prepares a care plan that includes monitoring of the client for side effects/adverse reactions of this drug. Which is a side effect of zolpidem? a. Insomnia b. Headache c. Laryngospasm d. Blood dyscrasias

b. Headache

The nurse has been measuring the blood pressure of an African-American client every 4 hours for the past 3 days in a hospital setting. The blood pressure is consistently above 140/90. The client has been compliant with the antihypertensive drug therapy while hospitalized. The nurse will initially perform which action? a. Question the client about types of food consumes in the past 3 to 4 days b. Inform the prescriber that the antihypertensive drug therapy is not working c. Increase blood pressure measurements to every 2 hours d. Place the client on a restricted fluid intake

b. Inform the prescriber that the antihypertensive drug therapy is not working

5. Nursing interventions for the client taking carbidopa-levodopa for parkinsonism include which? a. Encouraging client to adhere to a high-protein diet b. Informing client that perspiration may be dark and stain clothing c. Advising client that glucose levels should be checked through urine testing d. Warning client that it may take 4 to 5 days before symptoms are controlled

b. Informing client that perspiration may be dark and stain clothing

The client is receiving potassium supplements. What is the most important nursing implication when administering this drug? a. It cannot be given as an IV bolus b. It must not be diluted c. It b=must be chilled before administration d. It must be given only at bedtime

b. It must not be diluted

The nurse educator on the unit receives a list of high-alert drugs. Which strategies are recommended to decrease the risk of errors with these medications? (select all that apply) a. Store medications alphabetically on their usual shelf b. Limit access to these drugs c. Use special labels d. Provide increased information to the staff

b. Limit access to these drugs c. Use special labels d. Provide increased information to the staff

An Asian client is being treated in the emergency department for a fractured right ankle. The physician that after administration of the codeine, the client which experience which response? a. Quick relief of the pain b. Little relief of the pain c. Idiosyncratic responses d. Signs of anaphylaxis

b. Little relief of the pain

5. A nurse is teaching a client about zolpidem. Which is important for the nurse to include in the teaching of this drug? a. Maximum dose is 20 mg/d b. May lead to psychological dependence c. For older adults, dose is 15 mg at bedtime d. Should only be used for 21 days or less

b. May lead to psychological dependence

2. Phenytoin (Dilantin) has been prescribed for a client with seizures. The nurse should include which appropriate nursing intervention in the plan of care? a. Reporting an abnormal phenytoin level of 18 mcg/mL b. Monitoring CBC levels for early detection of blood dyscrasias c. Encouraging the client to brush teeth vigorously to prevent plaque buildup d. Teaching the client to stop the drug immediately when passing pinkish-red or reddish-brown urine

b. Monitoring CBC levels for early detection of blood dyscrasias

The nurse routinely includes health teaching about vitamins to clients/ vitamin D has a major role in which process? a. Ensuring night and color vision b. Regulating calcium and phosphorous metabolism c. Body growth d. DNA and prothrombin synthesis

b. Regulating calcium and phosphorous metabolism

6. A client is receiving haloperidol (Haldol). Which nursing intervention(s) should the nurse perform? (Select all that apply.) a. Monitor vital signs to detect bradycardia. b. Remain with the client until medication is swallowed. c. Monitor vital signs to detect orthostatic hypotension. d. Assess the client for evidence of neuroleptic malignant syndrome. e. Observe the client for acute dystonia, akathisia, and tardive dyskinesia.

b. Remain with the client until medication is swallowed. c. Monitor vital signs to detect orthostatic hypotension. d. Assess the client for evidence of neuroleptic malignant syndrome. e. Observe the client for acute dystonia, akathisia, and tardive dyskinesia.

4. The nurse realizes that which is the laboratory test ordered to determine the presence of the amino acid that can contribute to cardiovascular disease and stroke?

b. homocysteine

1. A client is taking hydrochlorothiazide 50 mg/day and digoxin 0.25 mg/day. What type of electrolyte imbalance does the nurse expect to occur?

b.Hypokalemia

terbutaline

beta1 direct agonist. Reduces premature uterine contractions

adverse reactions to hyperthyroid medications

bronchospasms, iodism, weight gain and sleeping patterns increase

4.Atenolol (Tenormin) is prescribed for a client. The nurse realizes that this drug is a beta-adrenergic blocker and that this drug classification is contraindicated for clients with which condition? a. Hypothyroidism b. Angina pectoris c. Cardiogenic shock d. Liver dysfunction

c. Cardiogenic shock

3. A client has spasticity following a spinal cord injury. The nurse should expect which drug to be prescribed to treat this client's spasticity? a. Tacrine b. Ropinirole c. Carisoprodol d. Pyridostigmine

c. Carisoprodol

The nurse reviews the client's list of medications and results of laboratory tests. Which drug type may cause an elevated serum sodium level? a. Antifungals b. Oral contraceptives c. Cortisone preparations d. antiepileptics

c. Cortisone preparations

The client asks the nurse about fat-soluble vitamins. What is the nurse's best response? a. Fat-soluble vitamins are metabolized rapidly b. Fat-soluble vitamins cannot be stored in the liver c. Fat-soluble vitamins are excreted slowly in urine d. Fat-soluble vitamins cannot be toxic

c. Fat-soluble vitamins are excreted slowly in urine

A Native American client is newly diagnosed with diabetes mellitus type 2 and is prescribed the antidiabetic drug metformin (Glucophage) 500 mg PO with morning and evening meals. Which statement best indicated to the nurse that the client will adhere to the pharmacotherapy? a. "I will no longer put sugar on my cereal" b. "I will feel better soon if I take this medicine" c. I need to take the medicine as scheduled to reduce the possibility of damage to my body" d. "I have diabetes because of my ancestry"

c. I need to take the medicine as scheduled to reduce the possibility of damage to my body"

2. A client taking lorazepam (Ativan) asks the nurse how this drug works. The nurse should respond by stating that it is a benzodiazepine that acts by which mechanism? a. Depressing the central nervous system (CNS), leading to a loss of consciousness b. Depressing the CNS, including the motor and sensory activities c. Increasing the action of the inhibitory neurotransmitter gamma-aminobutyric acid (GABA) to GABA receptors d. Creating an epidural block by placement of the local anesthetic in the outer covering of the spinal cord

c. Increasing the action of the inhibitory neurotransmitter gamma-aminobutyric acid (GABA) to GABA receptors

Chapter 22 1. The nurse witnesses a client's seizure involving generalized contraction of the body followed by jerkiness of arms and legs. The nurse reports that this is which type of seizure? a. Myoclonic b. Petit mal c. Tonic clonic d. Psychomotor

c. Tonic clonic

A 4-year old client is to be discharged home on an oral liquid suspension of 4mL per dose. Which would the nurse recommend to ensure the highest level of accuracy in home administration of the medication? a. Using a household teaspoon b. Using a cooking measuring spoon c. Using an oral syringe d. Using a graduated medicine cup

c. Using an oral syringe

1. A client has a serum cholesterol level of 265 mg/dL, triglyceride level of 235 mg/dL, and LDL of 180 mg/dL. What do these serum levels indicate?

c.Hyperlipidemia

2. A client is receiving carbidopa-levodopa for parkinsonism. What should the nurse know about this drug? a. Carbidopa-levodopa may lead to hypertension. b. Carbidopa-levodopa may lead to excessive saliva. c. Dopaminergic and anticholinergic therapy may lead to drowsiness and sedation. d. Dopaminergics and anticholinergics are contraindicated in clients with glaucoma.

d. Dopaminergics and anticholinergics are contraindicated in clients with glaucoma.

6. A client is given epinephrine (Adrenalin), an adrenergic agonist (sympathomimetic). The nurse should monitor the client for which condition? a. Decreased pulse b. Pupil constriction c. Bronchial constriction d. Increased blood pressure

d. Increased blood pressure

The nurse reviews the client's list of medications with the client. The nurse knows that the 88-year old client's slower absorption or oral medications is primarily because of which phenomenon? a. Decreased cardiac output b. Decreased blood flow c. Decreased enzyme function d. Increased pH of gastric secretions

d. Increased pH of gastric secretions

6. The nurse is administering a cholinergic agonist and should know that the expected cholinergic effects include which of the following? a. Increased heart rate b. Decreased peristalsis c. Decreased salivation d. Increased pupil constriction

d. Increased pupil constriction

4. Dicyclomine (Bentyl) is an anticholinergic, which the nurse realizes is given to treat which condition? a. Mydriasis b. Constipation c. Urinary retention d. Irritable bowel syndrome

d. Irritable bowel syndrome

The client's goals have been met during hospitalization. At the time of discharge, which nursing diagnosis is most probable? a. Knowledge deficient b. Ineffective coping c. Readiness for enhanced social interaction d. Readiness for enhanced self-care activities

d. Readiness for enhanced self-care activities

It is essential for the client who self-administers the enteral feedings to know that the feeding should be administered at which temperature? a. Slightly warmed b. Chilled c. Ice cold d. Room temperature

d. Room temperature

3. A nurse is teaching a client who has diabetes mellitus and is taking hydrochlorothiazide 50 mg/day. The teaching should include the importance of monitoring which levels?

d. Serum glucose (sugar)

The client is due to receive Kayexalate for complaints of nausea, vomiting, abdominal cramps, short QT interval, weakness, and oliguria. The nurse is aware that this drug is used to treat which imbalance? a. Hypocalcemia b. Severe hypercalcemia c. Hypokalemia d. Severe hyperkalemia

d. Severe hyperkalemia

Antidiarrheals

decrease gastric mobility and reduce water content in the intestinal tract

When assessing for therapeutic effects of mannitol, the nurse would expect to see

decreased intracranial pressure

barbituates

drugs that depress the activity of the central nervous system, reducing anxiety but imparing memory and judgement induces sleepiness

Renal Impairment Assessment

elevated BUN, CR Altered K, Na Decreased Hct Edema Fatigue

digoxin is given to treat

elevated heart rate in chf client

The nurse administering donepezil (Aricept) to a patient understands that the expected therapeutic action of this drug is to

increase levels of acetylcholine in the brain by blocking its breakdown

what position is client to be in if air embolism occurs during TPN

left trendlenberg

-cillin

penicillin-inhibits synthesis of cell wall severe infx-syphillis, lyme disease... adfx: stomatitis, allergy take without food

The nurse understands that a patient's international normalized ratio (INR) value of 2.5 means that

the patient's warfarin dose is therapeutic

-thiazide

thiazide diuretic (water pill)

-ase; -plase

thrombolytic agent (-ase usually indicates enzyme) * streptokinase, alteplase, reteplase

While preparing a patient with acute chest pain for an emergency angioplasty, the nurse would

tirofiban (Aggrastat)

vistaril

tused as an antiemetic or in higher doses as a tranquilizer

triple therapy of h. pylori infection include

two antibacterial drugs and a proton pump inhibitor

During postoperative teaching, the nurse explains that the patient is receiving bethanechol (Urecholine) to treat

urinary atony

tricyclic antidepressant

used also for sleep apnea amitryptalin (Elavil), imipramine (Tofranil), desipramine (Norpramin) adfx: postural hypotension, confusion, urinary retention suicide risk take at hs 2-6 weeks for effectiveness wean off no alcohol, care in sun

PENICILLINS (antibiotics)

used for more significant infections such as lyme disease. syphilis, Family of "cillin's" MOST ALLERGIES (SEVERE: ANAPHYLACTIC=ABCs) ASSESS HX OF REACTION BEFORE ADMINISTRATION CROSS ALLERGY WITH CEPHALASPORIN Renal and Liver damage BUN, AST ALT Urine output

aminophylline

used in bronchoconstriction broncial asthma and chronic obstructive pulmonary disease, and congestive heart failure, relaxes smooth muscle of the respiratory tract

glucagon

used to reverse hypoglycemia change glycogen to glucose and release it to the body adfx: hypotn, bronchospasm. may repeat in 15 minutes. once alert, give longer acting carbohydrates

leukine

useful in treating patients with bone marrow transplant

SULFONAMIDES (antibiotics)

uses: ulcerative colitis, crohns disease, otitis media UUTI's and upper respiratory names: bactrim, Gantrisin, azulfidine often seen with other antibiotics UNIQUE SIDE EFFECTS peripheral neuropathy, crystalluria, photosensitivity, GI upset, stomatitis Take with meals and food !!!!!!!!!!!!!!!!!!!!!! FLUIDS IMPORTANT 3000ml/day ! MOUTH CARE due to stomatitis MOUTH CARE MOUTH CARE ANTIACIDS WILL INTERFERE WITH UPTAKE

whats the medication of choice for an alcoholic going through withdrawls

valium vit b1 and b12 and folic acid

what does the client need to do when a tube or cap is changed on TPN

valsalva maneuver

-triptan

vascular headache suppressant, serotonin (5HT-1) agonist * sumatriptan

The nurse recognized that patient teaching regarding warfarin (Coumadin) has been successful when the patient acknowledges an increased risk of bleeding with conccurent use of which herbal product? Select all that apply

*Garlic *Ginko *Ginseng

Potassium-sparing diuretics may cause which adverse reactions? Select all that apply

*Hyperkalemia *Dizziness *Headache

31.) A community health nurse visits a client at home. Prednisone 10 mg orally daily has been prescribed for the client and the nurse reinforces teaching for the client about the medication. Which statement, if made by the client, indicates that further teaching is necessary? 1. "I can take aspirin or my antihistamine if I need it." 2. "I need to take the medication every day at the same time." 3. "I need to avoid coffee, tea, cola, and chocolate in my diet." 4. "If I gain more than 5 pounds a week, I will call my doctor."

1. "I can take aspirin or my antihistamine if I need it." Rationale: Aspirin and other over-the-counter medications should not be taken unless the client consults with the health care provider (HCP). The client needs to take the medication at the same time every day and should be instructed not to stop the medication. A slight weight gain as a result of an improved appetite is expected, but after the dosage is stabilized, a weight gain of 5 lb or more weekly should be reported to the HCP. Caffeine-containing foods and fluids need to be avoided because they may contribute to steroid-ulcer development.

179.) A nurse provides medication instructions to a client who had a kidney transplant about therapy with cyclosporine (Sandimmune). Which statement by the client indicates a need for further instruction? 1. "I need to obtain a yearly influenza vaccine." 2. "I need to have dental checkups every 3 months." 3. "I need to self-monitor my blood pressure at home." 4. "I need to call the health care provider (HCP) if my urine volume decreases or my urine becomes cloudy."

1. "I need to obtain a yearly influenza vaccine." Rationale: Cyclosporine is an immunosuppressant medication. Because of the medication's effects, the client should not receive any vaccinations without first consulting the HCP. The client should report decreased urine output or cloudy urine, which could indicate kidney rejection or infection, respectively. The client must be able to self-monitor blood pressure to check for the side effect of hypertension. The client needs meticulous oral care and dental cleaning every 3 months to help prevent gingival hyperplasia.

77.) Phenytoin (Dilantin), 100 mg orally three times daily, has been prescribed for a client for seizure control. The nurse reinforces instructions regarding the medication to the client. Which statement by the client indicates an understanding of the instructions? 1. "I will use a soft toothbrush to brush my teeth." 2. "It's all right to break the capsules to make it easier for me to swallow them." 3. "If I forget to take my medication, I can wait until the next dose and eliminate that dose." 4. "If my throat becomes sore, it's a normal effect of the medication and it's nothing to be concerned about."

1. "I will use a soft toothbrush to brush my teeth." Rationale: Phenytoin (Dilantin) is an anticonvulsant. Gingival hyperplasia, bleeding, swelling, and tenderness of the gums can occur with the use of this medication. The client needs to be taught good oral hygiene, gum massage, and the need for regular dentist visits. The client should not skip medication doses, because this could precipitate a seizure. Capsules should not be chewed or broken and they must be swallowed. The client needs to be instructed to report a sore throat, fever, glandular swelling, or any skin reaction, because this indicates hematological toxicity.

115.) A client received 20 units of NPH insulin subcutaneously at 8:00 AM. The nurse should check the client for a potential hypoglycemic reaction at what time? 1. 5:00 PM 2. 10:00 AM 3. 11:00 AM 4. 11:00 PM

1. 5:00 PM Rationale: NPH is intermediate-acting insulin. Its onset of action is 1 to 2½ hours, it peaks in 4 to 12 hours, and its duration of action is 24 hours. Hypoglycemic reactions most likely occur during peak time.

10.) The clinic nurse is performing an admission assessment on a client. The nurse notes that the client is taking azelaic acid (Azelex). Because of the medication prescription, the nurse would suspect that the client is being treated for: 1. Acne 2. Eczema 3. Hair loss 4. Herpes simplex

1. Acne Rationale: Azelaic acid is a topical medication used to treat mild to moderate acne. The acid appears to work by suppressing the growth of Propionibacterium acnes and decreasing the proliferation of keratinocytes. Options 2, 3, and 4 are incorrect.

145.) A nurse has a prescription to give a client albuterol (Proventil HFA) (two puffs) and beclomethasone dipropionate (Qvar) (nasal inhalation, two puffs), by metered-dose inhaler. The nurse administers the medication by giving the: 1. Albuterol first and then the beclomethasone dipropionate 2. Beclomethasone dipropionate first and then the albuterol 3. Alternating a single puff of each, beginning with the albuterol 4. Alternating a single puff of each, beginning with the beclomethasone dipropionate

1. Albuterol first and then the beclomethasone dipropionate Rationale: Albuterol is a bronchodilator. Beclomethasone dipropionate is a glucocorticoid. Bronchodilators are always administered before glucocorticoids when both are to be given on the same time schedule. This allows for widening of the air passages by the bronchodilator, which then makes the glucocorticoid more effective.

26.) Glimepiride (Amaryl) is prescribed for a client with diabetes mellitus. A nurse reinforces instructions for the client and tells the client to avoid which of the following while taking this medication? 1. Alcohol 2. Organ meats 3. Whole-grain cereals 4. Carbonated beverages

1. Alcohol Rationale: When alcohol is combined with glimepiride (Amaryl), a disulfiram-like reaction may occur. This syndrome includes flushing, palpitations, and nausea. Alcohol can also potentiate the hypoglycemic effects of the medication. Clients need to be instructed to avoid alcohol consumption while taking this medication. The items in options 2, 3, and 4 do not need to be avoided.

100.) Saquinavir (Invirase) is prescribed for the client who is human immunodeficiency virus seropositive. The nurse reinforces medication instructions and tells the client to: 1. Avoid sun exposure. 2. Eat low-calorie foods. 3. Eat foods that are low in fat. 4. Take the medication on an empty stomach.

1. Avoid sun exposure. Rationale: Saquinavir (Invirase) is an antiretroviral (protease inhibitor) used with other antiretroviral medications to manage human immunodeficiency virus infection. Saquinavir is administered with meals and is best absorbed if the client consumes high-calorie, high-fat meals. Saquinavir can cause photosensitivity, and the nurse should instruct the client to avoid sun exposure.

A 15-year-old child is scheduled to receive a series of the hepatitis B vaccine. The child arrives at the clinic for the first dose. The nurse collects data on the child before administering the vaccine and asks the child about a history of an allergy to: 1. Baker's yeast 2. Eggs 3. Penicillin 4. Sulfonamides

1. Baker's yeast Rationale: A contraindication to receiving the hepatitis B vaccine is a previous anaphylactic reaction to common baker's yeast. An allergy to eggs, penicillin, and sulfonamides is unrelated to the contraindication to receiving this vaccine.

105.) A nurse is collecting data from a client and the client's spouse reports that the client is taking donepezil hydrochloride (Aricept). Which disorder would the nurse suspect that this client may have based on the use of this medication? 1. Dementia 2. Schizophrenia 3. Seizure disorder 4. Obsessive-compulsive disorder

1. Dementia Rationale: Donepezil hydrochloride is a cholinergic agent used in the treatment of mild to moderate dementia of the Alzheimer type. It enhances cholinergic functions by increasing the concentration of acetylcholine. It slows the progression of Alzheimer's disease. Options 2, 3, and 4 are incorrect.

33.) The home health care nurse is visiting a client who was recently diagnosed with type 2 diabetes mellitus. The client is prescribed repaglinide (Prandin) and metformin (Glucophage) and asks the nurse to explain these medications. The nurse should reinforce which instructions to the client? Select all that apply. 1. Diarrhea can occur secondary to the metformin. 2. The repaglinide is not taken if a meal is skipped. 3. The repaglinide is taken 30 minutes before eating. 4. Candy or another simple sugar is carried and used to treat mild hypoglycemia episodes. 5. Metformin increases hepatic glucose production to prevent hypoglycemia associated with repaglinide. 6. Muscle pain is an expected side effect of metformin and may be treated with acetaminophen (Tylenol).

1. Diarrhea can occur secondary to the metformin. 2. The repaglinide is not taken if a meal is skipped. 3. The repaglinide is taken 30 minutes before eating. 4. Candy or another simple sugar is carried and used to treat mild hypoglycemia episodes. Rationale: Repaglinide is a rapid-acting oral hypoglycemic agent that stimulates pancreatic insulin secretion that should be taken before meals, and that should be withheld if the client does not eat. Hypoglycemia is a side effect of repaglinide and the client should always be prepared by carrying a simple sugar with her or him at all times. Metformin is an oral hypoglycemic given in combination with repaglinide and works by decreasing hepatic glucose production. A common side effect of metformin is diarrhea. Muscle pain may occur as an adverse effect from metformin but it might signify a more serious condition that warrants health care provider notification, not the use of acetaminophen.

A child is hospitalized with Rocky Mountain spotted fever (RMSF). The health record reveals documentation that the child was bitten by a tick 2 weeks ago. The child presents with complaints of headache, fever, and anorexia, and the nurse notes a rash on the palms of the hands and soles of the feet. The nurse reviews the health care provider's prescriptions and anticipates that which of the following will be prescribed? 1. Doxycycline (Vibramycin) 2. Amphotericin B 3. Ganciclovir (Cytovene) 4. Amantadine (Symmetrel)

1. Doxycycline (Vibramycin) Rationale: The care of a child with RMSF caused by the bacterium Rickettsia rickettsii will include the administration of the antibacterial, doxycycline. Amphotericin B is used for fungal infections. Ganciclovir is used to treat cytomegalovirus. Amantadine is used to treat influenza A virus.

116.) A nurse administers a dose of scopolamine (Transderm-Scop) to a postoperative client. The nurse tells the client to expect which of the following side effects of this medication? 1. Dry mouth 2. Diaphoresis 3. Excessive urination 4. Pupillary constriction

1. Dry mouth Rationale: Scopolamine is an anticholinergic medication for the prevention of nausea and vomiting that causes the frequent side effects of dry mouth, urinary retention, decreased sweating, and dilation of the pupils. The other options describe the opposite effects of cholinergic-blocking agents and therefore are incorrect.

112.) A hospitalized client is started on phenelzine sulfate (Nardil) for the treatment of depression. The nurse instructs the client to avoid consuming which foods while taking this medication? Select all that apply. 1. Figs 2. Yogurt 3. Crackers 4. Aged cheese 5 Tossed salad 6. Oatmeal cookies

1. Figs 2. Yogurt 4. Aged cheese Rationale: Phenelzine sulfate (Nardil) is a monoamine oxidase inhibitor(MAOI). The client should avoid taking in foods that are high in tyramine. Use of these foods could trigger a potentially fatal hypertensive crisis. Some foods to avoid include yogurt, aged cheeses, smoked or processed meats, red wines, and fruits such as avocados, raisins, and figs.

91.) Cyclobenzaprine (Flexeril) is prescribed for a client to treat muscle spasms, and the nurse is reviewing the client's record. Which of the following disorders, if noted in the client's record, would indicate a need to contact the health care provider regarding the administration of this medication? 1. Glaucoma 2. Emphysema 3. Hyperthyroidism 4. Diabetes mellitus

1. Glaucoma Rationale: Because this medication has anticholinergic effects, it should be used with caution in clients with a history of urinary retention, angle-closure glaucoma, and increased intraocular pressure. Cyclobenzaprine hydrochloride should be used only for short-term 2- to 3-week therapy.

125.) A nurse is preparing to administer digoxin (Lanoxin), 0.125 mg orally, to a client with heart failure. Which vital sign is most important for the nurse to check before administering the medication? 1. Heart rate 2. Temperature 3. Respirations 4. Blood pressure

1. Heart rate Rationale: Digoxin is a cardiac glycoside that is used to treat heart failure and acts by increasing the force of myocardial contraction. Because bradycardia may be a clinical sign of toxicity, the nurse counts the apical heart rate for 1 full minute before administering the medication. If the pulse rate is less than 60 beats/minute in an adult client, the nurse would withhold the medication and report the pulse rate to the registered nurse, who would then contact the health care provider.

158.) A client with chronic renal failure is receiving epoetin alfa (Epogen, Procrit). Which laboratory result would indicate a therapeutic effect of the medication? 1. Hematocrit of 32% 2. Platelet count of 400,000 cells/mm3 3. White blood cell count of 6000 cells/mm3 4. Blood urea nitrogen (BUN) level of 15 mg/dL

1. Hematocrit of 32% Rationale: Epoetin alfa is used to reverse anemia associated with chronic renal failure. A therapeutic effect is seen when the hematocrit is between 30% and 33%. The laboratory tests noted in the other options are unrelated to the use of this medication.

167.) A nurse prepares to reinforce instructions to a client who is taking allopurinol (Zyloprim). The nurse plans to include which of the following in the instructions? 1. Instruct the client to drink 3000 mL of fluid per day. 2. Instruct the client to take the medication on an empty stomach. 3. Inform the client that the effect of the medication will occur immediately. 4. Instruct the client that, if swelling of the lips occurs, this is a normal expected response.

1. Instruct the client to drink 3000 mL of fluid per day. Rationale: Allopurinol (Zyloprim) is an antigout medication used to decrease uric acid levels. Clients taking allopurinol are encouraged to drink 3000 mL of fluid a day. A full therapeutic effect may take 1 week or longer. Allopurinol is to be given with or immediately following meals or milk to prevent gastrointestinal irritation. If the client develops a rash, irritation of the eyes, or swelling of the lips or mouth, he or she should contact the health care provider because this may indicate hypersensitivity.

113.) A nurse is reinforcing discharge instructions to a client receiving sulfisoxazole. Which of the following would be included in the plan of care for instructions? 1. Maintain a high fluid intake. 2. Discontinue the medication when feeling better. 3. If the urine turns dark brown, call the health care provider immediately. 4. Decrease the dosage when symptoms are improving to prevent an allergic response.

1. Maintain a high fluid intake. Rationale: Each dose of sulfisoxazole should be administered with a full glass of water, and the client should maintain a high fluid intake. The medication is more soluble in alkaline urine. The client should not be instructed to taper or discontinue the dose. Some forms of sulfisoxazole cause the urine to turn dark brown or red. This does not indicate the need to notify the health care provider.

12.) A nurse is caring for a client who is receiving an intravenous (IV) infusion of an antineoplastic medication. During the infusion, the client complains of pain at the insertion site. During an inspection of the site, the nurse notes redness and swelling and that the rate of infusion of the medication has slowed. The nurse should take which appropriate action? 1. Notify the registered nurse. 2. Administer pain medication to reduce the discomfort. 3. Apply ice and maintain the infusion rate, as prescribed. 4. Elevate the extremity of the IV site, and slow the infusion.

1. Notify the registered nurse. Rationale: When antineoplastic medications (Chemotheraputic Agents) are administered via IV, great care must be taken to prevent the medication from escaping into the tissues surrounding the injection site, because pain, tissue damage, and necrosis can result. The nurse monitors for signs of extravasation, such as redness or swelling at the insertion site and a decreased infusion rate. If extravasation occurs, the registered nurse needs to be notified; he or she will then contact the health care provider.

18.) The nurse is reviewing the history and physical examination of a client who will be receiving asparaginase (Elspar), an antineoplastic agent. The nurse consults with the registered nurse regarding the administration of the medication if which of the following is documented in the client's history? 1. Pancreatitis 2. Diabetes mellitus 3. Myocardial infarction 4. Chronic obstructive pulmonary disease

1. Pancreatitis Rationale: Asparaginase (Elspar) is contraindicated if hypersensitivity exists, in pancreatitis, or if the client has a history of pancreatitis. The medication impairs pancreatic function and pancreatic function tests should be performed before therapy begins and when a week or more has elapsed between administration of the doses. The client needs to be monitored for signs of pancreatitis, which include nausea, vomiting, and abdominal pain. The conditions noted in options 2, 3, and 4 are not contraindicated with this medication.

30.) A client with diabetes mellitus visits a health care clinic. The client's diabetes mellitus previously had been well controlled with glyburide (DiaBeta) daily, but recently the fasting blood glucose level has been 180 to 200 mg/dL. Which medication, if added to the client's regimen, may have contributed to the hyperglycemia? 1. Prednisone 2. Phenelzine (Nardil) 3. Atenolol (Tenormin) 4. Allopurinol (Zyloprim)

1. Prednisone Rationale: Prednisone may decrease the effect of oral hypoglycemics, insulin, diuretics, and potassium supplements. Option 2, a monoamine oxidase inhibitor, and option 3, a β-blocker, have their own intrinsic hypoglycemic activity. Option 4 decreases urinary excretion of sulfonylurea agents, causing increased levels of the oral agents, which can lead to hypoglycemia.

152.) Intravenous heparin therapy is prescribed for a client. While implementing this prescription, a nurse ensures that which of the following medications is available on the nursing unit? 1. Protamine sulfate 2. Potassium chloride 3. Phytonadione (vitamin K ) 4. Aminocaproic acid (Amicar)

1. Protamine sulfate Rationale: The antidote to heparin is protamine sulfate; it should be readily available for use if excessive bleeding or hemorrhage occurs. Potassium chloride is administered for a potassium deficit. Vitamin K is an antidote for warfarin sodium. Aminocaproic acid is the antidote for thrombolytic therapy.

72.) Cinoxacin (Cinobac), a urinary antiseptic, is prescribed for the client. The nurse reviews the client's medical record and should contact the health care provider (HCP) regarding which documented finding to verify the prescription? Refer to chart. 1. Renal insufficiency 2. Chest x-ray: normal 3. Blood glucose, 102 mg/dL 4. Folic acid (vitamin B6) 0.5 mg, orally daily

1. Renal insufficiency Rationale: Cinoxacin should be administered with caution in clients with renal impairment. The dosage should be reduced, and failure to do so could result in accumulation of cinoxacin to toxic levels. Therefore the nurse would verify the prescription if the client had a documented history of renal insufficiency. The laboratory and diagnostic test results are normal findings. Folic acid (vitamin B6) may be prescribed for a client with renal insufficiency to prevent anemia.

53.) Rifabutin (Mycobutin) is prescribed for a client with active Mycobacterium avium complex (MAC) disease and tuberculosis. The nurse monitors for which side effects of the medication? Select all that apply. 1. Signs of hepatitis 2. Flu-like syndrome 3. Low neutrophil count 4. Vitamin B6 deficiency 5. Ocular pain or blurred vision 6. Tingling and numbness of the fingers

1. Signs of hepatitis 2. Flu-like syndrome 3. Low neutrophil count 5. Ocular pain or blurred vision Rationale: Rifabutin (Mycobutin) may be prescribed for a client with active MAC disease and tuberculosis. It inhibits mycobacterial DNA-dependent RNA polymerase and suppresses protein synthesis. Side effects include rash, gastrointestinal disturbances, neutropenia (low neutrophil count), red-orange body secretions, uveitis (blurred vision and eye pain), myositis, arthralgia, hepatitis, chest pain with dyspnea, and flu-like syndrome. Vitamin B6 deficiency and numbness and tingling in the extremities are associated with the use of isoniazid (INH). Ethambutol (Myambutol) also causes peripheral neuritis.

92.) In monitoring a client's response to disease-modifying antirheumatic drugs (DMARDs), which findings would the nurse interpret as acceptable responses? Select all that apply. 1. Symptom control during periods of emotional stress 2. Normal white blood cell counts, platelet, and neutrophil counts 3. Radiological findings that show nonprogression of joint degeneration 4. An increased range of motion in the affected joints 3 months into therapy 5. Inflammation and irritation at the injection site 3 days after injection is given 6. A low-grade temperature upon rising in the morning that remains throughout the day

1. Symptom control during periods of emotional stress 2. Normal white blood cell counts, platelet, and neutrophil counts 3. Radiological findings that show nonprogression of joint degeneration 4. An increased range of motion in the affected joints 3 months into therapy Rationale: Because emotional stress frequently exacerbates the symptoms of rheumatoid arthritis, the absence of symptoms is a positive finding. DMARDs are given to slow progression of joint degeneration. In addition, the improvement in the range of motion after 3 months of therapy with normal blood work is a positive finding. Temperature elevation and inflammation and irritation at the medication injection site could indicate signs of infection.

678. A nurse teaches a client about the effects of diphenhydramine (Benadryl), which has been prescribed as a cough suppressant. The nurse determines that the client needs further instructions if the client states that he or she will: 1. Take the medication on an empty stomach. 2. Avoid using alcohol while taking this medication. 3. Use sugarless gum, candy, or oral rinses to decrease dry mouth. 4. Avoid activities requiring mental alertness while taking this medication.

1. Take the medication on an empty stomach.

149.) A client taking fexofenadine (Allegra) is scheduled for allergy skin testing and tells the nurse in the health care provider's office that a dose was taken this morning. The nurse determines that: 1. The client should reschedule the appointment. 2. A lower dose of allergen will need to be injected. 3. A higher dose of allergen will need to be injected. 4. The client should have the skin test read a day later than usual.

1. The client should reschedule the appointment. Rationale: Fexofenadine is an antihistamine, which provides relief of symptoms caused by allergy. Antihistamines should be discontinued for at least 3 days (72 hours) before allergy skin testing to avoid false-negative readings. This client should have the appointment rescheduled for 3 days after discontinuing the medication.

213.) A client is admitted to the hospital with complaints of back spasms. The client states, "I have been taking two or three aspirin every 4 hours for the past week and it hasn't helped my back." Aspirin intoxication is suspected. Which of the following complaints would indicate aspirin intoxication? 1. Tinnitus 2. Constipation 3. Photosensitivity 4. Abdominal cramps

1. Tinnitus Rationale: Mild intoxication with acetylsalicylic acid (aspirin) is called salicylism and is commonly experienced when the daily dosage is higher than 4 g. Tinnitus (ringing in the ears) is the most frequently occurring effect noted with intoxication. Hyperventilation may occur because salicylate stimulates the respiratory center. Fever may result because salicylate interferes with the metabolic pathways involved with oxygen consumption and heat production. Options 2, 3, and 4 are incorrect.

A nurse notes that a client in a long-term care facility is receiving a daily dose of furosemide (Lasix). The nurse writes in the care plan to monitor which of the following parameters on a daily basis? 1. Weight 2. Radial pulse 3. Hemoglobin 4. Serum creatinine clearance

1. Weight Rationale: Daily weight should be monitored because this reflects the fluid status of the client who is receiving a diuretic. Option 2 is a general assessment and does not relate directly to fluid balance. Options 3 and 4 are laboratory measurements that are not prescribed routinely by the nurse and would not be done on a daily basis in a long-term care facility.

176.) A nurse notes that a client is taking lansoprazole (Prevacid). On data collection, the nurse asks which question to determine medication effectiveness? 1. "Has your appetite increased?" 2. "Are you experiencing any heartburn?" 3. "Do you have any problems with vision?" 4. "Do you experience any leg pain when walking?"

2. "Are you experiencing any heartburn?" Rationale: Lansoprazole is a gastric acid pump inhibitor used to treat gastric and duodenal ulcers, erosive esophagitis, and hypersecretory conditions. It also is used to treat gastroesophageal reflux disease (GERD). It is not used to treat visual problems, problems with appetite, or leg pain. **NOTE: "-zole" refers to gastric acid pump inhibitors**

183.) A client who received a kidney transplant is taking azathioprine (Imuran), and the nurse provides instructions about the medication. Which statement by the client indicates a need for further instructions? 1. "I need to watch for signs of infection." 2. "I need to discontinue the medication after 14 days of use." 3. "I can take the medication with meals to minimize nausea." 4. "I need to call the health care provider (HCP) if more than one dose is missed."

2. "I need to discontinue the medication after 14 days of use." Rationale: Azathioprine is an immunosuppressant medication that is taken for life. Because of the effects of the medication, the client must watch for signs of infection, which are reported immediately to the HCP. The client should also call the HCP if more than one dose is missed. The medication may be taken with meals to minimize nausea.

194.) A nurse provides instructions to a client regarding the use of tretinoin (Retin-A). Which statement by the client indicates the need for further instructions? 1. "Optimal results will be seen after 6 weeks." 2. "I should apply a very thin layer to my skin." 3. "I should wash my hands thoroughly after applying the medication." 4. "I should cleanse my skin thoroughly before applying the medication."

2. "I should apply a very thin layer to my skin." Rationale: Tretinoin is applied liberally to the skin. The hands are washed thoroughly immediately after applying. Therapeutic results should be seen after 2 to 3 weeks but may not be optimal until after 6 weeks. The skin needs to be cleansed thoroughly before applying the medication.

A client receiving therapy with carbidopa/levodopa (Sinemet) is upset and tells the nurse that his urine has turned a darker color since he began to take the medication. The client wants to discontinue its use. In formulating a response to the client's concerns, the nurse interprets that this change is: 1. Indicative of developing toxicity 2. A harmless side effect of the medication 3. A result of taking the medication with milk 4. A sign of interaction with another medication

2. A harmless side effect of the medication Rationale: With carbidopa/levodopa therapy, a darkening of the urine or sweat may occur. The client should be reassured that this is a harmless effect of the medication, and its use should be continued. **Eliminate options 1 and 4 first because they are comparable or alike.**

Chapter 61- cardio 717. A client with atrial fibrillation is receiving a continuous heparin infusion at 1000 units/hr. The nurse would determine that the client is receiving the therapeutic effect based on which of the following results? 1. Prothrombin time of 12.5 seconds 2. Activated partial thromboplastin time of 60 seconds 3. Activated partial thromboplastin time of 28 seconds 4. Activated partial thromboplastin time longer than 120 seconds

2. Activated partial thromboplastin time of 60 seconds

chapter 77- psych 944. A client's medication sheet contains a prescription for sertraline (Zoloft). To ensure safe administration of the medication, a nurse would administer the dose: 1. On an empty stomach 2. At the same time each evening 3. Evenly spaced around the clock 4. As needed when the client complains of depression

2. At the same time each evening

74.) A client with myasthenia gravis is receiving pyridostigmine (Mestinon). The nurse monitors for signs and symptoms of cholinergic crisis caused by overdose of the medication. The nurse checks the medication supply to ensure that which medication is available for administration if a cholinergic crisis occurs? 1. Vitamin K 2. Atropine sulfate 3. Protamine sulfate 4. Acetylcysteine (Mucomyst)

2. Atropine sulfate Rationale: The antidote for cholinergic crisis is atropine sulfate. Vitamin K is the antidote for warfarin (Coumadin). Protamine sulfate is the antidote for heparin, and acetylcysteine (Mucomyst) is the antidote for acetaminophen (Tylenol).

86.) A nurse is reinforcing discharge instructions to a client receiving baclofen (Lioresal). Which of the following would the nurse include in the instructions? 1. Restrict fluid intake. 2. Avoid the use of alcohol. 3. Stop the medication if diarrhea occurs. 4. Notify the health care provider if fatigue occurs.

2. Avoid the use of alcohol. Rationale: Baclofen is a central nervous system (CNS) depressant. The client should be cautioned against the use of alcohol and other CNS depressants, because baclofen potentiates the depressant activity of these agents. Constipation rather than diarrhea is an adverse effect of baclofen. It is not necessary to restrict fluids, but the client should be warned that urinary retention can occur. Fatigue is related to a CNS effect that is most intense during the early phase of therapy and diminishes with continued medication use. It is not necessary that the client notify the health care provider if fatigue occurs.

199.) A nurse is applying a topical glucocorticoid to a client with eczema. The nurse monitors for systemic absorption of the medication if the medication is being applied to which of the following body areas? 1. Back 2. Axilla 3. Soles of the feet 4. Palms of the hands

2. Axilla Rationale: Topical glucocorticoids can be absorbed into the systemic circulation. Absorption is higher from regions where the skin is especially permeable (scalp, axillae, face, eyelids, neck, perineum, genitalia), and lower from regions where penetrability is poor (back, palms, soles). **Eliminate options 3 and 4 because these body areas are similar in terms of skin characteristics**

123.) A nurse is planning to administer amlodipine (Norvasc) to a client. The nurse plans to check which of the following before giving the medication? 1. Respiratory rate 2. Blood pressure and heart rate 3. Heart rate and respiratory rate 4. Level of consciousness and blood pressure

2. Blood pressure and heart rate Rationale: Amlodipine is a calcium channel blocker. This medication decreases the rate and force of cardiac contraction. Before administering a calcium channel blocking agent, the nurse should check the blood pressure and heart rate, which could both decrease in response to the action of this medication. This action will help to prevent or identify early problems related to decreased cardiac contractility, heart rate, and conduction. **amlodipine is a calcium channel blocker, and this group of medications decreases the rate and force of cardiac contraction. This in turn lowers the pulse rate and blood pressure.**

170.) Atenolol hydrochloride (Tenormin) is prescribed for a hospitalized client. The nurse should perform which of the following as a priority action before administering the medication? 1. Listen to the client's lung sounds. 2. Check the client's blood pressure. 3. Check the recent electrolyte levels. 4. Assess the client for muscle weakness.

2. Check the client's blood pressure. Rationale: Atenolol hydrochloride is a beta-blocker used to treat hypertension. Therefore the priority nursing action before administration of the medication is to check the client's blood pressure. The nurse also checks the client's apical heart rate. If the systolic blood pressure is below 90 mm Hg or the apical pulse is 60 beats per minute or lower, the medication is withheld and the registered nurse and/or health care provider is notified. The nurse would check baseline renal and liver function tests. The medication may cause weakness, and the nurse would assist the client with activities if weakness occurs. **Beta-blockers have "-lol" at the end of the medication name**

950. A client receiving tricyclic antidepressants arrives at the mental health clinic. Which observation would indicate that the client is following the medication plan correctly? 1. Client reports not going to work for this past week. 2. Client arrives at the clinic neat and appropriate in appearance. 3. Client complains of not being able to "do anything" anymore. 4. Client reports sleeping 12 hours per night and 3 to 4 hours during the day.

2. Client arrives at the clinic neat and appropriate in appearance.

146.) A client has begun therapy with theophylline (Theo-24). The nurse tells the client to limit the intake of which of the following while taking this medication? 1. Oranges and pineapple 2. Coffee, cola, and chocolate 3. Oysters, lobster, and shrimp 4. Cottage cheese, cream cheese, and dairy creamers

2. Coffee, cola, and chocolate Rationale: Theophylline is a xanthine bronchodilator. The nurse teaches the client to limit the intake of xanthine-containing foods while taking this medication. These include coffee, cola, and chocolate.

63.) A client with coronary artery disease complains of substernal chest pain. After checking the client's heart rate and blood pressure, a nurse administers nitroglycerin, 0.4 mg, sublingually. After 5 minutes, the client states, "My chest still hurts." Select the appropriate actions that the nurse should take. Select all that apply. 1. Call a code blue. 2. Contact the registered nurse. 3. Contact the client's family. 4. Assess the client's pain level. 5. Check the client's blood pressure. 6. Administer a second nitroglycerin, 0.4 mg, sublingually.

2. Contact the registered nurse. 4. Assess the client's pain level. 5. Check the client's blood pressure. 6. Administer a second nitroglycerin, 0.4 mg, sublingually. Rationale: The usual guideline for administering nitroglycerin tablets for a hospitalized client with chest pain is to administer one tablet every 5 minutes PRN for chest pain, for a total dose of three tablets. The registered nurse should be notified of the client's condition, who will then notify the health care provider as appropriate. Because the client is still complaining of chest pain, the nurse would administer a second nitroglycerin tablet. The nurse would assess the client's pain level and check the client's blood pressure before administering each nitroglycerin dose. There are no data in the question that indicate the need to call a code blue. In addition, it is not necessary to contact the client's family unless the client has requested this.

A client with a history of spinal cord injury is receiving baclofen (Lioresal) for muscle spasms. The nurse determines that the client is experiencing a side effect of this medication if the client experiences: 1. Photosensitivity 2. Drowsiness 3. Hypertension 4. Muscle pain

2. Drowsiness Rationale: Baclofen is a centrally acting skeletal muscle relaxant. Side effects of baclofen include drowsiness, dizziness, weakness, and nausea. Occasional side effects include headache, paresthesias of the hands and feet, constipation or diarrhea, anorexia, hypotension, confusion, and nasal congestion. Options 1, 3, and 4 are incorrect.

133.) A nurse is monitoring a client receiving desmopressin acetate (DDAVP) for adverse effects to the medication. Which of the following indicates the presence of an adverse effect? 1. Insomnia 2. Drowsiness 3. Weight loss 4. Increased urination

2. Drowsiness Rationale: Water intoxication (overhydration) or hyponatremia is an adverse effect to desmopressin. Early signs include drowsiness, listlessness, and headache. Decreased urination, rapid weight gain, confusion, seizures, and coma also may occur in overhydration. **Recall that this medication is used to treat diabetes insipidus to eliminate weight loss and increased urination.**

131.) The nurse is reinforcing medication instructions to a client with breast cancer who is receiving cyclophosphamide (Neosar). The nurse tells the client to: 1. Take the medication with food. 2. Increase fluid intake to 2000 to 3000 mL daily. 3. Decrease sodium intake while taking the medication. 4. Increase potassium intake while taking the medication.

2. Increase fluid intake to 2000 to 3000 mL daily. Rationale: Hemorrhagic cystitis is a toxic effect that can occur with the use of cyclophosphamide. The client needs to be instructed to drink copious amounts of fluid during the administration of this medication. Clients also should monitor urine output for hematuria. The medication should be taken on an empty stomach, unless gastrointestinal (GI) upset occurs. Hyperkalemia can result from the use of the medication; therefore the client would not be told to increase potassium intake. The client would not be instructed to alter sodium intake.

101.) Ketoconazole is prescribed for a client with a diagnosis of candidiasis. Select the interventions that the nurse includes when administering this medication. Select all that apply. 1. Restrict fluid intake. 2. Instruct the client to avoid alcohol. 3. Monitor hepatic and liver function studies. 4. Administer the medication with an antacid. 5. Instruct the client to avoid exposure to the sun. 6. Administer the medication on an empty stomach.

2. Instruct the client to avoid alcohol. 3. Monitor hepatic and liver function studies. 5. Instruct the client to avoid exposure to the sun. Rationale: Ketoconazole is an antifungal medication. It is administered with food (not on an empty stomach) and antacids are avoided for 2 hours after taking the medication to ensure absorption. The medication is hepatotoxic and the nurse monitors liver function studies. The client is instructed to avoid exposure to the sun because the medication increases photosensitivity. The client is also instructed to avoid alcohol. There is no reason for the client to restrict fluid intake. In fact, this could be harmful to the client.

162.) Carbamazepine (Tegretol) is prescribed for a client with a diagnosis of psychomotor seizures. The nurse reviews the client's health history, knowing that this medication is contraindicated if which of the following disorders is present? 1. Headaches 2. Liver disease 3. Hypothyroidism 4. Diabetes mellitus

2. Liver disease Rationale: Carbamazepine (Tegretol) is contraindicated in liver disease, and liver function tests are routinely prescribed for baseline purposes and are monitored during therapy. It is also contraindicated if the client has a history of blood dyscrasias. It is not contraindicated in the conditions noted in the incorrect options.

89.) A nurse is reviewing the laboratory studies on a client receiving dantrolene sodium (Dantrium). Which laboratory test would identify an adverse effect associated with the administration of this medication? 1. Creatinine 2. Liver function tests 3. Blood urea nitrogen 4. Hematological function tests

2. Liver function tests Rationale: Dose-related liver damage is the most serious adverse effect of dantrolene. To reduce the risk of liver damage, liver function tests should be performed before treatment and periodically throughout the treatment course. It is administered in the lowest effective dosage for the shortest time necessary. **Eliminate options 1 and 3 because these tests both assess kidney function.**

161.) A nurse is caring for a client with severe back pain, and codeine sulfate has been prescribed for the client. Which of the following would the nurse include in the plan of care while the client is taking this medication? 1. Restrict fluid intake. 2. Monitor bowel activity. 3. Monitor for hypertension. 4. Monitor peripheral pulses.

2. Monitor bowel activity. Rationale: While the client is taking codeine sulfate, an opioid analgesic, the nurse would monitor vital signs and monitor for hypotension. The nurse should also increase fluid intake, palpate the bladder for urinary retention, auscultate bowel sounds, and monitor the pattern of daily bowel activity and stool consistency (codeine can cause constipation). The nurse should monitor respiratory status and initiate breathing and coughing exercises. In addition, the nurse monitors the effectiveness of the pain medication.

204.) A client receives a dose of edrophonium (Enlon). The client shows improvement in muscle strength for a period of time following the injection. The nurse interprets that this finding is compatible with: 1. Multiple sclerosis 2. Myasthenia gravis 3. Muscular dystrophy 4. Amyotrophic lateral sclerosis

2. Myasthenia gravis Rationale: Myasthenia gravis can often be diagnosed based on clinical signs and symptoms. The diagnosis can be confirmed by injecting the client with a dose of edrophonium . This medication inhibits the breakdown of an enzyme in the neuromuscular junction, so more acetylcholine binds to receptors. If the muscle is strengthened for 3 to 5 minutes after this injection, it confirms a diagnosis of myasthenia gravis. Another medication, neostigmine (Prostigmin), also may be used because its effect lasts for 1 to 2 hours, providing a better analysis. For either medication, atropine sulfate should be available as the antidote.

682. A client has been taking isoniazid (INH) for 1½ months. The client complains to a nurse about numbness, paresthesias, and tingling in the extremities. The nurse interprets that the client is experiencing: 1. Hypercalcemia 2. Peripheral neuritis 3. Small blood vessel spasm 4. Impaired peripheral circulation

2. Peripheral neuritis

953. A client who has been taking buspirone (BuSpar) for 1 month returns to the clinic for a follow-up assessment. A nurse determines that the medication is effective if the absence of which manifestation has occurred? 1. Paranoid thought process 2. Rapid heartbeat or anxiety 3. Alcohol withdrawal symptoms 4. Thought broadcasting or delusions

2. Rapid heartbeat or anxiety

40.) The client who chronically uses nonsteroidal anti-inflammatory drugs has been taking misoprostol (Cytotec). The nurse determines that the medication is having the intended therapeutic effect if which of the following is noted? 1. Resolved diarrhea 2. Relief of epigastric pain 3. Decreased platelet count 4. Decreased white blood cell count

2. Relief of epigastric pain Rationale: The client who chronically uses nonsteroidal anti-inflammatory drugs (NSAIDs) is prone to gastric mucosal injury. Misoprostol is a gastric protectant and is given specifically to prevent this occurrence. Diarrhea can be a side effect of the medication, but is not an intended effect. Options 3 and 4 are incorrect.

48.) A client is to begin a 6-month course of therapy with isoniazid (INH). A nurse plans to teach the client to: 1. Drink alcohol in small amounts only. 2. Report yellow eyes or skin immediately. 3. Increase intake of Swiss or aged cheeses. 4. Avoid vitamin supplements during therapy.

2. Report yellow eyes or skin immediately. Rationale: INH is hepatotoxic, and therefore the client is taught to report signs and symptoms of hepatitis immediately (which include yellow skin and sclera). For the same reason, alcohol should be avoided during therapy. The client should avoid intake of Swiss cheese, fish such as tuna, and foods containing tyramine because they may cause a reaction characterized by redness and itching of the skin, flushing, sweating, tachycardia, headache, or lightheadedness. The client can avoid developing peripheral neuritis by increasing the intake of pyridoxine (vitamin B6) during the course of INH therapy for TB.

188.) The nurse should anticipate that the most likely medication to be prescribed prophylactically for a child with spina bifida (myelomeningocele) who has a neurogenic bladder would be: 1. Prednisone 2. Sulfisoxazole 3. Furosemide (Lasix) 4. Intravenous immune globulin (IVIG)

2. Sulfisoxazole Rationale: A neurogenic bladder prevents the bladder from completely emptying because of the decrease in muscle tone. The most likely medication to be prescribed to prevent urinary tract infection would be an antibiotic. A common prescribed medication is sulfisoxazole. Prednisone relieves allergic reactions and inflammation rather than preventing infection. Furosemide promotes diuresis and decreases edema caused by congestive heart failure. IVIG assists with antibody production in immunocompromised clients.

147.) A client with a prescription to take theophylline (Theo-24) daily has been given medication instructions by the nurse. The nurse determines that the client needs further information about the medication if the client states that he or she will: 1. Drink at least 2 L of fluid per day. 2. Take the daily dose at bedtime. 3. Avoid changing brands of the medication without health care provider (HCP) approval. 4. Avoid over-the-counter (OTC) cough and cold medications unless approved by the HCP.

2. Take the daily dose at bedtime. Rationale: The client taking a single daily dose of theophylline, a xanthine bronchodilator, should take the medication early in the morning. This enables the client to have maximal benefit from the medication during daytime activities. In addition, this medication causes insomnia. The client should take in at least 2 L of fluid per day to decrease viscosity of secretions. The client should check with the physician before changing brands of the medication. The client also checks with the HCP before taking OTC cough, cold, or other respiratory preparations because they could cause interactive effects, increasing the side effects of theophylline and causing dysrhythmias.

214.) A health care provider initiates carbidopa/levodopa (Sinemet) therapy for the client with Parkinson's disease. A few days after the client starts the medication, the client complains of nausea and vomiting. The nurse tells the client that: 1. Taking an antiemetic is the best measure to prevent the nausea. 2. Taking the medication with food will help to prevent the nausea. 3. This is an expected side effect of the medication and will decrease over time. 4. The nausea and vomiting will decrease when the dose of levodopa is stabilized.

2. Taking the medication with food will help to prevent the nausea. Rationale: If carbidopa/levodopa is causing nausea and vomiting, the nurse would tell the client that taking the medication with food will prevent the nausea. Additionally, the client should be instructed not to take the medication with a high-protein meal because the high-protein will affect absorption. Antiemetics from the phenothiazine class should not be used because they block the therapeutic action of dopamine. **eliminate options 3 and 4 because they are comparable or alike**

28.) The health care provider (HCP) prescribes exenatide (Byetta) for a client with type 1 diabetes mellitus who takes insulin. The nurse knows that which of the following is the appropriate intervention? 1. The medication is administered within 60 minutes before the morning and evening meal. 2. The medication is withheld and the HCP is called to question the prescription for the client. 3. The client is monitored for gastrointestinal side effects after administration of the medication. 4. The insulin is withdrawn from the Penlet into an insulin syringe to prepare for administration.

2. The medication is withheld and the HCP is called to question the prescription for the client. Rationale: Exenatide (Byetta) is an incretin mimetic used for type 2 diabetes mellitus only. It is not recommended for clients taking insulin. Hence, the nurse should hold the medication and question the HCP regarding this prescription. Although options 1 and 3 are correct statements about the medication, in this situation the medication should not be administered. The medication is packaged in prefilled pens ready for injection without the need for drawing it up into another syringe.

165.) The client has been on treatment for rheumatoid arthritis for 3 weeks. During the administration of etanercept (Enbrel), it is most important for the nurse to assess: 1. The injection site for itching and edema 2. The white blood cell counts and platelet counts 3. Whether the client is experiencing fatigue and joint pain 4. A metallic taste in the mouth and a loss of appetite

2. The white blood cell counts and platelet counts Rationale: Infection and pancytopenia are adverse effects of etanercept (Enbrel). Laboratory studies are performed before and during treatment. The appearance of abnormal white blood cell counts and abnormal platelet counts can alert the nurse to a potential life-threatening infection. Injection site itching is a common occurrence following administration of the medication. In early treatment, residual fatigue and joint pain may still be apparent. A metallic taste and loss of appetite are not common signs of side effects of this medication.

83.) The client has been on treatment for rheumatoid arthritis for 3 weeks. During the administration of etanercept (Enbrel), it is most important for the nurse to check: 1. The injection site for itching and edema 2. The white blood cell counts and platelet counts 3. Whether the client is experiencing fatigue and joint pain 4. A metallic taste in the mouth, with a loss of appetite

2. The white blood cell counts and platelet counts Rationale: Infection and pancytopenia are side effects of etanercept (Enbrel). Laboratory studies are performed before and during drug treatment. The appearance of abnormal white blood cell counts and abnormal platelet counts can alert the nurse to a potentially life-threatening infection. Injection site itching is a common occurrence following administration. A metallic taste with loss of appetite are not common signs of side effects of this medication.

7.) Isotretinoin is prescribed for a client with severe acne. Before the administration of this medication, the nurse anticipates that which laboratory test will be prescribed? 1. Platelet count 2. Triglyceride level 3. Complete blood count 4. White blood cell count

2. Triglyceride level Rationale: Isotretinoin can elevate triglyceride levels. Blood triglyceride levels should be measured before treatment and periodically thereafter until the effect on the triglycerides has been evaluated. Options 1, 3, and 4 do not need to be monitored specifically during this treatment.

68.) Bethanechol chloride (Urecholine) is prescribed for a client with urinary retention. Which disorder would be a contraindication to the administration of this medication? 1. Gastric atony 2. Urinary strictures 3. Neurogenic atony 4. Gastroesophageal reflux

2. Urinary strictures Rationale: Bethanechol chloride (Urecholine) can be harmful to clients with urinary tract obstruction or weakness of the bladder wall. The medication has the ability to contract the bladder and thereby increase pressure within the urinary tract. Elevation of pressure within the urinary tract could rupture the bladder in clients with these conditions.

A nurse has given medication instructions to a client beginning anticonvulsant therapy with carbamazepine (Tegretol). The nurse determines that the client understands the use of the medication if the client knows to: 1. Drive as long as it is not at night. 2. Use sunscreen when outsides. 3. Discontinue the medication if fever or sore throat occurs. 4. Keep tissues handy because of excess salivation that may occur.

2. Use sunscreen when outsides. Rationale: Carbamazepine acts by depressing synaptic transmission in the central nervous system (CNS). Because of this, the client should avoid driving or doing other activities that require mental alertness until the effect on the client is known. The client should use protective clothing and sunscreen to avoid photosensitivity reactions. The medication may cause dry mouth (not excessive salivation), and the client should be instructed to provide good oral hygiene and use sugarless candy or gum as needed. The medication should not be abruptly discontinued because it could cause return of seizures or status epilepticus. Fever and sore throat (leukopenia) should be reported to the health care provider (HCP).

55.) A client who is receiving digoxin (Lanoxin) daily has a serum potassium level of 3.0 mEq/L and is complaining of anorexia. A health care provider prescribes a digoxin level to rule out digoxin toxicity. A nurse checks the results, knowing that which of the following is the therapeutic serum level (range) for digoxin? 1. 3 to 5 ng/mL 2. 0.5 to 2 ng/mL 3. 1.2 to 2.8 ng/mL 4. 3.5 to 5.5 ng/mL

2.) 0.5 to 2 ng/mL Rationale: Therapeutic levels for digoxin range from 0.5 to 2 ng/mL. Therefore, options 1, 3, and 4 are incorrect.

11.) The health care provider has prescribed silver sulfadiazine (Silvadene) for the client with a partial-thickness burn, which has cultured positive for gram-negative bacteria. The nurse is reinforcing information to the client about the medication. Which statement made by the client indicates a lack of understanding about the treatments? 1. "The medication is an antibacterial." 2. "The medication will help heal the burn." 3. "The medication will permanently stain my skin." 4. "The medication should be applied directly to the wound."

3. "The medication will permanently stain my skin." Rationale: Silver sulfadiazine (Silvadene) is an antibacterial that has a broad spectrum of activity against gram-negative bacteria, gram-positive bacteria, and yeast. It is applied directly to the wound to assist in healing. It does not stain the skin.

A client is receiving a maintenance dose of oral dantrolene sodium (Dantrium) for the treatment of spasticity. The nurse reviews the medication record, expecting that which of the following doses would be prescribed? 1. 50 mg daily 2. 100 mg daily 3. 100 mg twice daily 4. 200 mg four times daily

3. 100 mg twice daily Rationale: For treatment of spasticity, dantrolene is administered orally. The initial dosage in adults is 25 mg once daily. The usual maintenance dosage is 100 mg two to four times daily. If beneficial effects do not develop within 45 days, dantrolene therapy should be discontinued.

219.) A health care provider instructs a client with rheumatoid arthritis to take ibuprofen (Motrin). The nurse reinforces the instructions, knowing that the normal adult dose for this client is which of the following? 1. 100 mg orally twice a day 2. 200 mg orally twice a day 3. 400 mg orally three times a day 4. 1000 mg orally four times a day

3. 400 mg orally three times a day Rationale: For acute or chronic rheumatoid arthritis or osteoarthritis, the normal oral adult dose is 400 to 800 mg three or four times daily.

A nurse is caring for a child receiving carbamazepine (Tegretol) who has a carbamazepine level drawn. Which of the following results indicates a therapeutic level? 1. 1 mcg/mL 2. 3 mcg/mL 3. 6 mcg/mL 4. 15 mcg/mL

3. 6 mcg/mL Rationale: When carbamazepine is administered, blood levels need to be drawn periodically to check for the child's absorption of the medication. The amount of the medication prescribed is based on the blood level achieved. The therapeutic serum level for this medication is 4 to 12 mcg/mL.

191.) A child is brought to the emergency department for treatment of an acute asthma attack. The nurse prepares to administer which of the following medications first? 1. Oral corticosteroids 2. A leukotriene modifier 3. A β2 agonist 4. A nonsteroidal anti-inflammatory

3. A β2 agonist Rationale: In treating an acute asthma attack, a short acting β2 agonist such as albuterol (Proventil HFA) will be given to produce bronchodilation. Options 1, 2, and 4 are long-term control (preventive) medications.

207.) A client is suspected of having myasthenia gravis, and the health care provider administers edrophonium (Enlon) to determine the diagnosis. After administration of this medication, which of the following would indicate the presence of myasthenia gravis? 1. Joint pain 2. A decrease in muscle strength 3. An increase in muscle strength 4. Feelings of faintness, dizziness, hypotension, and signs of flushing in the client

3. An increase in muscle strength Rationale: Edrophonium is a short-acting acetylcholinesterase inhibitor used as a diagnostic agent. When a client with suspected myasthenia gravis is given the medication intravenously, an increase in muscle strength would be seen in 1 to 3 minutes. If no response occurs, another dose is given over the next 2 minutes, and muscle strength is again tested. If no increase in muscle strength occurs with this higher dose, the muscle weakness is not caused by myasthenia gravis. Clients receiving injections of this medication commonly demonstrate a drop of blood pressure, feel faint and dizzy, and are flushed.

A 6-month-old infant receives a diphtheria, tetanus, and pertussis (DTaP) immunization at the well-baby clinic. The mother returns home and calls the clinic to report that the infant has developed swelling and redness at the site of injection. The appropriate suggestion to the mother should be to: 1. Apply a warm pack to the injection site. 2. Bring the infant back to the clinic. 3. Apply an ice pack to the injection site. 4. Monitor the infant for a fever.

3. Apply an ice pack to the injection site. Rationale: Occasionally, tenderness, redness, or swelling may occur at the site of the injection. This can be relieved with ice packs for the first 24 hours followed by warm compresses if the inflammation persists. It is not necessary to bring the infant back to the clinic. Option 4 may be an appropriate intervention but is not specific to the subject of the question.

174.) A client with portosystemic encephalopathy is receiving oral lactulose (Chronulac) daily. The nurse assesses which of the following to determine medication effectiveness? 1. Lung sounds 2. Blood pressure 3. Blood ammonia level 4. Serum potassium level

3. Blood ammonia level Rationale: Lactulose is a hyperosmotic laxative and ammonia detoxicant. It is used to prevent or treat portosystemic encephalopathy, including hepatic precoma and coma. It also is used to treat constipation. The medication retains ammonia in the colon (decreases the blood ammonia concentration), producing an osmotic effect. It promotes increased peristalsis and bowel evacuation, expelling ammonia from the colon.

96.) The nurse is caring for a postrenal transplant client taking cyclosporine (Sandimmune, Gengraf, Neoral). The nurse notes an increase in one of the client's vital signs, and the client is complaining of a headache. What is the vital sign that is most likely increased? 1. Pulse 2. Respirations 3. Blood pressure 4. Pulse oximetry

3. Blood pressure Rationale: Hypertension can occur in a client taking cyclosporine (Sandimmune, Gengraf, Neoral), and because this client is also complaining of a headache, the blood pressure is the vital sign to be monitoring most closely. Other adverse effects include infection, nephrotoxicity, and hirsutism. Options 1, 2, and 4 are unrelated to the use of this medication.

731. A client is admitted with pulmonary embolism and is to be treated with streptokinase (Streptase). A nurse would report which of the following assessments to the physician before initiating this therapy?

3. Blood pressure of 198/110 mm Hg

731. A client is admitted with pulmonary embolism and is to be treated with streptokinase (Streptase). A nurse would report which of the following assessments to the physician before initiating this therapy? 1. Adventitious breath sounds 2. Temperature of 99.4° F orally 3. Blood pressure of 198/110 mm Hg 4. Respiratory rate of 28 breaths/min

3. Blood pressure of 198/110 mm Hg

153.) A client is diagnosed with pulmonary embolism and is to be treated with streptokinase (Streptase). A nurse would report which priority data collection finding to the registered nurse before initiating this therapy? 1. Adventitious breath sounds 2. Temperature of 99.4° F orally 3. Blood pressure of 198/110 mm Hg 4. Respiratory rate of 28 breaths/min

3. Blood pressure of 198/110 mm Hg Rationale: Thrombolytic therapy is contraindicated in a number of preexisting conditions in which there is a risk of uncontrolled bleeding, similar to the case in anticoagulant therapy. Thrombolytic therapy also is contraindicated in severe uncontrolled hypertension because of the risk of cerebral hemorrhage. Therefore the nurse would report the results of the blood pressure to the registered nurse before initiating therapy. The findings in options 1, 2, and 4 may be present in the client with pulmonary embolism.

1) A nurse is caring for a client with hyperparathyroidism and notes that the client's serum calcium level is 13 mg/dL. Which medication should the nurse prepare to administer as prescribed to the client? 1. Calcium chloride 2. Calcium gluconate 3. Calcitonin (Miacalcin) 4. Large doses of vitamin D

3. Calcitonin (Miacalcin) Rationale: The normal serum calcium level is 8.6 to 10.0 mg/dL. This client is experiencing hypercalcemia. Calcium gluconate and calcium chloride are medications used for the treatment of tetany, which occurs as a result of acute hypocalcemia. In hypercalcemia, large doses of vitamin D need to be avoided. Calcitonin, a thyroid hormone, decreases the plasma calcium level by inhibiting bone resorption and lowering the serum calcium concentration.

684. A client has been started on long-term therapy with rifampin (Rifadin). A nurse teaches the client that the medication: 1. Should always be taken with food or antacids 2. Should be double-dosed if one dose is for-gotten 3. Causes orange discoloration of sweat, tears, urine, and feces 4. May be discontinued independently if symptoms are gone in 3 months

3. Causes orange discoloration of sweat, tears, urine, and feces

680. Terbutaline (Brethine) is prescribed for a client with bronchitis. A nurse understands that this medication should be used with caution if which of the following medical conditions is present in the client? 1. Osteoarthritis 2. Hypothyroidism 3. Diabetes mellitus 4. Polycystic disease

3. Diabetes mellitus

150.) A client complaining of not feeling well is seen in a clinic. The client is taking several medications for the control of heart disease and hypertension. These medications include a β-blocker, digoxin (Lanoxin), and a diuretic. A tentative diagnosis of digoxin toxicity is made. Which of the following assessment data would support this diagnosis? 1. Dyspnea, edema, and palpitations 2. Chest pain, hypotension, and paresthesia 3. Double vision, loss of appetite, and nausea 4. Constipation, dry mouth, and sleep disorder

3. Double vision, loss of appetite, and nausea Rationale: Double vision, loss of appetite, and nausea are signs of digoxin toxicity. Additional signs of digoxin toxicity include bradycardia, difficulty reading, visual alterations such as green and yellow vision or seeing spots or halos, confusion, vomiting, diarrhea, decreased libido, and impotence. **gastrointestinal (GI) and visual disturbances occur with digoxin toxicity**

148.) A client is taking cetirizine hydrochloride (Zyrtec). The nurse checks for which of the following side effects of this medication? 1. Diarrhea 2. Excitability 3. Drowsiness 4. Excess salivation

3. Drowsiness Rationale: A frequent side effect of cetirizine hydrochloride (Zyrtec), an antihistamine, is drowsiness or sedation. Others include blurred vision, hypertension (and sometimes hypotension), dry mouth, constipation, urinary retention, and sweating.

60.) A nurse is planning to administer hydrochlorothiazide (HydroDIURIL) to a client. The nurse understands that which of the following are concerns related to the administration of this medication? 1. Hypouricemia, hyperkalemia 2. Increased risk of osteoporosis 3. Hypokalemia, hyperglycemia, sulfa allergy 4. Hyperkalemia, hypoglycemia, penicillin allergy

3. Hypokalemia, hyperglycemia, sulfa allergy Rationale: Thiazide diuretics such as hydrochlorothiazide are sulfa-based medications, and a client with a sulfa allergy is at risk for an allergic reaction. Also, clients are at risk for hypokalemia, hyperglycemia, hypercalcemia, hyperlipidemia, and hyperuricemia.

130.) The nurse is analyzing the laboratory results of a client with leukemia who has received a regimen of chemotherapy. Which laboratory value would the nurse specifically note as a result of the massive cell destruction that occurred from the chemotherapy? 1. Anemia 2. Decreased platelets 3. Increased uric acid level 4. Decreased leukocyte count

3. Increased uric acid level Rationale: Hyperuricemia is especially common following treatment for leukemias and lymphomas because chemotherapy results in a massive cell kill. Although options 1, 2, and 4 also may be noted, an increased uric acid level is related specifically to cell destruction.

A client with diabetes mellitus has a foot infection and is to be started on antibiotic therapy with an aminoglycoside. The nurse collects data from the client and notes that the client has a hearing loss. The nurse should take which of the following actions next? 1. Suggest a peak and trough to ensure safe medication administration. 2. Have the client drink extra water to avoid toxic side effects. 3. Inform the registered nurse (RN) about the hearing loss. 4. Give the medication but at half the prescribed dose.

3. Inform the registered nurse (RN) about the hearing loss. Rationale: A preexisting hearing loss is a contraindication for the administration of aminoglycosides because these medications can cause ototoxicity and irreversible hearing loss. The nurse should report the findings to the RN to protect the client's safety. The RN will in turn notify the health care provider. Options 1 and 2 are not beneficial because hearing loss has already occurred in this client. Nurses do not change medication prescriptions independently.

230.) A client is placed on chloral hydrate (Somnote) for short-term treatment. Which nursing action indicates an understanding of the major side effect of this medication? 1. Monitoring neurological signs every 2 hours 2. Monitoring the blood pressure every 4 hours 3. Instructing the client to call for ambulation assistance 4. Lowering the bed and clearing a path to the bathroom at bedtime

3. Instructing the client to call for ambulation assistance Rationale: Chloral hydrate (a sedative-hypnotic) causes sedation and impairment of motor coordination; therefore, safety measures need to be implemented. The client is instructed to call for assistance with ambulation. Options 1 and 2 are not specifically associated with the use of this medication. Although option 4 is an appropriate nursing intervention, it is most important to instruct the client to call for assistance with ambulation.

A nurse is reviewing medications with the client receiving colchicine for the treatment of gout. The nurse determines that the medication is effective if the client reports a decrease in: 1. Blood glucose 2. Blood pressure 3. Joint inflammation 4. Headaches

3. Joint inflammation Rationale: Colchicine is classified as an antigout agent. It interferes with the ability of the white blood cells to initiate and maintain an inflammatory response to monosodium urate crystals. The client should report a decrease in pain and inflammation in the affected joints, as well as a decrease in the number of gout attacks. Colchicine has no effect on the client's blood glucose or blood pressure; it is not used to treat a headache.

241.) A client with a history of simple partial seizures is taking clorazepate (Tranxene), and asks the nurse if there is a risk of addiction. The nurse's response is based on the understanding that clorazepate: 1. Is not habit forming, either physically or psychologically 2. Leads to physical tolerance, but only after 10 or more years of therapy 3. Leads to physical and psychological dependence with prolonged high-dose therapy 4. Can result in psychological dependence only, because of the nature of the medication

3. Leads to physical and psychological dependence with prolonged high-dose therapy Rationale: Clorazepate is classified as an anticonvulsant, antianxiety agent, and sedative-hypnotic (benzodiazepine). One of the concerns with clorazepate therapy is that the medication can lead to physical or psychological dependence with prolonged therapy at high doses. For this reason, the amount of medication that is readily available to the client at any one time is restricted. **Eliminate options 2 and 4 first because of the closed-ended word "only"**

52.) A client with tuberculosis is being started on antituberculosis therapy with isoniazid (INH). Before giving the client the first dose, a nurse ensures that which of the following baseline studies has been completed? 1. Electrolyte levels 2. Coagulation times 3. Liver enzyme levels 4. Serum creatinine level

3. Liver enzyme levels Rationale: INH therapy can cause an elevation of hepatic enzyme levels and hepatitis. Therefore, liver enzyme levels are monitored when therapy is initiated and during the first 3 months of therapy. They may be monitored longer in the client who is greater than age 50 or abuses alcohol.

210.) Dantrolene (Dantrium) is prescribed for a client with a spinal cord injury for discomfort resulting from spasticity. The nurse tells the client about the importance of follow-up and the need for which blood study? 1. Creatinine level 2. Sedimentation rate 3. Liver function studies 4. White blood cell count

3. Liver function studies Rationale: Dantrolene can cause liver damage, and the nurse should monitor liver function studies. Baseline liver function studies are done before therapy starts, and regular liver function studies are performed throughout therapy. Dantrolene is discontinued if no relief of spasticity is achieved in 6 weeks.

723. A client is diagnosed with an acute myocardial infarction and is receiving tissue plasminogen activator, alteplase (Activase, tPA). Which of the following is a priority nursing intervention?

3. Monitor for signs of bleeding.

728. A client is being treated for acute congestive heart failure with intravenously administered bumetanide (Bumex). The vital signs are as follows: blood pressure, 100/60 mm Hg; pulse, 96 beats/min; and respirations, 24 breaths/min. After the initial dose, which of the following is the priority assessment?

3. Monitoring blood pressure

728. A client is being treated for acute congestive heart failure with intravenously administered bumetanide (Bumex). The vital signs are as follows: blood pressure, 100/60 mm Hg; pulse, 96 beats/min; and respirations, 24 breaths/min. After the initial dose, which of the following is the priority assessment? 1. Monitoring weight loss 2. Monitoring urine output 3. Monitoring blood pressure 4. Monitoring potassium level

3. Monitoring blood pressure

151.) A client is being treated for acute congestive heart failure with intravenously administered bumetanide. The vital signs are as follows: blood pressure, 100/60 mm Hg; pulse, 96 beats/min; and respirations, 24 breaths/min. After the initial dose, which of the following is the priority assessment? 1. Monitoring weight loss 2. Monitoring temperature 3. Monitoring blood pressure 4. Monitoring potassium level

3. Monitoring blood pressure Rationale: Bumetanide is a loop diuretic. Hypotension is a common side effect associated with the use of this medication. The other options also require assessment but are not the priority. **priority ABCs—airway, breathing, and circulation**

A nurse is caring for a hospitalized child with a history of seizures who is receiving oral phenytoin sodium (Dilantin). Which of the following should be included in the plan of care for this child? 1. Monitoring intake and output 2. Checking the heart rate before administering the phenytoin 3. Providing oral hygiene especially care of the gums 4. Administering medications 1 hour before food intake

3. Providing oral hygiene especially care of the gums Rationale: Phenytoin sodium causes gum bleeding and hypertrophy, and therefore oral hygiene is important. Soft toothbrushes and gum massage should be instituted to reduce the risk of complications and prevent further trauma. Options 1 and 2 are incorrect because the intake and output as well as heart rate are not affected by this medication. Option 4 is incorrect because directions for administration of this medication include administering with food to minimize gastrointestinal upset.

201.) A nurse is preparing to administer eardrops to an infant. The nurse plans to: 1. Pull up and back on the ear and direct the solution onto the eardrum. 2. Pull down and back on the ear and direct the solution onto the eardrum. 3. Pull down and back on the ear and direct the solution toward the wall of the canal. 4. Pull up and back on the ear lobe and direct the solution toward the wall of the canal.

3. Pull down and back on the ear and direct the solution toward the wall of the canal. Rationale: When administering eardrops to an infant, the nurse pulls the ear down and straight back. In the adult or a child older than 3 years, the ear is pulled up and back to straighten the auditory canal. The medication is administered by aiming it at the wall of the canal rather than directly onto the eardrum.

677. A nurse is preparing to administer a dose of naloxone hydrochloride (Narcan) intravenously to a client with an intravenous opioid overdose. Which supportive medical equipment would the nurse plan to have at the client's bedside if needed? 1. Nasogastric tube 2. Paracentesis tray 3. Resuscitation equipment 4. Central line insertion tray

3. Resuscitation equipment

A client began taking amantadine (Symmetrel) approximately 2 weeks ago. The nurse would evaluate that the medication was having a therapeutic effect if the client exhibited decreased: 1. Voiding 2. Blood pressure 3. Rigidity and akinesia 4. White blood cell count

3. Rigidity and akinesia Rationale: Amantadine is an antiparkinson agent that potentiates the action of dopamine in the central nervous system. The expected effect of therapy is a decrease in akinesia and rigidity. Leukopenia, urinary retention, and hypotension are all adverse effects of the medication.

95.) The nurse is reviewing the results of serum laboratory studies drawn on a client with acquired immunodeficiency syndrome who is receiving didanosine (Videx). The nurse interprets that the client may have the medication discontinued by the health care provider if which of the following significantly elevated results is noted? 1. Serum protein 2. Blood glucose 3. Serum amylase 4. Serum creatinine

3. Serum amylase Rationale: Didanosine (Videx) can cause pancreatitis. A serum amylase level that is increased 1.5 to 2 times normal may signify pancreatitis in the client with acquired immunodeficiency syndrome and is potentially fatal. The medication may have to be discontinued. The medication is also hepatotoxic and can result in liver failure.

46.) A postoperative client has received a dose of naloxone hydrochloride for respiratory depression shortly after transfer to the nursing unit from the postanesthesia care unit. After administration of the medication, the nurse checks the client for: 1. Pupillary changes 2. Scattered lung wheezes 3. Sudden increase in pain 4. Sudden episodes of diarrhea

3. Sudden increase in pain Rationale: Naloxone hydrochloride is an antidote to opioids and may also be given to the postoperative client to treat respiratory depression. When given to the postoperative client for respiratory depression, it may also reverse the effects of analgesics. Therefore, the nurse must check the client for a sudden increase in the level of pain experienced. Options 1, 2, and 4 are not associated with this medication.

58.) Isosorbide mononitrate (Imdur) is prescribed for a client with angina pectoris. The client tells the nurse that the medication is causing a chronic headache. The nurse appropriately suggests that the client: 1. Cut the dose in half. 2. Discontinue the medication. 3. Take the medication with food. 4. Contact the health care provider (HCP).

3. Take the medication with food. Rationale: Isosorbide mononitrate is an antianginal medication. Headache is a frequent side effect of isosorbide mononitrate and usually disappears during continued therapy. If a headache occurs during therapy, the client should be instructed to take the medication with food or meals. It is not necessary to contact the HCP unless the headaches persist with therapy. It is not appropriate to instruct the client to discontinue therapy or adjust the dosages.

80.) A nurse is caring for a client who is taking phenytoin (Dilantin) for control of seizures. During data collection, the nurse notes that the client is taking birth control pills. Which of the following information should the nurse provide to the client? 1. Pregnancy should be avoided while taking phenytoin (Dilantin). 2. The client may stop taking the phenytoin (Dilantin) if it is causing severe gastrointestinal effects. 3. The potential for decreased effectiveness of the birth control pills exists while taking phenytoin (Dilantin). 4. The increased risk of thrombophlebitis exists while taking phenytoin (Dilantin) and birth control pills together.

3. The potential for decreased effectiveness of the birth control pills exists while taking phenytoin (Dilantin). Rationale: Phenytoin (Dilantin) enhances the rate of estrogen metabolism, which can decrease the effectiveness of some birth control pills. Options 1, 2, are 4 are not accurate.

103.) A nurse is caring for a hospitalized client who has been taking clozapine (Clozaril) for the treatment of a schizophrenic disorder. Which laboratory study prescribed for the client will the nurse specifically review to monitor for an adverse effect associated with the use of this medication? 1. Platelet count 2. Cholesterol level 3. White blood cell count 4. Blood urea nitrogen level

3. White blood cell count Rationale: Hematological reactions can occur in the client taking clozapine and include agranulocytosis and mild leukopenia. The white blood cell count should be checked before initiating treatment and should be monitored closely during the use of this medication. The client should also be monitored for signs indicating agranulocytosis, which may include sore throat, malaise, and fever. Options 1, 2, and 4 are unrelated to this medication.

118.) A nurse is caring for an older client with a diagnosis of myasthenia gravis and has reinforced self-care instructions. Which statement by the client indicates that further teaching is necessary? 1. "I rest each afternoon after my walk." 2. "I cough and deep breathe many times during the day." 3. "If I get abdominal cramps and diarrhea, I should call my doctor." 4. "I can change the time of my medication on the mornings that I feel strong."

4. "I can change the time of my medication on the mornings that I feel strong." Rationale: The client with myasthenia gravis should be taught that timing of anticholinesterase medication is critical. It is important to instruct the client to administer the medication on time to maintain a chemical balance at the neuromuscular junction. If not given on time, the client may become too weak to swallow. Options 1, 2, and 3 include the necessary information that the client needs to understand to maintain health with this neurological degenerative disease.

718. A nurse provides discharge instructions to a postoperative client who is taking warfarin sodium (Coumadin). Which statement, if made by the client, reflects the need for further teaching?

4. "I will take Ecotrin (enteric-coated aspirin) for my headaches because it is coated."

718. A nurse provides discharge instructions to a postoperative client who is taking warfarin sodium (Coumadin). Which statement, if made by the client, reflects the need for further teaching? 1. "I will take my pills every day at the same time." 2. "I will avoid alcohol consumption." 3. "I have already called my family to pick up a Medic-Alert bracelet." 4. "I will take Ecotrin (enteric-coated aspirin) for my headaches because it is coated."

4. "I will take Ecotrin (enteric-coated aspirin) for my headaches because it is coated."

725. A home health care nurse is visiting a client with elevated triglyceride levels and a serum cholesterol level of 398 mg/dL. The client is taking cholestyramine (Questran). Which of the following statements, if made by the client, indicates the need for further education? 1. "Constipation and bloating might be a problem." 2. "I'll continue to watch my diet and reduce my fats." 3. "Walking a mile each day will help the whole process." 4. "I'll continue my nicotinic acid from the health food store."

4. "I'll continue my nicotinic acid from the health food store."

62.) A client is on nicotinic acid (niacin) for hyperlipidemia and the nurse provides instructions to the client about the medication. Which statement by the client would indicate an understanding of the instructions? 1. "It is not necessary to avoid the use of alcohol." 2. "The medication should be taken with meals to decrease flushing." 3. "Clay-colored stools are a common side effect and should not be of concern." 4. "Ibuprofen (Motrin) taken 30 minutes before the nicotinic acid should decrease the flushing."

4. "Ibuprofen (Motrin) taken 30 minutes before the nicotinic acid should decrease the flushing." Rationale: Flushing is a side effect of this medication. Aspirin or a nonsteroidal anti-inflammatory drug can be taken 30 minutes before taking the medication to decrease flushing. Alcohol consumption needs to be avoided because it will enhance this side effect. The medication should be taken with meals, this will decrease gastrointestinal upset. Taking the medication with meals has no effect on the flushing. Clay-colored stools are a sign of hepatic dysfunction and should be immediately reported to the health care provider (HCP).

87.) A client with acute muscle spasms has been taking baclofen (Lioresal). The client calls the clinic nurse because of continuous feelings of weakness and fatigue and asks the nurse about discontinuing the medication. The nurse should make which appropriate response to the client? 1. "You should never stop the medication." 2. "It is best that you taper the dose if you intend to stop the medication." 3. "It is okay to stop the medication if you think that you can tolerate the muscle spasms." 4. "Weakness and fatigue commonly occur and will diminish with continued medication use."

4. "Weakness and fatigue commonly occur and will diminish with continued medication use." Rationale: The client should be instructed that symptoms such as drowsiness, weakness, and fatigue are more intense in the early phase of therapy and diminish with continued medication use. The client should be instructed never to withdraw or stop the medication abruptly, because abrupt withdrawal can cause visual hallucinations, paranoid ideation, and seizures. It is best for the nurse to inform the client that these symptoms will subside and encourage the client to continue the use of the medication.

226.) A client receiving lithium carbonate (Lithobid) complains of loose, watery stools and difficulty walking. The nurse would expect the serum lithium level to be which of the following? 1. 0.7 mEq/L 2. 1.0 mEq/L 3. 1.2 mEq/L 4. 1.7 mEq/L

4. 1.7 mEq/L Rationale: The therapeutic serum level of lithium ranges from 0.6 to 1.2 mEq/L. Serum lithium levels above the therapeutic level will produce signs of toxicity.

75.) A client with myasthenia gravis becomes increasingly weak. The health care provider prepares to identify whether the client is reacting to an overdose of the medication (cholinergic crisis) or increasing severity of the disease (myasthenic crisis). An injection of edrophonium (Enlon) is administered. Which of the following indicates that the client is in cholinergic crisis? 1. No change in the condition 2. Complaints of muscle spasms 3. An improvement of the weakness 4. A temporary worsening of the condition

4. A temporary worsening of the condition Rationale: An edrophonium (Enlon) injection, a cholinergic drug, makes the client in cholinergic crisis temporarily worse. This is known as a negative test. An improvement of weakness would occur if the client were experiencing myasthenia gravis. Options 1 and 2 would not occur in either crisis.

722. A nurse is caring for a client receiving a heparin intravenous (IV) infusion. The nurse anticipates that which laboratory study will be prescribed to monitor the therapeutic effect of heparin?

4. Activated partial thromboplastin time

722. A nurse is caring for a client receiving a heparin intravenous (IV) infusion. The nurse anticipates that which laboratory study will be prescribed to monitor the therapeutic effect of heparin? 1. Hematocrit 2. Hemoglobin 3. Prothrombin time 4. Activated partial thromboplastin time

4. Activated partial thromboplastin time

242.) A client who was started on anticonvulsant therapy with clonazepam (Klonopin) tells the nurse of increasing clumsiness and unsteadiness since starting the medication. The client is visibly upset by these manifestations and asks the nurse what to do. The nurse's response is based on the understanding that these symptoms: 1. Usually occur if the client takes the medication with food 2. Are probably the result of an interaction with another medication 3. Indicate that the client is experiencing a severe untoward reaction to the medication 4. Are worse during initial therapy and decrease or disappear with long-term use

4. Are worse during initial therapy and decrease or disappear with long-term use Rationale: Drowsiness, unsteadiness, and clumsiness are expected effects of the medication during early therapy. They are dose related and usually diminish or disappear altogether with continued use of the medication. It does not indicate that a severe side effect is occurring. It is also unrelated to interaction with another medication. The client is encouraged to take this medication with food to minimize gastrointestinal upset. **Eliminate options 2 and 3 first because they are comparable or alike and because of the word "severe" in option 3**

946. A client is scheduled for discharge and will be taking phenobarbital (Luminal) for an extended period. A nurse would place highest priority on teaching the client which of the following points that directly relates to client safety? 1. Take the medication only with meals. 2. Take the medication at the same time each day. 3. Use a dose container to help prevent missed doses. 4. Avoid drinking alcohol while taking this medication.

4. Avoid drinking alcohol while taking this medication.

178.) Methylergonovine (Methergine) is prescribed for a client with postpartum hemorrhage caused by uterine atony. Before administering the medication, the nurse checks which of the following as the important client parameter? 1. Temperature 2. Lochial flow 3. Urine output 4. Blood pressure

4. Blood pressure Rationale: Methylergonovine is an ergot alkaloid used for postpartum hemorrhage. It stimulates contraction of the uterus and causes arterial vasoconstriction. Ergot alkaloids are avoided in clients with significant cardiovascular disease, peripheral disease, hypertension, eclampsia, or preeclampsia. These conditions are worsened by the vasoconstrictive effects of the ergot alkaloids. The nurse would check the client's blood pressure before administering the medication and would follow agency protocols regarding withholding of the medication. Options 1, 2, and 3 are items that are checked in the postpartum period, but they are unrelated to the use of this medication.

237.) A client who is on lithium carbonate (Lithobid) will be discharged at the end of the week. In formulating a discharge teaching plan, the nurse will instruct the client that it is most important to: 1. Avoid soy sauce, wine, and aged cheese. 2. Have the lithium level checked every week. 3. Take medication only as prescribed because it can become addicting. 4. Check with the psychiatrist before using any over-the-counter (OTC) medications or prescription medications.

4. Check with the psychiatrist before using any over-the-counter (OTC) medications or prescription medications. Rationale: Lithium is the medication of choice to treat manic-depressive illness. Many OTC medications interact with lithium, and the client is instructed to avoid OTC medications while taking lithium. Lithium is not addicting, and, although serum lithium levels need to be monitored, it is not necessary to check these levels every week. A tyramine-free diet is associated with monoamine oxidase inhibitors.

A client is receiving anticonvulsant therapy with phenytoin (Dilantin). The nurse plans to monitor the results of which laboratory test closely? 1. Serum sodium 2. Serum potassium 3. Blood urea nitrogen 4. Complete blood cell count

4. Complete blood cell count Rationale: The nurse would monitor the client's complete blood cell counts because hematological side effects of this therapy include aplastic anemia, agranulocytosis, leukopenia, and thrombocytopenia. Other values that warrant monitoring include serum calcium levels and the results of urinalysis, hepatic, and thyroid function tests.

155.) Mycophenolate mofetil (CellCept) is prescribed for a client as prophylaxis for organ rejection following an allogeneic renal transplant. Which of the following instructions does the nurse reinforce regarding administration of this medication? 1. Administer following meals. 2. Take the medication with a magnesium-type antacid. 3. Open the capsule and mix with food for administration. 4. Contact the health care provider (HCP) if a sore throat occurs.

4. Contact the health care provider (HCP) if a sore throat occurs. Rationale: Mycophenolate mofetil should be administered on an empty stomach. The capsules should not be opened or crushed. The client should contact the HCP if unusual bleeding or bruising, sore throat, mouth sores, abdominal pain, or fever occurs because these are adverse effects of the medication. Antacids containing magnesium and aluminum may decrease the absorption of the medication and therefore should not be taken with the medication. The medication may be given in combination with corticosteroids and cyclosporine. **neutropenia can occur with this medication**

117.) A nurse has given the client taking ethambutol (Myambutol) information about the medication. The nurse determines that the client understands the instructions if the client immediately reports: 1. Impaired sense of hearing 2. Distressing gastrointestinal side effects 3. Orange-red discoloration of body secretions 4. Difficulty discriminating the color red from green

4. Difficulty discriminating the color red from green Rationale: Ethambutol causes optic neuritis, which decreases visual acuity and the ability to discriminate between the colors red and green. This poses a potential safety hazard when driving a motor vehicle. The client is taught to report this symptom immediately. The client is also taught to take the medication with food if gastrointestinal upset occurs. Impaired hearing results from antitubercular therapy with streptomycin. Orange-red discoloration of secretions occurs with rifampin (Rifadin).

190.) A child is hospitalized with a diagnosis of lead poisoning. The nurse assisting in caring for the child would prepare to assist in administering which of the following medications? 1. Activated charcoal 2. Sodium bicarbonate 3. Syrup of ipecac syrup 4. Dimercaprol (BAL in Oil)

4. Dimercaprol (BAL in Oil) Rationale: Dimercaprol is a chelating agent that is administered to remove lead from the circulating blood and from some tissues and organs for excretion in the urine. Sodium bicarbonate may be used in salicylate poisoning. Syrup of ipecac is used in the hospital setting in poisonings to induce vomiting. Activated charcoal is used to decrease absorption in certain poisoning situations. Note that dimercaprol is prepared with peanut oil, and hence should be avoided by clients with known or suspected peanut allergy.

159.) A nurse is caring for a client receiving morphine sulfate subcutaneously for pain. Because morphine sulfate has been prescribed for this client, which nursing action would be included in the plan of care? 1. Encourage fluid intake. 2. Monitor the client's temperature. 3. Maintain the client in a supine position. 4. Encourage the client to cough and deep breathe.

4. Encourage the client to cough and deep breathe. Rationale: Morphine sulfate suppresses the cough reflex. Clients need to be encouraged to cough and deep breathe to prevent pneumonia. **ABCs—airway, breathing, and circulation**

955. A home health nurse visits a client. The client gives the nurse a bottle of clomipramine (Anafranil). The nurse notes that the medication has not been taken by the client in 2 months. What behaviors observed in the client would validate noncompliance with this medication? 1. Complaints of insomnia 2. Complaints of hunger and fatigue 3. A pulse rate less than 60 beats/min 4. Frequent handwashing with hot soapy water

4. Frequent handwashing with hot soapy water

236.) A client is being treated for depression with amitriptyline hydrochloride. During the initial phases of treatment, the most important nursing intervention is: 1. Prescribing the client a tyramine-free diet 2. Checking the client for anticholinergic effects 3. Monitoring blood levels frequently because there is a narrow range between therapeutic and toxic blood levels of this medication 4. Getting baseline postural blood pressures before administering the medication and each time the medication is administered

4. Getting baseline postural blood pressures before administering the medication and each time the medication is administered Rationale: Amitriptyline hydrochloride is a tricyclic antidepressant often used to treat depression. It causes orthostatic changes and can produce hypotension and tachycardia. This can be frightening to the client and dangerous because it can result in dizziness and client falls. The client must be instructed to move slowly from a lying to a sitting to a standing position to avoid injury if these effects are experienced. The client may also experience sedation, dry mouth, constipation, blurred vision, and other anticholinergic effects, but these are transient and will diminish with time.

225.) A nursing student is assigned to care for a client with a diagnosis of schizophrenia. Haloperidol (Haldol) is prescribed for the client, and the nursing instructor asks the student to describe the action of the medication. Which statement by the nursing student indicates an understanding of the action of this medication? 1. It is a serotonin reuptake blocker. 2. It inhibits the breakdown of released acetylcholine. 3. It blocks the uptake of norepinephrine and serotonin. 4. It blocks the binding of dopamine to the postsynaptic dopamine receptors in the brain.

4. It blocks the binding of dopamine to the postsynaptic dopamine receptors in the brain. Rationale: Haloperidol acts by blocking the binding of dopamine to the postsynaptic dopamine receptors in the brain. Imipramine hydrochloride (Tofranil) blocks the reuptake of norepinephrine and serotonin. Donepezil hydrochloride (Aricept) inhibits the breakdown of released acetylcholine. Fluoxetine hydrochloride (Prozac) is a potent serotonin reuptake blocker.

17.) The client with ovarian cancer is being treated with vincristine (Oncovin). The nurse monitors the client, knowing that which of the following indicates a side effect specific to this medication? 1. Diarrhea 2. Hair loss 3. Chest pain 4. Numbness and tingling in the fingers and toes

4. Numbness and tingling in the fingers and toes Rationale: A side effect specific to vincristine is peripheral neuropathy, which occurs in almost every client. Peripheral neuropathy can be manifested as numbness and tingling in the fingers and toes. Depression of the Achilles tendon reflex may be the first clinical sign indicating peripheral neuropathy. Constipation rather than diarrhea is most likely to occur with this medication, although diarrhea may occur occasionally. Hair loss occurs with nearly all the antineoplastic medications. Chest pain is unrelated to this medication.

39.) The client with a gastric ulcer has a prescription for sucralfate (Carafate), 1 g by mouth four times daily. The nurse schedules the medication for which times? 1. With meals and at bedtime 2. Every 6 hours around the clock 3. One hour after meals and at bedtime 4. One hour before meals and at bedtime

4. One hour before meals and at bedtime Rationale: Sucralfate is a gastric protectant. The medication should be scheduled for administration 1 hour before meals and at bedtime. The medication is timed to allow it to form a protective coating over the ulcer before food intake stimulates gastric acid production and mechanical irritation. The other options are incorrect.

222.) A nurse has administered a dose of diazepam (Valium) to a client. The nurse would take which important action before leaving the client's room? 1. Giving the client a bedpan 2. Drawing the shades or blinds closed 3. Turning down the volume on the television 4. Per agency policy, putting up the side rails on the bed

4. Per agency policy, putting up the side rails on the bed Rationale: Diazepam is a sedative-hypnotic with anticonvulsant and skeletal muscle relaxant properties. The nurse should institute safety measures before leaving the client's room to ensure that the client does not injure herself or himself. The most frequent side effects of this medication are dizziness, drowsiness, and lethargy. For this reason, the nurse puts the side rails up on the bed before leaving the room to prevent falls. Options 1, 2, and 3 may be helpful measures that provide a comfortable, restful environment, but option 4 is the one that provides for the client's safety needs.

189.) Prostaglandin E1 is prescribed for a child with transposition of the great arteries. The mother of the child asks the nurse why the child needs the medication. The nurse tells the mother that the medication: 1. Prevents hypercyanotic (blue or tet) spells 2. Maintains an adequate hormone level 3. Maintains the position of the great arteries 4. Provides adequate oxygen saturation and maintains cardiac output

4. Provides adequate oxygen saturation and maintains cardiac output Rationale: A child with transposition of the great arteries may receive prostaglandin E1 temporarily to increase blood mixing if systemic and pulmonary mixing are inadequate to maintain adequate cardiac output. Options 1, 2, and 3 are incorrect. In addition, hypercyanotic spells occur in tetralogy of Fallot. **Use the ABCs—airway, breathing, and circulation—to answer the question. The correct option addresses circulation**

70.) Oxybutynin chloride (Ditropan XL) is prescribed for a client with neurogenic bladder. Which sign would indicate a possible toxic effect related to this medication? 1. Pallor 2. Drowsiness 3. Bradycardia 4. Restlessness

4. Restlessness Rationale: Toxicity (overdosage) of this medication produces central nervous system excitation, such as nervousness, restlessness, hallucinations, and irritability. Other signs of toxicity include hypotension or hypertension, confusion, tachycardia, flushed or red face, and signs of respiratory depression. Drowsiness is a frequent side effect of the medication but does not indicate overdosage.

166.) Alendronate (Fosamax) is prescribed for a client with osteoporosis. The client taking this medication is instructed to: 1. Take the medication at bedtime. 2. Take the medication in the morning with breakfast. 3. Lie down for 30 minutes after taking the medication. 4. Take the medication with a full glass of water after rising in the morning.

4. Take the medication with a full glass of water after rising in the morning. Rationale: Precautions need to be taken with administration of alendronate to prevent gastrointestinal side effects (especially esophageal irritation) and to increase absorption of the medication. The medication needs to be taken with a full glass of water after rising in the morning. The client should not eat or drink anything for 30 minutes following administration and should not lie down after taking the medication.

177.) A nurse is assisting in caring for a pregnant client who is receiving intravenous magnesium sulfate for the management of preeclampsia and notes that the client's deep tendon reflexes are absent. On the basis of this data, the nurse reports the finding and makes which determination? 1. The magnesium sulfate is effective. 2. The infusion rate needs to be increased. 3. The client is experiencing cerebral edema. 4. The client is experiencing magnesium toxicity.

4. The client is experiencing magnesium toxicity. Rationale: Magnesium toxicity can occur as a result of magnesium sulfate therapy. Signs of magnesium sulfate toxicity relate to the central nervous system depressant effects of the medication and include respiratory depression; loss of deep tendon reflexes; sudden decrease in fetal heart rate or maternal heart rate, or both; and sudden drop in blood pressure. Hyperreflexia indicates increased cerebral edema. An absence of reflexes indicates magnesium toxicity. The therapeutic serum level of magnesium for a client receiving magnesium sulfate ranges from 4 to 7.5 mEq/L (5 to 8 mg/dL).

99.) The client with acquired immunodeficiency syndrome and Pneumocystis jiroveci infection has been receiving pentamidine isethionate (Pentam 300). The client develops a temperature of 101° F. The nurse does further monitoring of the client, knowing that this sign would most likely indicate: 1. The dose of the medication is too low. 2. The client is experiencing toxic effects of the medication. 3. The client has developed inadequacy of thermoregulation. 4. The result of another infection caused by leukopenic effects of the medication.

4. The result of another infection caused by leukopenic effects of the medication. Rationale: Frequent side effects of this medication include leukopenia, thrombocytopenia, and anemia. The client should be monitored routinely for signs and symptoms of infection. Options 1, 2, and 3 are inaccurate interpretations.

A client has been prescribed cyclobenzaprine (Flexeril) in the treatment of painful muscle spasms accompanying a herniated intervertebral disk. The nurse would withhold the medication and question the prescription if the client had concurrent prescriptions to take: 1. Ibuprofen (Advil) 2. Furosemide (Lasix) 3. Valproic acid (Depakene) 4. Tranylcypromine (Parnate)

4. Tranylcypromine (Parnate) Rationale: The client should not receive cyclobenzaprine if the client has taken monoamine oxidase inhibitors (MAOIs) such as tranylcypromine (Parnate) or phenelzine (Nardil) within the past 14 days. Otherwise, the client could experience hyperpyretic crisis, seizures, or death.

121.) A client who is taking hydrochlorothiazide (HydroDIURIL, HCTZ) has been started on triamterene (Dyrenium) as well. The client asks the nurse why both medications are required. The nurse formulates a response, based on the understanding that: 1. Both are weak potassium-losing diuretics. 2. The combination of these medications prevents renal toxicity. 3. Hydrochlorothiazide is an expensive medication, so using a combination of diuretics is cost-effective. 4. Triamterene is a potassium-sparing diuretic, whereas hydrochlorothiazide is a potassium-losing diuretic.

4. Triamterene is a potassium-sparing diuretic, whereas hydrochlorothiazide is a potassium-losing diuretic. Rationale: Potassium-sparing diuretics include amiloride (Midamor), spironolactone (Aldactone), and triamterene (Dyrenium). They are weak diuretics that are used in combination with potassium-losing diuretics. This combination is useful when medication and dietary supplement of potassium is not appropriate. The use of two different diuretics does not prevent renal toxicity. Hydrochlorothiazide is an effective and inexpensive generic form of the thiazide classification of diuretics. **It is especially helpful to remember that hydrochlorothiazide is a potassium-losing diuretic and triamterene is a potassium-sparing diuretic**

822. A client with trigeminal neuralgia is being treated with carbamazepine (Tegretol), 400 mg orally daily. Which of the following indicates that the client is experiencing a side effect to the medication? 1. Uric acid level, 5 mg/dL 2. Sodium level, 140 mEq/L 3. Blood urea nitrogen level, 15 mg/dL 4. White blood cell count, 3000/mm3

4. White blood cell count, 3000/mm3

b. "These factors may put you at higher risk for myopathy."

A 70-year-old client who is taking several cardiac antidysrhythmic medications has been prescribed simvastatin (Zocor) 80 mg/day. What is essential information for the nurse to teach the client? a. "This dose may lower your cholesterol too much." b. "These factors may put you at higher risk for myopathy." c. "You should not take this drug with cardiac medications." d. "This combination will cause you to have nausea and vomiting."

a. A longer half-life than heparin

A client is prescribed enoxaparin (Lovenox). The nurse knows that low-molecular-weight heparin (LMWH) has what kind of half-life? a. A longer half-life than heparin b. A shorter half-life than heparin c. The same half-life as heparin d. A four-times shorter half-life than heparin

b. "You may experience headaches with this medication."

A client is prescribed gemfibrozil (Lopid) for treatment of hyperlipidemia type IV. What is important for the nurse to teach the client? a. "Take aspirin before the medication if you experience facial flushing." b. "You may experience headaches with this medication." c. "You will need to have weekly blood drawn to assess for hyperkalemia." d. "Cholesterol levels will need to be assessed daily for one week."

b. Blocking angiotensin II from AT1 receptors

A client is prescribed losartan (Cozaar). The nurse teaches the client that an angiotensin II receptor blocker (ARB) acts by doing what? a. Inhibiting angiotensin-converting enzyme b. Blocking angiotensin II from AT1 receptors c. Preventing the release of angiotensin I d. Promoting the release of aldosterone

b. Activated partial thromboplastin time (aPTT) of 120 seconds *normal therapeutic range is 45-75 secs

A client is receiving an intravenous heparin drip. Which laboratory value will require immediate action by the nurse? a. Platelet count of 150,000 b. Activated partial thromboplastin time (aPTT) of 120 seconds c. INR of 1.0 d. Blood urea nitrogen (BUN) level of 12 mg/dL

b. "It usually takes about 3 days to achieve a therapeutic effect for warfarin, so the heparin is continued until the warfarin is therapeutic."

A client is started on warfarin (Coumadin) therapy while still receiving intravenous heparin. The client questions the nurse about the risk for bleeding. How should the nurse respond? a. "Your concern is valid. I will call the doctor to discontinue the heparin." b. "It usually takes about 3 days to achieve a therapeutic effect for warfarin, so the heparin is continued until the warfarin is therapeutic." c. "Because of your valve replacement, it is especially important for you to be anticoagulated. The heparin and warfarin together are more effective than one alone." d. "Because you are now up and walking, you have a higher risk of blood clots and therefore need to be on both medications."

a. "Apply the patch to a nonhairy area of the upper torso or arm."

A client is to be discharged home with a transdermal nitroglycerin patch. Which instruction will the nurse include in the client's teaching plan? a. "Apply the patch to a nonhairy area of the upper torso or arm." b. "Apply the patch to the same site each day." c. "If you have a headache, remove the patch for 4 hours and then reapply." d. "If you have chest pain, apply a second patch next to the first patch."

b. Decrease the intravenous nitroglycerin by 10 mcg/min.

A client receiving intravenous nitroglycerin at 20 mcg/min complains of dizziness. Nursing assessment reveals a blood pressure of 85/40 mm Hg, heart rate of 110 beats/min, and respiratory rate of 16 breaths/min. What is the nurse's priority action? a. Assess the client's lung sounds. b. Decrease the intravenous nitroglycerin by 10 mcg/min. c. Stop the nitroglycerin infusion for 1 hour, and then restart. d. Recheck the client's vital signs in 15 minutes but continue the infusion.

2. Do not stop taking this medication abruptly; the dosage must be decreased gradually if it is discontinued. Rationale: Beta blockers such as propranolol should never be stopped abruptly because of the possible rebound hypertension and increased dysrhythmias that may occur. The nurse may teach the patient to take the medication on an empty stomach and to be cautious with drowsiness while taking beta blockers.

A patient is given a prescription for propranolol (Inderal) 40 mg bid. The most important instruction for the nurse to give this patient is: 1. Take this medication on an empty stomach, as food interferes with its absorption. 2. Do not stop taking this medication abruptly; the dosage must be decreased gradually if it is discontinued. 3. If the patient experiences any disturbances in hearing, the patient should notify the health care provider immediately. 4. The patient may become very sleepy while taking this medication; do not drive.

3. Bruising and epistaxis Rationale: Thrombolytic agents dissolve existing clots rapidly and continue to have effects for 2 to 4 days. All forms of bleeding must be monitored and reported immediately. Skin rash with urticaria, wheezing with labored respirations, and temperature elevation of 100.8º F are not symptoms of adverse effects directly attributed to thrombolytic therapy.

A patient is receiving a thrombolytic agent, alteplase (Activase), following an acute myocardial infarction. Which condition is most likely attributed to thrombolytic therapy with this agent? 1 Skin rash with urticaria 2 Wheezing with labored respirations 3 Bruising and epistaxis 4 Temperature elevation of 100.8º F

4. Whether the patient's pulse and blood pressure are within normal limits Rationale: Potassium channel blockers such as amiodarone, like other antidysrhythmics, may cause significant bradycardia and hypotension. The light-headedness and dizziness may be associated with a drop in cardiac output due to bradycardia and hypotension. The significant finding of dizziness would first be assessed in relation to the known adverse effects of the drug.

A patient was admitted from the emergency department after receiving treatment for dysrhythmias and will be started on amiodarone (Cordarone, Pacerone) because of lack of therapeutic effects from his other antidysrhythmic therapy. When the nurse checks with him in the afternoon, he complains of feeling lightheaded and dizzy. The nurse will first assess: 1. Whether there is the possibility of sleep deprivation from the stress of admission to the hospital 2. Whether an allergic reaction is occurring with anticholinergic-like symptoms 3. Whether the amiodarone level is not yet therapeutic enough to treat the dysrhythmias 4. Whether the patient's pulse and blood pressure are within normal limits

ending in: pril

ACE inhibitor. Antihypertensive. Contraindicated in pregnancy and with use of potassium supplements or potassium-sparing diuretics. Side effects include anxiety, fatigue, insomnia, nervousness, hypotension, palpitations, persistent dry non-productive cough, dyspnea, angioedema, CVA, MI & hypertensive crisis. Nursing considerations: D/C asap if pregnancy is suspected. Administer 1h before meals to increase absorption. Take BP before dose and monitor regularly. Diuretics should be d/c 2-3 days before ACE inhibitor tx. Report peripheral edema, signs of infection, facial swelling, loss of taste or dyspnea. Do not skip doses or stop drug abruptly. Notify provider if persistent dry cough develops. Take antacids 2h before/after ACE inhibitors. Eliminate licorice from diet. Interact with potassium-sparing diuretics, potassium supplments. lithium, digoxin, licorice, sodium substitutes, & high fat foods.

Tetracycline

Anti-infective

vancomycin

Anti-infective given for potentially life threatening infections(MRSA)

Procrit

Antianemic

Nitroglycerin

Antianginal

Atenolol

Antianginal & antihypertensive

Ativan

Antianxiety Agent

BuSpar

Antianxiety Agent

Hydroxyzine (Vistaril), (Atarax)

Antianxiety Agent

Klonopin

Antianxiety Agent

Librium

Antianxiety Agent

Serax

Antianxiety Agent

Valium

Antianxiety Agent

Versed

Antianxiety Agent

Xanax

Antianxiety Agent

Amiodarone

Antiarrhythmic

Dilatin

Antiarryhmic & Anticonvulsant

Digoxin

Antiarrythmic & Inotropic

Atropine sulfate

Anticholinergic Medication

Atrovent

Anticholinergic Medication

Belladonna

Anticholinergic Medication

Benztropine

Anticholinergic Medication

Iprotropium plus albuterol

Anticholinergic Medication

Pro-Banthine

Anticholinergic Medication

Spiriva

Anticholinergic Medication

Lomotil

Antidiarrheal Medication

Paregoric

Antidiarrheal Medication

Propananol (Inderol)

Antidysrhythmics

Tocainide Hydrochloride (Tonocard)

Antidysrhythmics

Verapamil (Calan)

Antidysrhythmics

Reglan

Antiemetic

Thorazine

Antiemetic & Antipsychotic

Compazine

Antiemetic Medications

Phenergan

Antiemetic Medications

Reglan

Antiemetic Medications

Tigan

Antiemetic Medications

Zofran

Antiemetic Medications

Ribavarin (Virazole)

Antiviral

Rimantadine (Flumadine)

Antiviral

ending in: zepam or zolam

Benzodiazepine. Sedative-hypnotic, antianxiety. Contraindicated in pregnancy/lactation, preexisting CNS depression, severe uncontrolled pain, with narrow-angle glaucoma, and hepatic dysfunction. Side effects include decreased mental alertness, drop in BP (especially when standing), dry mouth, ataxia, dizziness, drowsiness, n/v, withdrawal symptoms, and overdose (s/s somnolence, confusion, coma, diminished reflexes & hypotension). Nursing considerations: Start at a low dose & gradually increased until desired results are achieved. Assess degree & manifestation of anxiet before beginning tx. Monitor BP, HR, & RR and provide supportive care as indicated. Prolonged tx can lead to dependence. Take exactly as prescribed. Abrupt withdrawal of med may cause sweating, vomiting, muscle cramps, tremors & convulsions. Avoid alcohol. Interact with CNS depressants, antacids, TCAs, & MAOIs.

ANTILIPIDEMIC AGENTS page 88

Bile Acid sequestrants: eg questrant (blockers) HMG-CoA reductase inhibitor (statins) e.g: lipitor/atorvastatin (inferes with synthesis) side effects: constipation, fat-solubale vitamin deficiency cannot be combined with other meds take PM before bedtime keep assessing diet

ANTICOAGULANT

COUMADIN (warfarin) action: interferes with synthesis of vitamin K-dependent clotting factors used for PE, venous thrombosis, DVT, prophylaxis after MI SIDE EFFECT: bleeding, hemorrhage, alopecia (hairloss) Monitor bleeding times with prothrombin test (PT) Normal is 9-12 seconds Therapeutic levels 1.5 times control (look for more than normal) Antagonist vitamin K (AquaMephyton) Monitor for bleeding (look at puncture sites, IV sites, hematuria) Give PO Avoid aspirin and NSAIDS Pay attention with foods with vitamin K Good diet hx frequently is very important, at every point of contact High in Vitamin K: green vegetables pork rice yogurt fish milk some cheeses Vitamin C DECREASES effectiveness of this med (orange juice) change in vitamin C intake will effect med outcome Vitamin E will INCREASE the effectiveness of this med

ending in: dipine

Calcium channel blocker. Antiarrhythmic. Contraindicated in sick sinus syndrome, second & third degree heart blocks, severe hypotension. Side effects include h/a, fatigue, constipation, postural hypotension, heart block, profound bradycardia, CHF, syncope, palpitations, fluid overload, dizziness, nervousness, insomnia, confusion, tremor, gait disturbance, impaired taste, & skin rash. Nursing considerations: Evaluate BP & ECG before initiation of tx. Monitor for h/a, analgesic may be required. Hold med if BP is less than 90/60. May induce hyperglycemia, monitor diabetic patients closely. Report gradual weight gain & evidence of edema. Take pulse before each dose. Stop smoking and avoid alcohol consumption. Interact with beta blockers, digoxin, furosemide (incompatible in IV solution), grapefruit and grapefruit juice.

721. A nurse is monitoring a client who is taking propranolol (Inderal). Which assessment data would indicate a potential serious complication associated with propranolol?

2. The development of audible expiratory wheezes

721. A nurse is monitoring a client who is taking propranolol (Inderal). Which assessment data would indicate a potential serious complication associated with propranolol? 1. The development of complaints of insomnia 2. The development of audible expiratory wheezes 3. A baseline blood pressure of 150/80 mm Hg followed by a blood pressure of 138/72 mm Hg after two doses of the medication 4. A baseline resting heart rate of 88 beats/min followed by a resting heart rate of 72 beats/min after two doses of the medication

2. The development of audible expiratory wheezes

65.) A nurse is reinforcing discharge instructions to a client receiving sulfisoxazole. Which of the following should be included in the list of instructions? 1. Restrict fluid intake. 2. Maintain a high fluid intake. 3. If the urine turns dark brown, call the health care provider (HCP) immediately. 4. Decrease the dosage when symptoms are improving to prevent an allergic response.

2. Maintain a high fluid intake. Rationale: Each dose of sulfisoxazole should be administered with a full glass of water, and the client should maintain a high fluid intake. The medication is more soluble in alkaline urine. The client should not be instructed to taper or discontinue the dose. Some forms of sulfisoxazole cause urine to turn dark brown or red. This does not indicate the need to notify the HCP.

228.) A client receiving an anxiolytic medication complains that he feels very "faint" when he tries to get out of bed in the morning. The nurse recognizes this complaint as a symptom of: 1. Cardiac dysrhythmias 2. Postural hypotension 3. Psychosomatic symptoms 4. Respiratory insufficiency

2. Postural hypotension Rationale: Anxiolytic medications can cause postural hypotension. The client needs to be taught to rise to a sitting position and get out of bed slowly because of this adverse effect related to the medication. Options 1, 3, and 4 are unrelated to the use of this medication.

25.) A home care nurse visits a client recently diagnosed with diabetes mellitus who is taking Humulin NPH insulin daily. The client asks the nurse how to store the unopened vials of insulin. The nurse tells the client to: 1. Freeze the insulin. 2. Refrigerate the insulin. 3. Store the insulin in a dark, dry place. 4. Keep the insulin at room temperature.

2. Refrigerate the insulin. Rationale: Insulin in unopened vials should be stored under refrigeration until needed. Vials should not be frozen. When stored unopened under refrigeration, insulin can be used up to the expiration date on the vial. Options 1, 3, and 4 are incorrect.

730. A client is receiving thrombolytic therapy with a continuous infusion of streptokinase (Streptase). The client suddenly becomes extremely anxious and complains of itching. A nurse hears stridor and on examination of the client notes generalized urticaria and hypotension. Which of the following should be the priority action of the nurse? 1. Administer oxygen and protamine sulfate. 2. Stop the infusion and call the physician. 3. Cut the infusion rate in half and sit the client up in bed. 4. Administer diphenhydramine (Benadryl) and continue the infusion.

2. Stop the infusion and call the physician.

233.) Diphenhydramine hydrochloride (Benadryl) is used in the treatment of allergic rhinitis for a hospitalized client with a chronic psychotic disorder. The client asks the nurse why the medication is being discontinued before hospital discharge. The nurse responds, knowing that: 1. Allergic symptoms are short in duration. 2. This medication promotes long-term extrapyramidal symptoms. 3. Addictive properties are enhanced in the presence of psychotropic medications. 4. Poor compliance causes this medication to fail to reach its therapeutic blood level.

3. Addictive properties are enhanced in the presence of psychotropic medications. Rationale: The addictive properties of diphenhydramine hydrochloride are enhanced when used with psychotropic medications. Allergic symptoms may not be short term and will occur if allergens are present in the environment. Poor compliance may be a problem with psychotic clients but is not the subject of the question. Diphenhydramine hydrochloride may be used for extrapyramidal symptoms and mild medication-induced movement disorders.

122.) A client who has begun taking fosinopril (Monopril) is very distressed, telling the nurse that he cannot taste food normally since beginning the medication 2 weeks ago. The nurse provides the best support to the client by: 1. Telling the client not to take the medication with food 2. Suggesting that the client taper the dose until taste returns to normal 3. Informing the client that impaired taste is expected and generally disappears in 2 to 3 months 4. Requesting that the health care provider (HCP) change the prescription to another brand of angiotensin-converting enzyme (ACE) inhibitor

3. Informing the client that impaired taste is expected and generally disappears in 2 to 3 months Rationale: ACE inhibitors, such as fosinopril, cause temporary impairment of taste (dysgeusia). The nurse can tell the client that this effect usually disappears in 2 to 3 months, even with continued therapy, and provide nutritional counseling if appropriate to avoid weight loss. Options 1, 2, and 4 are inappropriate actions. Taking this medication with or without food does not affect absorption and action. The dosage should never be tapered without HCP approval and the medication should never be stopped abruptly.

what size needle would you use to administer vit. b12 to an average size person

22 gauge 1inch needle

the appropriate size needle for subcutaneanous injections

25 gauge, 5/8 inch needle

Which statement, made by the patient, demonstrates a knowledge deficit regarding colestipol (Colestid)?

"I should mix and stir the powder in as small an amount of fluid as possible in order to maintain potency of the medication."

Which statement indicates that the patient understands discharge teaching about nitroglycerin?

"I will need to keep the nitroglycerin stored in the bottle it comes in."

Patient teaching for a patient being discharged on a beta-blocker includes which statement?

"If you take your pulse and it is less than 60, hold your medicine and call your health care provider for instructions."

When administering nitroprusside (Nipride) intravenously, the nurse would monitro for which sign of toxicity? Select all that apply

*Abdominal pain *Diaphoresis *Muscle twitching *Hypertension

For which potential side effects would the nurse monitor patients prescribed amiodarone? Select all that apply

*Bluish skin discoloration *Hypothyroidism *Photosensitivity

Side effects to expect from anticholinergic (parasympatholytic) drugs, such as atropine, include which effects? (Select all that apply)

*Dilated pupils (mydriasis) *Urinary retention *Dry mouth

Acetazolamide (Diamox) is used to treat which disorder? Select all that apply

*Edema associated with heart failure *High-altitude sickness *Open-angle glaucoma

Which are common side effects of nitroglycerin? Select all that apply

*Flushing *Headache *Hypotension

Beta-blockers are used to treat which disorders? (Select all that apply)

*Hypertension *Angina pectoris *Cardiac dysrhythmias

A patient taking spironolactone (Aldacone) requests assistance with dietary choices. The nurse would recomment which food choices? (Select all that apply)

*Lean meat *Apples *Squash

A patient presents with symptomatic bardycardia. The nurse prepared to administer which dose of atropine intravenously?

0.5 mg

lithium (Eskalith)

0.5-1.5

digoxin (Lanoxin)

0.5-2.0

61.) A home health care nurse is visiting a client with elevated triglyceride levels and a serum cholesterol level of 398 mg/dL. The client is taking cholestyramine (Questran). Which of the following statements, if made by the client, indicates the need for further education? 1. "Constipation and bloating might be a problem." 2. "I'll continue to watch my diet and reduce my fats." 3. "Walking a mile each day will help the whole process." 4. "I'll continue my nicotinic acid from the health food store."

4. "I'll continue my nicotinic acid from the health food store." Rationale: Nicotinic acid, even an over-the-counter form, should be avoided because it may lead to liver abnormalities. All lipid-lowering medications also can cause liver abnormalities, so a combination of nicotinic acid and cholestyramine resin is to be avoided. Constipation and bloating are the two most common side effects. Walking and the reduction of fats in the diet are therapeutic measures to reduce cholesterol and triglyceride levels.

A transcutaneous electrical nerve stimulation (TENS) is prescribed for a client with pain, and the nurse provides information to the client about TENS unit. Which statement by the client indicates the need for further information? 1. "Electrodes are attached to the skin." 2. "The unit relieves pain." 3. "The unit will reduce the needs for analgesics." 4. "Needles are inserted in the subcutaneous tissue to stimulate the nerve."

4. "Needles are inserted in the subcutaneous tissue to stimulate the nerve." Rationale: The TENS unit is a portable unit, and the client controls the system for relieving pain and reducing the need for analgesics. It is attached to the skin of the body by electrodes. Needles are not used.

218.) A film-coated form of diflunisal has been prescribed for a client for the treatment of chronic rheumatoid arthritis. The client calls the clinic nurse because of difficulty swallowing the tablets. Which initial instruction should the nurse provide to the client? 1. "Crush the tablets and mix them with food." 2. "Notify the health care provider for a medication change." 3. "Open the tablet and mix the contents with food." 4. "Swallow the tablets with large amounts of water or milk."

4. "Swallow the tablets with large amounts of water or milk." Rationale: Diflunisal may be given with water, milk, or meals. The tablets should not be crushed or broken open. Taking the medication with a large amount of water or milk should be tried before contacting the health care provider.

163.) A client with trigeminal neuralgia tells the nurse that acetaminophen (Tylenol) is taken on a frequent daily basis for relief of generalized discomfort. The nurse reviews the client's laboratory results and determines that which of the following indicates toxicity associated with the medication? 1. Sodium of 140 mEq/L 2. Prothrombin time of 12 seconds 3. Platelet count of 400,000 cells/mm3 4. A direct bilirubin level of 2 mg/dL

4. A direct bilirubin level of 2 mg/dL Rationale: In adults, overdose of acetaminophen (Tylenol) causes liver damage. Option 4 is an indicator of liver function and is the only option that indicates an abnormal laboratory value. The normal direct bilirubin is 0 to 0.4 mg/dL. The normal platelet count is 150,000 to 400,000 cells/mm3. The normal prothrombin time is 10 to 13 seconds. The normal sodium level is 135 to 145 mEq/L.

4.) The camp nurse asks the children preparing to swim in the lake if they have applied sunscreen. The nurse reminds the children that chemical sunscreens are most effective when applied: 1. Immediately before swimming 2. 15 minutes before exposure to the sun 3. Immediately before exposure to the sun 4. At least 30 minutes before exposure to the sun

4. At least 30 minutes before exposure to the sun Rationale: Sunscreens are most effective when applied at least 30 minutes before exposure to the sun so that they can penetrate the skin. All sunscreens should be reapplied after swimming or sweating.

A health care provider is planning to administer a skeletal muscle relaxant to a client with a spinal cord injury. The medication is going to be administered intrathecally. Which of the following medications should the nurse expect to be prescribed and administered by this route? 1. Cyclobenzaprine hydrochloride (Flexeril) 2. Chlorzoxazone (Paraflex) 3. Dantrolene sodium (Dantrium) 4. Baclofen (Lioresal)

4. Baclofen (Lioresal) Rationale: Baclofen is the only skeletal muscle relaxant that can be administered intrathecally. Therefore options 1, 2, and 3 are incorrect.

A client on the nursing unit has a prescription for dextroamphetamine (Dexedrine) orally daily. The nurse collaborates with the dietitian to limit the amount of which of the following items on the client's dietary trays? 1. Fat 2. Protein 3. Starch 4. Caffeine

4. Caffeine Rationale: Dextroamphetamine is a central nervous system (CNS) stimulant. Caffeine is a stimulant also and should be limited in the client taking this medication. The client should be taught to limit caffeine intake as well.

231.) A client admitted to the hospital gives the nurse a bottle of clomipramine (Anafranil). The nurse notes that the medication has not been taken by the client in 2 months. What behaviors observed in the client would validate noncompliance with this medication? 1. Complaints of hunger 2. Complaints of insomnia 3. A pulse rate less than 60 beats per minute 4. Frequent handwashing with hot, soapy water

4. Frequent handwashing with hot, soapy water Rationale: Clomipramine is commonly used in the treatment of obsessive-compulsive disorder. Handwashing is a common obsessive-compulsive behavior. Weight gain is a common side effect of this medication. Tachycardia and sedation are side effects. Insomnia may occur but is seldom a side effect.

59.) A client is diagnosed with an acute myocardial infarction and is receiving tissue plasminogen activator, alteplase (Activase, tPA). Which action is a priority nursing intervention? 1. Monitor for renal failure. 2. Monitor psychosocial status. 3. Monitor for signs of bleeding. 4. Have heparin sodium available.

3. Monitor for signs of bleeding. Rationale: Tissue plasminogen activator is a thrombolytic. Hemorrhage is a complication of any type of thrombolytic medication. The client is monitored for bleeding. Monitoring for renal failure and monitoring the client's psychosocial status are important but are not the most critical interventions. Heparin is given after thrombolytic therapy, but the question is not asking about follow-up medications.

chapter 67- neuro 816. Carbidopa-levodopa (Sinemet) is prescribed for the client with Parkinson's disease. The nurse monitors the client for side effects to the medication. Which of the following would indicate that the client is experiencing a side effect? 1. Pruritus 2. Tachycardia 3. Hypertension 4. Impaired voluntary movements

4. Impaired voluntary movements

at what rate should TPN solution be started

50 ml/hr

NURSING IMPLICATIONSof ace inhibitors

: Administer 1 hr before meals to increase absorption

Beta adrenergic blockers

<BP, pulse & CO

b. Elevated INR range *therapeutic range is 2.0-3.0

A client is taking warfarin 5 mg/day for atrial fibrillation. The client's international normalized ration (INR) is 3.8. The nurse would consider the INR to be what? a. Within normal range b. Elevated INR range c. Low INR range d. Low average INR range

a. Have the client lie down when taking a nitroglycerin sublingual tablet. b. Teach client to repeat taking a tablet in 5 minutes if chest pain persists. e. Warn client against ingesting alcohol while taking nitroglycerin.

A client who has angina is prescribed nitroglycerin. The nurse reviews which appropriate nursing interventions for nitroglycerin (Select all that apply.) a. Have the client lie down when taking a nitroglycerin sublingual tablet. b. Teach client to repeat taking a tablet in 5 minutes if chest pain persists. c. Apply Transderm-Nitro patch to a hairy area to protect skin from burning. d. Call the health care provider after taking 5 tablets if chest pain persists. e. Warn client against ingesting alcohol while taking nitroglycerin.

a. Call the health care provider to switch the medication.

A nurse is caring for a client who is taking an angiotensin-converting enzyme inhibitor and develops a dry, nonproductive cough. What is the nurse's priority action? a. Call the health care provider to switch the medication. b. Assess the client for other symptoms of upper respiratory infection. c. Instruct the client to take antitussive medication until the symptoms subside. d. Tell the client that the cough will subside in a few days.

ending in: terol

Beta-adrenergic bronchodilator. Contraindicated in tachyarrhythmias, severe CAD, cardiovascular disease, & children under 12 years old (with the inhaled forms). Side effects include nervousness, tremors, increased heart rate, increased BP, insomnia, restlessness, anorexia, cardiac stimulation, vascular h/a, anginal pain, hypotension, arrhythmias, paradoxic bronchospasm, urinary retention, & agitation. Nursing considerations: Monitor elderly clients closely. Monitor BP & pulse closely. Ensure proper use of MDIs. Note amount, color & character of sputum. Monitor blood glucose levels in diabetic patients. Avoid IM injections. Report any chest pain, palpitations, seizures, h/a, hallucinations or blurred vision. Teach proper care of nebulizer and/or inhaler. Wait 1-3 minutes between inhalations of aerosol meds. Take exactly as ordered, do not double-up doses or increase frequency. Increase fluid intake if not otherwise contraindicated. Interact with MAOIs, sympathomimetics, beta blockers, potassium-losing diuretics, cardiac glycosides, other bronchodilators, digoxin, thyroid hormones, decongestants, & antihistamines.

Singulair

Bronchodilator

Cefepime (maxipime)

Cephalosporin 4th generation

CARDIOVASCULAR

Digoxin (Lanoxin): Signs of toxicity: Pt will complain of visual change in colors. They would also complain of loss of appetite.

ANTI-CONVULSANTS:

Dilantin: Causes gum hyperplasia. Advice client to visit dentist frequently

The nurse would monitro for reflex tachycardia in a patient receiving which classification of antihypertensive drugs?

Direct-acting vasodilators

-ide

Diuretics ('P" it out ==> decreases excess fluid; decrease heart workload) tx: Hypertension, Edema

When explaining different medication regimens to treat hypertension during a community education program, it would be accurate to state that African-American probably respond best to which combination of medications?

Diuretics and calcium channel blockers

Aspirin

Do not give together with other anticoagulants. Stop taking Aspirin some days before surgery. Do not give to children with viral infection(Reye syndrome)

Calcium carbonate

Electrolyte and Replacement solution

Calcium chloride

Electrolyte and Replacement solution

K-Dur, Slow K, Micro-K

Electrolyte and Replacement solution

Kaon Liquid

Electrolyte and Replacement solution

MgCl, Slow mag

Electrolyte and Replacement solution

Sodium chloride

Electrolyte and Replacement solution

ANTIBIOTICS

General Information allergy super infection GI, GU, organ toxicity: liver & kidneys teachings: taking it till its gone culture and sensitivity encourage fluids check expiration dates (meds will work but will hurt organs more)

Propantheline (Pro-Banthine)

Genitourinary Medication, Anticholinergic

Finasteride (Proscar, Propecia)

Genitourinary Medication, Testosterone inhibitor

ANTICOAGULANTS

HEPARIN Action: blocks conversion of prothrombin to thrombin used for PE, DVT, prophylaxis after MI RISK = BLEEDING SIDE EFFECTS: Hematuria (blood in urine) tissue irritation Monitor clotting time PTT, normal is 20-45 sec Therapeutic levels 1.5 - 2.5 times control Antagonist - protamine sulfate !!!!!! (produced by sperm of salmon) SC or IV (not given PO destroyed by GI tract) Avoid use of aspirin and NSAIDS

-statin

HMG CoA reductase inhibitor, lipid lowering agent * atorvastatin, simvastatin

-statin

HMG-CoA reductase inhibitor; statins

The nurse would question an order for simvastatin (Zocor) in a patient with which condition?

Hepatic disease

What is the mechanism of action of ezetimibe (Zetia)?

Inhibiting absorption of dietary and biliary cholesterol in the small intestine

MAO

Inhibitor Anti-depressant

Milk of Magnesia

Laxatives and Stool Softeners

Mineral oil

Laxatives and Stool Softeners

Phenolphthalein (Feen-A-Mint)

Laxatives and Stool Softeners

Polyethylene glycol and electrolytes (Colyte)

Laxatives and Stool Softeners

When teaching a patient about signs and symptoms of hypokalemia, the nurse will instruct the patient to notify the health care provider if which occurs?

Leg cramps

ending in: mycin

Macrolide. Antibiotic. Contraindicated in liver/renal dysfunction, GI disorders, UC/enteritis, and in infants/elderly. Side effects include palpitations, chest pain, dizziness, vertigo, lethargy, somnolence, confusion, hearing loss, stomatitis, hepatotoxicity, ototoxicity, nephrotoxicity. Nursing considerations: Assess GI function. Observe for bleeding. Increase protein in diet. Interact with anticoagulants, antihistamines, penicillins.

1,2,3,4 1. teaching the patient or family to give sulx.utaneous injections at home. 2. teaching the patient or family nol to ta~ any OTC drugs without first consulting with the health care provider. 3. teaching the patient to observe for unexplained bleeding such as pink, red, or dark brown urine or bloody gums. 4. teaching the patient to monitor for the development of DVT.

Nursing interventions for a patient receiving enoxaparin (Lovenox) may include: (Select all that apply.) 1. teaching the patient or family to give sulx.utaneous injections at home. 2. teaching the patient or family nol to ta~ any OTC drugs without first consulting with the health care provider. 3. teaching the patient to observe for unexplained bleeding such as pink, red, or dark brown urine or bloody gums. 4. teaching the patient to monitor for the development of DVT. 5. teaching the importance of drinking grapefruit juice daily.

Ambien

Sedative/Hypnotic

Which would be a priorit assessment prior to administering eplerenone (Inspra)?

Serum electrolytes

Novolin R

Short-Acting Insulin

ANTIDIARRHEALS page 83

Action: slow peristalsis, increase tone of sphincters lomotil (diphenoxylate/atropine), Kaopectate (bismuth subsalicate), immodium (loperamide) side effect: constipation may have anticholinergic effects dizziness, drowsiness DO NOT USE WITH ABDOMINAL PAIN Monitor for urinary retention Give 2 hr before or 3 hr after meds

Calcium carbonate (Titralac)

Antacid Medication

Keflex

Anti-infective

Atropine

Antiarrhythmic

Pronestyl

Antiarrhythmic

Amphotericin

Antifungal

Diflucan

Antifungal Medication

Mycostatin

Antifungal Medication

ending in: azole

Antifungal antimicrobial. Fungicide. Side effects include fever, chills, shaking, myalgia, arthralgia, weakness, hypotension, insomnia, vertigo, ototoxicity, nephrotoxicity, Stevens-Johnson syndrome, hepatic necrosis, cardiovascular collapse. Nursing considerations: Risk of cardiac toxicity. Use with caution in renal impairment & severe bone marrow suppression. Take with food to increase absoprtion & decrease n/v. Monitor serum levels. Eat small frequent meals with increased protein. Interact with anticoagulants, corticosteriods, nephrotoxic drugs, antidiabetic agents, antacids, benzodiazepines, oral contraceptives, tetracyclines, acetaminophen, Rifampin, Dilantin, & cyclosporine.

Allopurinol

Antigout Agent & Antihyperuricemic

-prin, -dine, -grel, -mole

Antiplatelets (keeps blood platelets from forming by preventing them from sticking together)

Flagyl

Antiprotozoas, Anti-infective & Antiulcer

Clozaril

Antipsychotic

Haloperidol

Antipsychotic

AZT

Antiretroviral

Remicade

Antirheumatic

Protonix

Antiulcer Agent

Tagamet

Antiulcer Agent

Acyclovir

Antiviral

-ipine

antihypertensives also diltiazem, verapamil calcium channel blocker-inhibit calcium influx across cell membrane to slow conduction and dilate coronary vessels and decrease HR uses: angina, HTN, interstitial cystitis adfx: hypotn, GI upset SR-do not chew, crush, open

-artan

antihypertensives angiotensin II-receptors uses: HTN, heart failure, MI, neuropathy, stroke prophylaxis adfx: dizziness, GI distress SR-do not chew, crush, open

hormonal agents

antineoplastic- block hormone input into CA cells ex: tomoxifen, testosterone use: hormone-sensitive CA adfx: leukopenia, bone pain, hypercalcemia, effects of individual hormone

-vudine

antiviral; nucleoside analogues

Benzodiazepine

anxiety & seizures

adrenergic stimulants adverse reactions

anxiety apprehension headache cerebral hemorrhage

antitussive

any medicine used to suppress or relieve coughing

Chapter 27 1. The nurse realizes that facial grimacing, involuntary upward eye movement, and muscle spasms of the tongue and face are indicative of which condition? a. Akathisia b. Acute dystonia c. Tardive dyskinesia d. Pseudoparkinsonism

b. Acute dystonia

synergistic

drugs that work together so the total effect is greater than if given seperatly

Anticholinergic Effects assessment

dry mouth blurred vision nasal congestion urinary retention heart burn impotence

ANTIDYSRHYTHMICS

e.g.: Atropine sulfate, lidocaine, pronestyl (procainamide), quinidine, Isuprel (Isoproterenol) Sub category: beta blockers (unique side effect: bronchospasm-check always airway) action: interfere with electrical excitability of heart Uses: a fib and flutter, tachycardia, PVC's side effects: light headiness, BRADYCARDIA, HYPOTENSION, urinary retention nursing considerations: Monitor VS especial BP and pulses Monitor Cardiac rhythm POSITION CHANGES EDUCATE SLOW FROM LAYING TO SITTING, SITTING TO STANDING ETC (safety consideration)

when should nurse assess pt. TPN insertion site for infection and patency

each shift

Bone Marrow Depression Assessment

fever chills sore throat back pain dark urine low blood cells

Superinfection assessment

fever diarrhea black hairy tounge vaginal itching/discharge

Liver Impairment Assessment

fever malaise n/v jaundice light stools, dark urine elevated bilirubin

MACROLIDES (antibiotics)

sample: eurythromycin, azethromycin, clindamycin acute infections, acne, URI, prophylaxis before dental procedures if allergic to PENICILLIN Different from other mycins: Side effects: diarrhea, CONFUSION, HEPATOXICITY TAKE BEFORE MEALS AND NOT WITH FOODS TAKE WITH FULL GLASS OF WATER NEVER FRUIT JUICE LIVER WILL CORRECT DAMAGE OVER TIME INCREASES COUMADIN INCREASES THEOPHILLIN

a patient with hyperthyroidism is given phenobarbital to achieve what?

sedation

antihistamines adverse reactions

sedation, dry mouth, blurred vision, urinary retention

-setron

serotonin 5-HT3 receptor antagonist; antiemetic and antinauseant

Disulfiram (Antabuse)

Used for alcohol aversion therapy. Clients started on Disulfiram must avoid any form of alcohol or they would develop a severe reaction. Teach pt to avoid some over-the-counter cough preparations, mouthwash etc.

Which is an indication for use of the antidysrhythmic drug lidocaine?

Ventricular arrhythmias

d. "This medication will work for 24 hours and you will need to change the patch daily."

What statement is the most important for the nurse to include in the teaching plan for a client who has started on a transdermal nitroglycerin patch? a. "This medication works faster than sublingual nitroglycerin works." b. "This medication is the strongest of any nitroglycerin preparation available." c. "This medication should be used only when you are experiencing chest pain." d. "This medication will work for 24 hours and you will need to change the patch daily."

c. Hydrochlorothiazide

What would cause the same client's electrolyte imbalance? a. High dose of digoxin b. Digoxin taken daily c. Hydrochlorothiazide d. Low dose of hydrochlorothiaizde

b. Headaches

When a client first takes a nitrate, the nurse expects which symptom that often occurs? a. Nausea and vomiting b. Headaches c. Stomach cramps d. Irregular pulse rate

a. Inhibits absorption of dietary cholesterol in the intestines.

When a client is taking ezetimibe (Zetia), she asks the nurse how it works. The nurse should explain that Zetia does what? a. Inhibits absorption of dietary cholesterol in the intestines. b. Binds with bile acids in the intestines to reduce LDL levels. c. Inhibits HMG-CoA reductase, which is necessary for cholesterol production in the liver. d. Forms insoluble complexes and reduces circulating cholesterol in blood.

a. Coronary thrombosis b. Acute myocardial infarction c. Deep vein thrombosis (DVT) d. Cerebrovascular accident (CVA) (stroke) e. Venous disorders

When a newly admitted client is placed on heparin, the nurse acknowledges that heparin is effective for preventing new clot formation in clients who have which disorder(s)? (Select all that apply.) a. Coronary thrombosis b. Acute myocardial infarction c. Deep vein thrombosis (DVT) d. Cerebrovascular accident (CVA) (stroke) e. Venous disorders

Pulse Rationale: In the absence of ECG monitoring, the nurse would assess the pulse for rate, regularity, quality, and volume, noting any changes. The nurse should also teach the patient to monitor the pulse for rate and regularity, before sending the patient home. The nurse is monitoring for the therapeutic effects of antidysrhythmic therapy.

When monitoring for therapeutic effect of any antidysrhythmic agent, the nurse would be sure to assess: 1.Pulse 2.Blood pressure 3.Drug level 4.Hourly urine output

a. Loss of appetite with slight bradycardia

Which assessment finding will alert the nurse to suspect early digoxin toxicity? a. Loss of appetite with slight bradycardia b. Blood pressure 90/60 mm Hg c. Heart rate 110 beats per minute d. Confusion and diarrhea

c. The client who has stopped taking a beta blocker due to cost.

Which client will the nurse assess first? a. The client who has been on beta blockers for 1 day. b. The client who is on a beta blocker and a thiazide diuretic. c. The client who has stopped taking a beta blocker due to cost. d. The client who is taking a beta blocker and Lasix (furosemide).

c. A 47-year-old client with anuria

Which client would the nurse need to assess first if the client is receiving mannitol (Osmitrol)? a. A 67-year-old client with type 1 diabetes mellitus b. A 21-year-old client with a head injury c. A 47-year-old client with anuria d. A 55-year-old client receiving cisplatin to treat ovarian cancer

b. "I will increase fiber in my diet."

Which statement indicates the client understands discharge instructions regarding cholestyramine (Questran)? a. "I will take Questran 1 hour before my other medications." b. "I will increase fiber in my diet." c. "I will weigh myself weekly." d. "I will have my blood pressure checked weekly."

d. "I should stir the powder in as small an amount of fluid as possible to maintain potency of the medication."

Which statement indicates to the nurse that the client needs further medication instruction about colestipol (Colestid)? a. "The medication may cause constipation, so I will increase fluid and fiber in my diet." b. "I should take this medication 1 hour after or 4 hours before my other medications." c. "I might need to take fat-soluble vitamins to supplement my diet." d. "I should stir the powder in as small an amount of fluid as possible to maintain potency of the medication."

d. "I should sit or lie down after I take a nitroglycerin tablet to prevent dizziness."

Which statement indicates to the nurse that the client understands sublingual nitroglycerin medication instructions? a. "I will take up to five doses every 3 minutes for chest pain." b. "I can chew the tablet for the quickest effect." c. "I will keep the tablets locked in a safe place until I need them." d. "I should sit or lie down after I take a nitroglycerin tablet to prevent dizziness."

Ovral

Women's Health Medication

Premarin

Women's Health Medication

ending in: phylline

Xanthine bronchodilator. Contraindicated in patients with tachyarrhythmias, PUD, GI disorders, cardiovascular disorders (use with caution), seizure disorders, and hyperthyroidism. Side effects include anorexia, GERD, sinus tachycardia, extrasystoles, palpitations, ventricular dysrhythmias, hyperglycemia, transient increased urination, tremors, dizziness, hallucinations, restlessness, agitation, h/a, insomnia, & chest pain. Nursing considerations: Administer cautiously in elderly due to potential for increased sensitivity. Give to children over 6 months of age with caution. Dosages should be based on lean body weight as it does not enter adipose tissue. Start dosage low and titrate up until therapeutic response is achieved. Do not omit or double-up doses, take exactly as prescribed. Avoid use of caffeine as this can increase CNS stimulation. Increase fluid intake if not contraindicated. Interact with allopurinol, cimetidine, erythromycin, flu vax, oral contraceptives, sympathomimetics, smoking, barbiturates, phenobarbital, caffeine, and tobacco.

lithium

a medication used to treat bi-polar disorders

fosamax

a medication used to treat osteoporosis, limit dairy products while taking this med

When assessing for cardiovascular effects of a beta-blocker, the nurse understands that these drugs produce

a negative inotropic, negative chronotropic, and negative dromotropic effect

When assessing for cardiovascular effects of an adrenerigic (sympathomimetic) drug, the nurse understands that these drugs produce

a positive inotropic, positive chronotropic, and positive dromotropic effect

BUN

a test ordered for suspected renal disease

valium

a tranquilizer used to relieve anxiety and relax muscles

An 8-month old boy is discharged from the hospital with a plan of care to receive intramuscular (IM) injections each day. The parents have been taught how to administer IM injections. Which statement, if verbalized by the parents, indicates a need for more teaching? a. "I need to administer this medication in the upper, outer quadrant of this buttocks" b. "IM injections are safe for children if administered correctly" c. "When I give my child this injection, the safest place for insertion is in the thigh" d. "I will need someone to assist me to hold my child while I give the injection"

a. "I need to administer this medication in the upper, outer quadrant of this buttocks"

The client asks about disposal of medications. What are the nurse's best responses? (select all that apply) a. "You should mix medications with coffee grounds before disposal" b. "you should pour medications down the sink" c. "you should remove identifying information on the original container" d. "you should pulverize all tablets before disposal"

a. "You should mix medications with coffee grounds before disposal" c. "you should remove identifying information on the original container"

The nurse reviews the client's medications as part of the initial interview for admission to the cardiac clinic. Which comment by the client indicates a need for health teaching? (select all that apply) a. "tetracycline does not affect my medications" b. "I can take as much calcium as I want" c. "calcium increases the effects of my digoxin" d. Magnesium and potassium deficits can cause digoxin toxicity

a. "tetracycline does not affect my medications" b. "I can take as much calcium as I want"

5. The nurse explains that which beta blocker category is preferred for treating hypertension?

a. Beta1 blocker

7. A client appears to have had an overdose of phenothiazines. The nurse is aware that the potential treatment for phenothiazine overdose includes which intervention(s)? (Select all that apply.) a. Gastric lavage b. Adequate hydration c. Maintaining an airway d. fluphenazine (Prolixin) e. risperidone (Risperdal) f. Activated charcoal administration

a. Gastric lavage b. Adequate hydration c. Maintaining an airway f. Activated charcoal administration

The client has been vomiting and has weak, flabby muscles. The client's pulse is irregular. The nurse would correctly suspect what type of imbalance? a. Hypokalemia b. Hyperkalemia c. Hypocalcemia d. hypercalcemia

a. Hypokalemia

7. For the client who is taking nalbuphine (Nubain), what should the nurse do? (Select all that apply.) a. Monitor any changes in respirations. b. Instruct the client to report bradycardia. c. Administer IV nalbuphine undiluted. d. Explain to the client to expect an excessive amount of urine output. e. Instruct the client to avoid alcohol when taking nalbuphine to avoid respiratory depression.

a. Monitor any changes in respirations. c. Administer IV nalbuphine undiluted. e. Instruct the client to avoid alcohol when taking nalbuphine to avoid respiratory depression.

2.The nurse will monitor the client taking albuterol (Proventil) for which condition? a. Palpitations b. Hypoglycemia c. Bronchospasm d. Uterine contractions

a. Palpitations

3. When benztropine (Cogentin) is ordered for a client, the nurse acknowledges that this drug is an effective treatment for which condition? a. Parkinsonism b. Paralytic ileus c. Motion sickness d. Urinary retention

a. Parkinsonism

7. A client is to receive conscious sedation for a minor surgical procedure. Which drug administration should the nurse expect? (Select all that apply.) a. Propofol (Diprivan) to sustain natural sleep b. Lidocaine (Xylocaine) to provide local anesthesia c. Midazolam (Versed) to promote sedation and following of commands d. Ketamine (Ketalar) for rapid inductionand prolonged duration of action

a. Propofol (Diprivan) to sustain natural sleep c. Midazolam (Versed) to promote sedation and following of commands

An 80-year old client complains of recent onset of insomnia saying, "if only I could get to sleep!" If a drug is prescribed, which drug characteristics would be best for this situation? (select all that apply) a. Short-intermediate acting b. Rapidly eliminated c. Slowly eliminated d. Multiple metabolites

a. Short-intermediate acting b. Rapidly eliminated

The client has nine medications prescribed to take daily. Which are common reasons for nonadherence to the drug regimen in the older adult? (select all that apply) a. Taking multiple drugs at one time b. Impaired memory c. Decreased dexterity d. Increased mobility

a. Taking multiple drugs at one time b. Impaired memory c. Decreased dexterity

2. The client's serum digoxin level is 3.0 ng/mL. What does the nurse know about this serum digoxin level?

a.It is in the high (elevated) range

8. A client is diagnosed with peripheral arterial disease (PAD). He is prescribed isoxsuprine (Vasodilan). The nurse acknowledges that isoxsuprine does what? (Select all that apply.)

a.Relaxes the arterial walls within the skeletal muscles b.May cause hypotension, chest pain, and palpitations

2. A client who received heparin begins to bleed, and the physician calls for the antidote. The nurse knows that which is the antidote for heparin?

a.protamine sulfate

When teaching a patient about beta-blockers such as atenolol (Tenormin) and metoprolol (Lopressor), it is important to inform the patient that

abrupt medication withdrawal may lead to a rebound hypertensive crisis

silvadene

an antibiotic used topically in burn treatment

prilosec

antacid that suppresses acid secretion in the stomach best given on empty stomach

-dazole

anthelmintic; antibiotic; antibacterial

-zolamide

anti-glaucoma, decreases production of aqueous humor adfx: blurred vision, lethargy, depression, anorexia, decrease potassium

alkylating agents

anti-neoplastic- interferes with rapid DNA replication used: leukemia, multiple myeloma Csplatin, busulfan (Mylerin), cyclophosphamide (Cytoxin) adfx: very hepatotoxic, basic chemo adfx: bone marrow suppression, decreased WBC, platelets, RBCs, bleeding, GI s/s, alopecia, lost nails, infertility check hematopoesis, CBC

tetracycline

antibacterial (wide range)

-cycline

antibiotic

-floxacin

antibiotic

-mycin

antibiotic; antibacterial

-arin

anticoagulant (decreases clotting)

-arin

anticoagulant adfx: bleeding, hematuria heparin blocks conversion of prothrombin to thrombin, used in PE, venous thrombosis, after acute MI. PTT: 30-75? antidote for heparin: protamine sulfate warfarin interferes with vit k clotting factors. used in PE, vt, prophylaxis after acute MI. adtl adfx: alopecia. takes 5 days for therapy to work PT: 14-35 INR: 2-3 vit c decreases effectiveness antidote for warfarin: vit k. avoid vit k while taking it. AVOID GARLIC, GINKO, GINGER w/ anticoags

-lol

antihypertensives beta adrenergic blockers- decreases excitability of heart, decreases oxygen consumption use: angina, SVT, HTN adfx: bronchospasm, HR changes take with food, taper off to avoid rebound HTN

-statin

antilipidemic (lowers LDL's) tx: high LDL's

-triptan

antimigraine; selective 5-HT receptor agonist

-tinib

antineoplastic (kinase inhibitor)

-bicin

antineoplastic; cytotoxic agent

-dopa

antiparkinson. essentially replace dopamine (also parlodil) adfx: ataxia, anticholinergic, dizziness B6 can reverse effects, avoid CNS depressants

-vir

antiviral, antiinfective * acyclovir

-cyclovir

antiviral, inhibits DNA and RNA (includes non -cyclovirs too) use in herpes, HIV adfx: HA, dizziness, GI doesn't cure.

vir; -vir

antiviral; (anti-HIV, anti-hepatitis, anti-herpes, anti-cytomegalovirus (CMV), anti-flu)

The client's magnesium level is 2.7 mEq/L. Specific health teaching by the nurse for this client should include which suggestion? a. Eat fruits, fish, and peanut butter b. Avoid selected laxatives and antacids c. Avoid magnesium, which is irritating to the stomach d. Measure weight daily

b. Avoid selected laxatives and antacids

A male nurse has been assigned to care for a young married woman who practices Islam. It is important that the nurse perform which action? a. Not touch any part of the client's body b. Delegate nursing care that involves touching to a female member of the nursing team whenever appropriate c. Touch the client only when her spouse is present d. Communicate to the nurse manager that he cannot take care of female clients who practice Islam

b. Delegate nursing care that involves touching to a female member of the nursing team whenever appropriate

7. A client is beginning to take carisoprodol (Soma). Which interventions should the nurse include in the care of this client? (Select all that apply.) a. Ask the client if there is any history of narrow-angle glaucoma. b. Inform the client that muscular pain is usually relieved within 1 week. c. Tell the client to report dizziness and double vision to the health care provider. d. Advise the client to avoid alcohol and other CNS depressants. e. Instruct the client that this drug should not be stopped abruptly.

b. Inform the client that muscular pain is usually relieved within 1 week. c. Tell the client to report dizziness and double vision to the health care provider. d. Advise the client to avoid alcohol and other CNS depressants. e. Instruct the client that this drug should not be stopped abruptly.

The nurse is caring for a neonate with lower-than-normal albumin levels. The nurse is ordered to administer a medication that is highly bound protein. The nurse knows that the dose needs to be altered in which way to respond to these factors? a. The dose will be increased b. The dose will be decreased c. Highly protein-bound drugs will be contraindicated d. The nurse must further clarify the medication order before administration

b. The dose will be decrease

6. For the client taking a diuretic, a combination such as triamterene and hydrochlorothiazide may be prescribed. The nurse realizes that this combination is ordered for which purpose?

b. To increase the serum potassium level

Chapter 19 1. A client is receiving bethanechol (Urecholine). The nurse realizes that the action of this drug is to treat: a. Glaucoma b. Urinary retention c. Delayed gastric emptying d. Gastroesophageal reflux disease

b. Urinary retention

5. The nurse realizes that cholinergic agonists mimic which parasympathetic neurotransmitter? a. dopamine b. acetylcholine c. cholinesterase d. monoamine oxidase

b. acetylcholine

The nurse is aware that according to The Joint Commission, which abbreviations are not on the do-not-use list for ordering or documenting medications? (select all that apply) a. QD b. h.s c. T.I.W d. b.i.d

b. h.s c. T.I.W d. b.i.d

For the client taking epinephrine, the nurse realizes there is a possible drug interaction with which drug? a. albuterol (Proventil) b. metoprolol (Lopressor) c. bethanechol (Urecholine) d. tolterodine tartrate (Detrol)

b. metoprolol (Lopressor)

4. A client is admitted to the emergency department in respiratory depression following self-injection with hydromorphone. The admitting nurse knows that which drug will reverse respiratory depression caused by opioid overdose? a. fentanyl b. naloxone c. butorphanol d. sufenta

b. naloxone

5. When a client first takes a nitrate, the nurse expects which symptom that often occurs?

b.Headaches

7. The client has been receiving spironolactone (Aldactone) 50 mg/day for heart failure. The nurse should closely monitor the client for which condition?

b.Hyperkalemia

3. A client's high-density lipoprotein (HDL) is 60 mg/dL. What does the nurse acknowledge concerning this level?

b.It is the desired level of HDL.

6. The client is taking rosuvastatin (Crestor). What severe skeletal muscle adverse reaction should the nurse observe for?

b.Rhabdomyolysis

9. A client is to undergo a coronary angioplasty. The nurse acknowledges that which drug is used primarily for preventing reocclusion of coronary arteries following a coronary angioplasty?

b.abciximab (ReoPro

4. The nurse knows that which diuretic is most frequently combined with an antihypertensive drug?

b.hydrochlorothiazide

6. A client is being changed from an injectable anticoagulant to an oral anticoagulant. Which anticoagulant does the nurse realize is administered orally?

b.warfarin (Coumadin)

how often should TPN tube and bag be replaced

bag and tube every 24 hours

The patient has a serum digoxin level drawn and it comes back 0.4 ng/mL. The nurse recognized that this is:

below the therapeutic level

-zepam; zolam

benzodiazepene, anti-anxiety, sedative-hypnotic * diazepam, lorazepam, alprazolam, midazolam

-pam, -lam

benzodiazepine

-zepam

benzodiazepine

-zolam

benzodiazepine

-pam

benzodiazepine adfx: depression, CNS depression, addictive. d/c slowly caffeine and nicotine decreases effectiveness

-olol; -lol

beta adrenergic blocker, antihypertensive, and/or antianginal * atenolol, metoprolol, propranolol, timolol

-terol

beta agonist; bronchodilator

-olol

beta blocker

-olol

beta blocker (decreases heart rate & cardiac output ==> decreases b/p & slows beats & force of heart. tx: Hypertension, Heart arrhythmias, Angina, Prevent reoccuring heart attack

Cholestyramine (Questran) decreases lipid levels by

binding to bile in the intestinal tract, forming an insoluble complex that is excreted in the feces.

-dronate

biophosphenates- antiosteoporosis inhibit bone resorption adfx: esophagitis, arthralgia

adequate folic acid will helop to prevent

birth defects

-dronate

bisphosphonate, bone resorption inhibitor (helps with bone loss) * alendronate, risendronate

-dronate

bisphosphonate; bone resorption inhibitor

anticholinergic

block parasymp nerve dilate pupil, bronchodilation, decrease secretions, decrease GI motility and secretions uses: eye exams, motion sickness adfx:urinary retention, dry mouth, hypotn and inc HR probanthine, atropine, -bromide, ium not with meals or paralytic ileus

Antihistamines

block the release of histamine in allergic reactions

Before administering isosorbide mononitrate (Imdur), a priority assessment would include

blood pressure

Nitrates relieve angina pain by reducing preload, which is

blood volume within the heart

Ocular Impairment Assessment

blurred vision, color vision changes, blindness

When caring for a patient with angina, the nurse would question an order for a noncardioselective beta-blocker in a patient with coexisting

bronchial asthma

-tropium, -ol, -phylline

bronchodilators uses:COPD, asthma (terbutaline for preterm labor) adfx: tachycardia, dysrhythmias, palpitations, anticholinergic take bronchodilator first

3. When administering phenytoin (Dilantin), the nurse realizes more teaching is needed if the client makes which statement? a. "I must shake the oral suspension very well before pouring in the dose cup." b. "I cannot drink alcoholic beverages when taking phenytoin." c. "I should take phenytoin 1 hour before meals." d. "I will need to get periodic dental checkups."

c. "I should take phenytoin 1 hour before meals."

It is important for the nurse to be aware of the four sequential processes of the pharmacokinetic phase. What are these processes? a. Distribution, metabolism, excretion, absorption b. Biotransformation, excretion, absorption, metabolism c. Absorption, distribution, metabolism, excretion d. Metabolism, distribution, absorption, excretion

c. Absorption, distribution, metabolism, excretion

4. The nurse is teaching a client about clopidogrel (Plavix). What is important information to include?

c. Bleeding may increase when taken with aspirin.

The nurse reviews the client's plan of care, which includes strategies to prevent which common complication of enteral feedings? a. Aspiration b. Constipation c. Diarrhea d. Muscle weakness

c. Diarrhea

5.The nurse realizes that beta1 receptor stimulation is differentiated from beta2 stimulation in that stimulation of beta1 receptors leads to which condition? a. Increased bronchodilation b. Decreased uterine contractility c. Increased myocardial contractility d. Decreased blood flow to skeletal muscles

c. Increased myocardial contractility

A parent is learning to administer medication to a school-aged child. Which strategy should the nurse teach the parent to achieve cooperation in a child of this age? a. Enlisting physical restraint b. Tolerating violent reactions c. Providing age-appropriate explanations d. Establishing medication contracts

c. Providing age-appropriate explanations

3. For the client receiving periodic morphine IV push, which is most critical for the nurse to monitor? a. Fever b. Diarrhea c. Respirations d. White blood cell count

c. Respirations

A nurse is teaching a 16-year old female client about a newly prescribed medication. The client is bilingual in Spanish and English. Which behavior best indicated the client's understanding of the instructions? a. The client frequently nods her head while listening to the nurse's instructions b. The client stated that she understands the instructions c. The client repeats the nurse's instruction to her mother who is present during the teaching d. The client does not ask the nurse for any clarification of the instructions

c. The client repeats the nurse's instruction to her mother who is present during the teaching

The nurse is developing goals in collaboration with the client. Which is the best goal statement? a. The client will self-administer albuterol by tomorrow b. The client will self-administer the prescribed dose of albuterol by the end of the second teaching session c. The client will independently self-administer the prescribed dose of albuterol by the end of the second teaching session d. The client will organize her medications by tomorrow

c. The client will independently self-administer the prescribed dose of albuterol by the end of the second teaching session

The nurse understands the differences between drug excretion in children and that in adults. With this knowledge, the nurse makes the which decision in administering medication to children? a. Because most children need a higher dose of medications, the nurse will contact the physician for an increase in the ordered dose b. Because children excrete drugs rapidly, the nurse will need to assess carefully for therapeutic effects of the medication c. The most important assessment is to evaluate for drug accumulation, because excretion of drugs in children is slower d. The excretion of most drugs is the same in children as in adults, but assessments are important to avoid side effects

c. The most important assessment is to evaluate for drug accumulation, because excretion of drugs in children is slower

3. An antipsychotic agent, fluphenazine (Prolixin), is ordered for a client with psychosis. The nurse knows that this agent can lead to extrapyramidal symptoms (EPS) that may be treated with which medication? a. quetiapine (Seroquel) b. aripiprazole (Abilify) c. benztropine (Cogentin) d. chlorpromazine (Thorazine)

c. benztropine (Cogentin)

Chapter 26 1. The nurse knows that which medication will cause the least gastrointestinal distress? a. aspirin b. ketorolac c. celecoxib d. ibuprofen

c. celecoxib

7. A newborn client is in respiratory distress. The nurse anticipates preparation for which medication to be given? a. modafinil b. armodafinil c. theophylline d. amphetamine

c. theophylline

1. A client's blood pressure (BP) is 145/90. According to the guidelines for determining hypertension, the nurse realizes that the client's BP is at which stage?

c.Stage 1 hypertension

8. The beta blocker acebutolol (Sectral) is prescribed for dysrhythmias. The nurse knows that what is the primary purpose of the drug?

c.To block the beta1-adrenergic receptors in the cardiac tissues

-dipine

calcium channel blocker, antianginal, antihypertensive * amlodipine, nicardipine, nifedipine

monoclonal antibodies

cancer treatment

ANTIHYPERTENSIVES ACE inhibitors

captopril, enalapril ACE inhibitors = ACE = Arteries decrease BP by affecting the arteries in the body (primary function) A for ACE, pril for ACE inhibitors = A-pril side effects: GI upset, orthostatic hypotension, dizziness action: blocks Angiotensin I to Angiotensin II (vasoconstriction) in lungs, resulting in VASODILATION =systemic decrease in BP uses: HTN, CHF give 1 hr ac to 3 hr pc food change position slowly monitor VS makes pt feel lethargic, may go away

-lamide

carbonic anhydrase inhibitor

-oxin

cardiac glycoside-increase force of myocardial contraction, slow HR Left heart failure adfx: bradycardias, N/V, visual disturbance hold if pulse <60 in adult, <70 in older kids, <90 in infants, young kids check K+ levels- low potassium increases risk of toxicity. eat plenty of potassium loading doses--- 0.5, then 0.25, then 0.125 for daily dose antidote: digibind

CEPHALASPORINS (antibiotic)

cephalexin (Keflex) cefaclor, ceftriaxone, cefedipime, notice: cefs or cephs uses: pharingitis, tonsillitis, otitis, upper lower respiratory, gonorrhea, meningitis, perioperative prophylaxis, UTI side effects: bone marrow depression rash abdominal, pain, NV, diarrhea increased risk of bleeding nursing considerations: TAKE WITH FOOD cause GI upset !!!!!!!!!!!!!!!!!!!!!!!! CROSS ALLERGY with PENICILLIN !!!!!! Avoid Alcohol may cause flushing, muscle cramps Obtain culture and sensitivity may give false positives of: glucose in urine protein uria

ceph (cef)

cephalosporin-tonsillitis, prophylaxis, meningitis-bacterial wall inhibitor adfx: bone marrow depression, GI upset, cross allergy w/pcn, protein/glucose in urine do not drink etoh, take with food

angina pectoris

chest pain, which may radiate to the left arm and jaw, that occurs when there is an insufficient supply of blood to the heart muscle

Antihistamine

chlor-trimeton, diphenhydramine (Benadryl), promethazine (Phenergan) adfx: drowsiness, anticholinergic, photosensitivity, bronchospasm hard candy for dry mouth

Which of the following medications has antiplatelet properties?

clopidogrel

-fate

cytoprotective agents form barrier on duodenal ulcers adfx: constipation, vertigo, flatulance take without food

The client is taking duastride (Avodart). Which client comment indicates the need for more information about the drug? a. "I'm glad I can take the medication with or without food" b. "it is good that no lab tests and monitoring are required" c. "this drug is expensive" d. "I prefer to chew the drug before swallowing it"

d. "I prefer to chew the drug before swallowing it"

7. A client is taking valproic acid (Depakote). The nurse should monitor the client for a which therapeutic serum range? a. 10 to 20 mcg/mL b. 15 to 40 mcg/mL c. 20 to 80 ng/mL d. 40 to 100 mcg/mL

d. 40 to 100 mcg/mL

2. Several children are admitted for diagnosis with possible attention deficit/hyperactivity disorder. Which is most important for the nurse to observe? a. A girl who is lethargic b. A girl who lacks impulsivity c. A boy with smooth coordination d. A boy with an inability to complete tasks

d. A boy with an inability to complete tasks

ANTINEOPLASTIC AGENTS (chemo drugs)

decreased red blood cells - impaired O2 transport-air hunger decreased WBC - infection stomatitis encourage fluids 3000ml/day therapeutic communication very important

beta-antagonist

decreases contraction of smooth muscle

the drug of choice treating pancreatitis

demeral

Renal Impairment Nursing Considerations

diet/fluid restrictions electrolyte replacement dialysis

Bronchodilators

dilate large air passages and are commonly presecribed for clients with asthma and chronic obstructive lung disease

Auditory Impairment Assessment

dizziness ringing in ears loss of balance, hearing

-floaxcin

fluoroquinalones- dna inhibitor anthrax tx, e. coli adfx: decreased WBC, hct. elevated liver enzymes don't take with food

-floxacin

fluoroquinolone, antibiotics * ciprofloxacin, levofloxacin

Dermatological Reactions Nursing Considerations

frequent skin care avoid rubbing, tight clothing, harsh soaps, perfumed lotions oatmeal baths, gentle skin care

antifungals

fungizone (Amphotericin), mycostatin (Nystatin) impair cell membrane adfx: liver dysfunction, thrombocytopenia, leukopenia, pruritis take with food

uti meds

furadantin-anti-infective adfx: asthma attack, take with food and cranberry juice pyridian-urinary tract analgesic adfx: HA, vertigo, turns pee orange, take with meals anti-impotence drugs adfx: HA, hypotn, priaprism

The nurse prepares to administer which diuretic to treat a patient suffering from acute pulmonary edema?

furosemide (Lasix)

Misoprostyl (Cytotec) & Sucralfate (Carafate) are

gastric protectants

A patient with elevated triglyceride levels unresponsive to HMG-CoA reductase inhibitors will most likely be prescribed which drug?

gemfibrozil (Lopid)

tetracyclines (antibiotics)

general infections, prohylaxis such as baby eyes upon birth Uses: infection, acne ... Doxycycline Minocycline tatracycline UNIQUE TO THIS MED discoloration of teeth if taken during pregnancy, glossitis, rash, phototoxic reaction (blisters) AVOID SUNLIGHT/ sunblock NOTE EXPIRATION DATE make pt sick of beyond date MONITOR RENAL FUNCTION DO NOT TAKE WITH ANTACIDS OR MILK TOPICAL APPLICATION MAY STAIN CLOTHS

When giving adelnosine, it is important to remember to

give it as a fast intravenous push

Milk of magnesium

given for constipation side effects: diarrhea nursing considerations: fluid volume deficit - replace fluids

Tegretol and Depakote

given over the entire bipolar cycle dosages do not change too much KIDNEY FUNCTION and LIVER IMPORTANT BUN, ALT and AST

why do we give a client who is being treated for CHFa loading dose of digoxin

gives the patient an adequate blood level to achieve therapeutic relief as quick as possible

-semide

loop diuretic (water pill)

what is a hallmark sign of air embolism of TPN

low BP and churning heart murmur

-thromycin

macrolide-cell membrane binder used in pcn allergy. used for dental prophylaxis in valve disorders adfx: confusion, increases effects of coumadin, theophylline take without food

boniva

maintain adequate intake of calcium and vit. d

what are the outcomes for TPN goals

maintain fluid, electrolyte, and weight

-mab

monoclonal antibody

Tegretol:

mood stabilizer - bipolar / anticonvulsant - carbamezepine

report an increased weight of _- per week due to possible fluid overload or retention

more than 3 pounds per week

The nurse would question an order for cevimeline (Evoxac) in a patient with a history of

narrow-angle glaucoma

nitr-; -nitr-

nitrate, vasodilator, antianginal * nitroglycerin, isosorbide dinitrate or mononitrate

ANTIGUNGALS

nyastatin, FLAGYL (Metranidazole) , Diflucan candidasis, oral thrish THINK LIVER!!!! Hepatoxicity CAN CAUSE THROMBOCYTOPENIA Bleeding risk LEUKOPENIA pruritus (body itching) MUST GIVE WITH FOODS (GI upset) OR MILK AFTS LATS ORAL HYGIENE ASSESS, assess and reassess

Antacids are used for

peptic ulcer disease & GERD

The nurse assesses the intravenous infusion site of a patient receiving dopamine and finds it is infiltrated. The nurse will prepare which of the following medications to treat this infiltration?

phentolamine (Regitine)

Ondansetron is an antiemetic used for

post op N/V as well as N/V induced w/ chemotherapy

ANTIHYPERTENSIVES angiotensin II receptor antagonist (ACE blocker/ARBS)

powerful vasoconstrictor (effecting angiotensin II) Candesartan (atacand), losartan (cozaar) identify by: sartan's uses: HTN, HF, MI, Diabetic neuropathy, stroke prevention side effects: hypertension, dizziness, GI distress monitor VS (BP and pulses) do not chew or divide sustained release tabs

what is the best way to find out if TPN is effective

prealbumin levels

Anticholinergics are often administered

preoperatively

Nitrates relieve angina pain by reducing afterload, which is

pressure against which the heart must pump

Anticoagulants

prevent clot formation by decreasing vitamin K levels and blocking the clotting chain or by preventing platelet aggregation

Auditory Impairment Nursing Considerations

prevent falls monitor hearing ability

ACE inhibitors and angiotensin receptor blockers both work to decrease blood pressure by

preventing aldosterone secretion

TPN prevents catabolism of this electrolyte from this tissue

prevents catabolism of protien from muscle or fat

Angiotensin receptor blocker

primary and secondary HTN

Angiotensin-converting agents

primary and secondary HTN

-floxacin

quinolone antibiotic

Delayed Allergic Reaction Assessment

rash, hives swollen joints

hemoglobin laboratory test measures?

rbc's and other things

Antipyretics

reduce fever

Anticholinergic Effect nursing considerations

sugarless lozenges good mouth care void before taking medication safety measure for vision changes

how is TPN to be discontinued

taper down to prevent hypoglycemia

-cycline

tetracycline antibiotic

-cycline

tetracycline, antibiotic * doxycycline, tetracycline

absorption

the time it take the drug to be taken into the body to the time it enters the blood stream

ANTIHYPERTENSIVES Direct acting vasodilators

works in vascular bed, relaxes smooth muscle causing vasodilatation causing lower BP Apresoline (hydralazine), Menoxidil (also a treatment for alopecia) Rogan(?) side effect: tachy cardia (decreases BP in peripheral bed, increases HR, often seen together with beta blockers) increase in body hair MUST TEACH: check pulse rate when meds taken, if less then 60 MUST be held, then call primary to see what to do monitor BP hypotension, change position slowly, may cause lethargy and tiredness, goes away with time

beginning with: sulfa

Sulfonamide. Antibiotic. Contraindicated in pregnancy/lactation, age <2 months, porphyria, renal/hepatic dysfunction, intestinal/urinary blockage, asthma, blood dyscrasias, G6PD deficiency. Side effects include rash, jaundice, stomatits, insomnia, drowsiness, depression, psychosis, photosensitivity, crystalluria, peripheral neuropathy, tinnitus/hearing loss, vertigo, ataxia, convulsions, heptatitis, pancreatitis, exfoliative dermatitis, Stevens-Johnson syndrome, serum sickness, drug fever. Nursing considerations: Baseline assessments of renal/hepatic function and skin. Monitor for neurotoxicity if ataxia or convulsions develop. Small frequent meals. Empty bladder frequently (q2h when awake). Interact with oral anticoagulants, oral antidiabetic agents, iron, some abx, & phenytoin.

ending in: cycline

Tetracycline. Antibiotic. Contraindicated in renal/hepatic dysfunction, last 1/2 of pregnancy, 0-8 years old, asthma, hay fever, and myasthenia gravis. Side effects include dry mouth, dysphagia, bulky/loose stools, steatorrhea, photosensitivity, increased ICP, rash, urticaria, exfoliative dermatitis, angioedema, discoloration of developing teeth, conjuctive pigmentation, lossening of nails, increased liver labs, decreased cholesterol, jaundice, anaphylaxis, serum sickness, drug fever, retrosternal pain, increased sodium retention, hepatotoxic, renal toxic, metabolic acidosis, itching, and wheezing. Nursing considerations: Unstable with age & light exposure, store in tightly covered container in a dry area protected from light & heat. Practice good oral care/hygeine. Report onset of severe h/a or visual disturbances. Take oral doses with full glass of water on an empty stomach to increase absorption. Interact with antacids, antidiarrheals, anticoagulants, oral contraceptives, penicillins, , digoxin, dairy & iron supplements.

c. To block the beta1-adrenergic receptors in the cardiac tissues

The beta blocker acebutolol (Sectral) is prescribed for dysrhythmias. The nurse knows that what is the primary purpose of the drug? a. To increase the beta1 and beta2 receptors in the cardiac tissues b. To increase the flow of oxygen to the cardiac tissues c. To block the beta1-adrenergic receptors in the cardiac tissues d. To block the beta2-adrenergic receptors in the cardiac tissues

b. "It's best to keep it in its original container away from heat and light."

The client asks the nurse how nitroglycerin should be stored while traveling. What is the nurse's best response? a. "You can protect it from heat by placing the bottle in an ice chest." b. "It's best to keep it in its original container away from heat and light." c. "You can put a few tablets in a resealable bag and carry it in your pocket." d. "It's best to lock them in the glove compartment to keep them away from heat and light."

a. Administer an additional dose of warfarin (Coumadin).

The client has an international normalized ratio (INR) value of 1.5. What action will the nurse take? a. Administer an additional dose of warfarin (Coumadin). b. Hold the next dose of warfarin (Coumadin). c. Increase the heparin drip rate. d. Administer protamine sulfate.

b. Hyperkalemia

The client has been receiving spironolactone (Aldactone) 50 mg/day for heart failure. The nurse should closely monitor the client for which condition? a. Hypokalemia b. Hyperkalemia c. Hypoglycemia d. Hypermagnesemia

a. Increase the serum digoxin sensitivity level

The client is also taking a diuretic that decreases her potassium level. The nurse expects that a low potassium level (hypokalemia) could have what effect on the digoxin? a. Increase the serum digoxin sensitivity level b. Decrease the serum digoxin sensitivity level c. Not have any effect on the serum digoxin sensitivity level d. Cause a low average serum digoxin sensitivity level

b. Rhabdomyolysis

The client is taking rosuvastatin (Crestor). What severe skeletal muscle adverse reaction should the nurse observe for? a. Myasthenia gravis b. Rhabdomyolysis c. Dyskinesia d. Agranulocytosis

b. The beta blocker should NOT be abruptly stopped; the dose should be tapered down.

The health care provider is planning to discontinue a client's beta blocker. What instruction should the nurse give the client regarding the beta blocker? a. The beta blocker should be abruptly stopped when another cardiac drug is prescribed. b. The beta blocker should NOT be abruptly stopped; the dose should be tapered down. c. The beta blocker dose should be maintained while taking another antianginal drug. d. Half the beta blocker dose should be taken for the next several weeks.

a. Diuretic

The nurse acknowledges that the first-line drug for treating this client's blood pressure might be which drug? a. Diuretic b. Alpha blocker c. ACE inhibitor d. Alpha/beta blocker

a. Hypokalemia

The nurse acknowledges that which condition could occur when taking furosemide? a. Hypokalemia b. Hyperkalemia c. Hypoglycemia d. Hypermagnesemia

d. Get up slowly from a sitting to a standing position.

The nurse is caring for a client with hypertension who is prescribed Clonidine transdermal preparation. What is the correct information to teach this client? a. Change the patch daily at the same time. b. Remove the patch before taking a shower or bath. c. Do not take other antihypertensive medications while on this patch. d. Get up slowly from a sitting to a standing position.

b. "Take this medication at the same time each day."

The nurse is reviewing instructions for a client taking an HMG-CoA reductase inhibitor (statin). What information is essential for the nurse to include? a. "Take this medication on an empty stomach." b. "Take this medication at the same time each day." c. "Take this medication with breakfast." d. "Take this medication with an antacid."

c. Bleeding may increase when taken with aspirin.

The nurse is teaching a client about clopidogrel (Plavix). What is important information to include? a. Constipation may occur. b. Hypotension may occur. c. Bleeding may increase when taken with aspirin. d. Normal dose is 25 mg tablet per day.

4. aPT one and a half to two and a half times the control value. Rationale: aPT is the coagulation study that monitors oral anticoagulant use, such as warfarin. A result of one and a half to two and a half times the control value indicates adequate anticoagulation. aPTT is the coagulation study that monitors heparin use. An aPT level of one would indicate a less-than-therapeutic level of anticoagulation.

The nurse receives the patient's lab values throughout warfarin drug therapy. The expected therapeutic level is: 1 aPTT of three to four times the normal control value. 2 aPTT one to two times the patient's baseline level. 3 aPT one to two times the patient's last result. 4 aPT one and a half to two and a half times the control value.

c. Client is on oral contraceptives.

The nurse reviews the history for a client taking atorvastatin (Lipitor). What will the nurse act on immediately? a. Client takes medications with grape juice. b. Client takes herbal therapy including kava kava. c. Client is on oral contraceptives. d. Client was started on penicillin for a respiratory infection.

a. Impaction

The nurse would question an order for cholestyramine (Questran) if the client has which condition? a. Impaction b. Glaucoma c. Hepatic disease d. Renal disease

4. Thrombin, fibrin Rationale: Prothrombinase converts prothrombin to thrombin. Thrombin then converts fibrinogen to long strands of fibrin, which provide a framework for the clot. Thrombin and fibrin are formed only after the injury occurs. Fibrin strands form an insoluble web over the injured area to stop blood loss.

The nurse's understanding of the clotting mechanism is important in administering anticoagulant drugs. The nurse understands that which of the following clotting factors are formed after injury to the vessels? 1.Fibrin, vitamin K 2.Thromboplastin, fibrinogen 3.Prothrombin, thrombin 4.Thrombin, fibrin

distribution

the transport of drugs into the body

lipids aka __ may be added to TPN 1-3 x a week

fat emulsion

why ask pt. if they are allergic to eggs if receiving a lipid solution of TPN

fat emulsion may have egg yolk phospholipids

Clonazepam

Anticonvulsant Medication

Depakote

Anticonvulsant Medication

Dilantin

Anticonvulsant Medication

Keppra

Anticonvulsant Medication

Lamictal

Anticonvulsant Medication

Luminal

Anticonvulsant Medication

Magnesium Sulfate

Anticonvulsant Medication

Mysoline

Anticonvulsant Medication

Neurontin

Anticonvulsant Medication

Tegretol

Anticonvulsant Medication

Topamax

Anticonvulsant Medication

Valium

Anticonvulsant Medication

Zarontin

Anticonvulsant Medication

Tofranil

Antidepressant

Zalcitabine (HIVID)

Antiviral

Zanamivir (Relenza)

Antiviral

Which statements by the patient demonstrates a need for further education regarding nitroglycerin?

"I can take up to three tablets at 5-min. intervals for chest pain."

Zidovudine (AZT)

Antiviral

Which laborator value might indicate an adverse response to hydrochlorothiazide (HydroDiuril)?

Glucose levels

173.) A nurse reviews the medication history of a client admitted to the hospital and notes that the client is taking leflunomide (Arava). During data collection, the nurse asks which question to determine medication effectiveness? 1. "Do you have any joint pain?" 2. "Are you having any diarrhea?" 3. "Do you have frequent headaches?" 4. "Are you experiencing heartburn?"

1. "Do you have any joint pain?" Rationale: Leflunomide is an immunosuppressive agent and has an anti-inflammatory action. The medication provides symptomatic relief of rheumatoid arthritis. Diarrhea can occur as a side effect of the medication. The other options are unrelated to medication effectiveness.

691. Rifabutin (Mycobutin) is prescribed for a client with active Mycobacterium avium complex (MAC) disease and tuberculosis. For which of the following side effects of the medication should the nurse monitor? Select all that apply. 1. Signs of hepatitis 2. Flu-like syndrome 3. Low neutrophil count 4. Vitamin B6 deficiency 5. Ocular pain or blurred vision 6. Tingling and numbness of the fingers

1. Signs of hepatitis 2. Flu-like syndrome 3. Low neutrophil count 5. Ocular pain or blurred vision

-azosin

antihypertensive alpha 1-adrenergic blockers in peripheral vasculature use: HTN, BPH adfx: orthostatic hypotn, reflex tachycardia, Nasal Congestion, periph edema first dose at HS-causes fainting initially

14.) The client with acute myelocytic leukemia is being treated with busulfan (Myleran). Which laboratory value would the nurse specifically monitor during treatment with this medication? 1. Clotting time 2. Uric acid level 3. Potassium level 4. Blood glucose level

2. Uric acid level Rationale: Busulfan (Myleran) can cause an increase in the uric acid level. Hyperuricemia can produce uric acid nephropathy, renal stones, and acute renal failure. Options 1, 3, and 4 are not specifically related to this medication.

128.) A nurse is providing instructions to an adolescent who has a history of seizures and is taking an anticonvulsant medication. Which of the following statements indicates that the client understands the instructions? 1. "I will never be able to drive a car." 2. "My anticonvulsant medication will clear up my skin." 3. "I can't drink alcohol while I am taking my medication." 4. "If I forget my morning medication, I can take two pills at bedtime."

3. "I can't drink alcohol while I am taking my medication." Rationale: Alcohol will lower the seizure threshold and should be avoided. Adolescents can obtain a driver's license in most states when they have been seizure free for 1 year. Anticonvulsants cause acne and oily skin; therefore a dermatologist may need to be consulted. If an anticonvulsant medication is missed, the health care provider should be notified.

The client receives TPN at home. The visiting nurse assists the family with the care plan, which includes changing the TPN solution and tubing how often? a. Every 24 hours b. Every 36 hours c. Every 48 hours d. Every 72 hours

a. Every 24 hours

137.) A nurse is reinforcing instructions for a client regarding intranasal desmopressin acetate (DDAVP). The nurse tells the client that which of the following is a side effect of the medication? 1. Headache 2. Vulval pain 3. Runny nose 4. Flushed skin

3. Runny nose Rationale: Desmopressin administered by the intranasal route can cause a runny or stuffy nose. Headache, vulval pain, and flushed skin are side effects if the medication is administered by the intravenous (IV) route.

centrally acting alpha-adrenergic

antihypertensive centrally acting alpha-adrenergic-decrease rate and force of contraction Catapres, Aldomet uses: HTN adfx: orthostatic hypotn, Sedation taper off

b. Teach the client of potential drug interactions with anticoagulants.

A client who is taking warfarin (Coumadin) requests an aspirin for headache relief. What is the nurse's best response? a. Administer 650 mg of acetylsalicylic acid (ASA) and reassess pain in 30 minutes. b. Teach the client of potential drug interactions with anticoagulants. c. Explain to the client that ASA is contraindicated and administer ibuprofen as ordered. d. Explain that the headache is an expected side effect and will subside shortly.

a. protamine sulfate

A client who received heparin begins to bleed, and the physician calls for the antidote. The nurse knows that which is the antidote for heparin? a. protamine sulfate b. vitamin K c. aminocaproic acid d. vitamin C

-pril

antihypertensives ACE Inhibitors-affect arteries-vasodilation in whole body use for HTN and CHF adfx: lethargy initially, orthostatic hypotension take without food

2. To reduce the risk of a stroke from a blood clot Rationale: Antiplatelet drugs such as clopidogrel are given to inhibit platelet aggregation and thus reduce the risk of thrombus formation. Antiplatelet drugs do not exert antiinflammatory, antipyretic, or analgesic effects. The antiplatelet and anticoagulant drugs do not prevent emboli formation. Thrombolytics dissolve existing blood clots.

A patient has started clopidogrel (Plavix) after experiencing a TIA (transient ischemic attack). The desired therapeutic effects of this drug will be: 1 Anti-inflammatory and antipyretic effects 2 To reduce the risk of a stroke from a blood clot 3 Analgesic as well as clot-dissolving effects 4 To stop clots from becoming emboli

The beta blocker can mask symptoms of hypoglycemia. **Beta blockers decrease the body's adrenergic "fight-or-flight" response and may block the symptoms and signals of hypoglycemia that a diabetic normally perceives as the blood sugar drops. Beta blockers may inhibit glycogenolysis, resulting in hypoglycemia and have no effect on the development of insulin resistance.

A type I diabetic on insulin reports that he takes propranolol (Inderal) for his hypertension. This raises a concern and the nurse will teach the patient to check glucose levels more frequently because: 1. The beta blocker can produce insulin resistance. 2. The two agents used together will increase the risk of ketoacidosis. 3. Propranolol will increase insulin requirements by antagonizing the effects at the receptors. 4. The beta blocker can mask symptoms of hypoglycemia.

4. A client is taking clonazepam (Klonopin) for absence (petit mal) seizures. Which value(s) should the nurse report as outside the therapeutic range for clonazepam? (Select all that apply.) a. 5 mcg/mL b. 15 mcg/mL c. 60 ng/mL d. 120 ng/mL

a. 5 mcg/mL b. 15 mcg/mL d. 120 ng/mL

ending in: micin or mycin

Aminoglycoside. Antibiotic. Contraindicated in renal disease, myasthenia gravis, and concurrent use with renal toxic agents. Side effects include paresthesia, rash, fever, nephrotoxicity, ototoxicity, purpura, urticaria, dermatitis, pseudomembranous colitis. Nursing considerations: Monitor peak/trough levels of drug. Eat small frequent meals. Interact with penicillins, oral anticoagulants, and no other meds in the same IV fluid.

Trihexyphenydil

Anticholinergic Medication

-sartan

Angiotensin II Receptor Blockers (ARBS) Blocks Angiotensin II from having any effects on the heart and blood vessels. This keeps blood pressure from rising. tx: heart failure & hypertension

ending in: sartan

Angiotension II antagonists. Antihypertensive. Side effects include hypotension, dizziness, cough, GI upset, insomnia, nasal congestion, pharyngitis, myalgia/arthralgia, flu-like symptoms, angioedema, tachycardia & bradycardia. Nursing considerations: D/C asap if pregnancy is suspected. Use with caution in patients with renal/hepatic disease. Regularly assess renal function. Monitor potassium levels. Monitor BP & apical pulse often. Do not d/c abruptly. Use non-hormonal birth control options. Interact with potassium-sparing diuretics, salt subsititutes and red yeast rice.

Narcan

Antidote

Vitamin K

Antidote for Coumadin

Calcium Gluconate:

Antidote for magnesium sulfate

Acebutolol (Sectral)

Antidysrhythmics

Amiodarone Hydrochloride (Cordarone)

Antidysrhythmics

Diltiazem Hydrochloride (Cardizem)

Antidysrhythmics

Disopyramine (Norpace)

Antidysrhythmics

Esmolol Hydrochloride (Brevibloc)

Antidysrhythmics

Flecainide (Tambocor)

Antidysrhythmics

Ibutilide fumarate (Corvert)

Antidysrhythmics

Lidocaine

Antidysrhythmics

Mexiletine Hydrochloride (Mexitil)

Antidysrhythmics

Procainamide (Pronestyl)

Antidysrhythmics

Propafenone Hydrochloride (Rythmol)

Antidysrhythmics

ending in: vir

Antiviral. Contraindicated in preexisting hepatic/renal dysfunction, with concurrent use with nephrotoxic drugs, pregnancy & lactation. Side effects include anemia, h/a, mood changes, depression, seizures, diarrhea, local irritation/phlebtitis (with IV administration), inflammation (with topical admnistration), fever, metabolic acidosis, dysrhythmias, fluid overload, ocular hypertrophy, infertility, and pancreatitis. Nursing considerations: Administer as soon as possible to improve effectiveness. Wear gloves when administering topically. Preferred central venough access when given IV. Assess & monitor kidney/liver function. Assess skin and any lesions regularly. Hydrate to reduce risk of nephrotoxicity. Frequent, small meals in high in protein. Interact with nephrotoxic drugs, zidovudine, and probenecid.

Before administering a dose of an antiarrhythmic drug to an assigned patient, which assessments wold be of highest priority?

Apical pulse and blood pressure

In order to prevent the development of tolerance, the nurse instructs the patient to perform which action?

Apply the nitroglycerin patch for 14 hours each and remove for 10 hours at night

When applying nitroglycerin ointment, the nurse should perform which action?

Apply the ointment to a nofatty, nonhairy part of the upper torso

Truphylline

Bronchodilator

ANTIHYPERTENSIVES calcium channel blockers

C = cardio+ heart (same as B) Procardia (nifidipine), Calan (verapamil), cardizem (diltiazem) inhibits calcium across cell membrane, slow conduction uses: angina, HTN, interstitial cystitis (odd use) note-ipine! side effects: hypotension, GI stuff, AV blockm monitor for edema, pulses and BP important, SOB, remember: if you lower BP, pulse goes up, except if you also give meds to lower pulse as well. DO NOT CHEW < OPEN BROKEN time released

Chapter 21 1. It is important for the nurse teaching the client regarding secobarbital (Seconal) to include which information about secobarbital? a. It is a short-acting drug that may cause one to awaken early in the morning. b. It is an intermediate-acting drug that frequently causes REM rebound. c. It is an intermediate-acting drug that frequently causes a hangover effect. d. It is a long-acting drug that is frequently associated with dependence.

a. It is a short-acting drug that may cause one to awaken early in the morning.

beginning with: cef or ceph

Cephalosporins. Antibiotic (r/t penicillins). Contraindicated if allergy to penicillins. Side effects include lethargy, hallucinations, anxiety, depression, twitching/convulsions/seizures, coma, colitis, anemia, increased liver labs, increased bleeding time, alkalosis, taste alteration, sore mouth, discolored tongue, hives, pruritis, edema. Nursing considerations: Monitor for toxicities (renal, hepatic). Monitor BS in diabetics. Refrain from alcohol use during tx. Interact with loop diuretics, aminoglycosides, anticoagulants, ethanol, antacids, iron supplements/iron rich foods.

5. The nurse is aware of which fact regarding lorazepam (Ativan)? a. It may cause confusion and blurred vision. b. It has a maximum adult dose of 25 mg/day. c. When combined with cimetidine, it causes plasma levels to be decreased. d. It interferes with the binding of dopamine receptors.

a. It may cause confusion and blurred vision.

Neupogen

Colony Stimulating Factor

1. hypotension. 3. dizziness. 4. weakness

Common adverse effects of antidysrhythmic medications include: (Select all that apply.) 1. hypotension. 2. hypertension. 3. dizziness. 4. weakness 5. panic attacks

coumadin

an anticoagulant use to prevent and treat a thrombus or embolus

thorazine

an antipsychotic agen used in treating manic-depression and hallucination

mellaril

an antipsychotic agent used in treating psychotic and severe depression

The health teaching for a client with hypophosphatemia includes eating which foods? a. Meat, milk, whole grain cereals, nuts b. Dairy products, vitamin D supplements c. Dairy products, protein-rich foods d. Dairy products, nuts, vitamin c supplements

a. Meat, milk, whole grain cereals, nuts

Levofloxacin (Levoquin)

Fluoroquinolone

Norfloxacin (Noroxin)

Fluoroquinolone

Atropine

an antispasmodic that may be administered preoperattively to relax smooth muscles

GENITOURINARY INFECTION MEDICATIONS Furadantin (nitrofurantoin)

Furadantin anti infective UNIQUE side effects: asthma attacks, diarrhea give with food or milk monitor pulmonary status take with cranberry juice helps acidify urine do not eat alkaline foods almonds, milk, coconut and some veggies

Herplex

Eye Medication

Liquifilm Tears

Eye Medication

Ocu-Pred

Eye Medication

Ocufen

Eye Medication

Pontocaine

Eye Medication

Propine

Eye Medication

Stoxil

Eye Medication

Tearisol

Eye Medication

Timoptic

Eye Medication

Tobrex

Eye Medication

Visine, Murine Plus

Eye Medication

Treatment of hypoglycemic event

GlucaGen (glucagon), stimulates liver to change glycogen to glucose. GlucaGen side effects: hypotension, bronchospasm may repeat in 15 min once pt is alert, give carbohydrate orally to prevent secondary hypoglycemic reaction, side effect: coma When assessing consider: nutritional status, exercise status, stress, fevers, surgery, pregnancy etc. all affect metabolism

(Solu-Medrol)

Glucocorticoid

ending in: tidine

H2 antagonist. Antiulcer. Contraindicated in impaired renal/hepatic function. Side effects include somnolence, diaphoresis, rash, h/a, hypotension, taste disorder, diarrhea, constipation, dry mouth, cardiac dysrhythmias, impotence (with cimetidine), & anaphylaxis. Nursing considerations: Should not be given with other drugs via IV. Avoid antacids within 1h of administration. Reduced dosages for patients with renal/hepatic dysfunction. Evaluate need for tobacco/alcohol cessation. Cigarette smoking reduces drug effectiveness. Interact with beta blockers, lidocaine, benzodiazepines, TCAs, oral contraceptives, warfarin, iron salts, tetracyclines, diazepam, and glipizide.

ending in: parin

Heparin or heparinoid. Anticoagulant. Contraindicated in uncontrolled bleeding and in patients with pork allergies. Side effects include hemorrhage, hematuria, epistaxis, bleeding gums, thrombocytopenia, and osteoporosis (with long-term tx). Nursing considerations: Can be given IV or SUBQ. Monitor PTT when on heparin tx. Should be given via an infusion pump with a dedicated IV line due to its incompatibility with other drugs. Protamine sulfate is the antidote for heparin overdose. Monitor for signs of bleeding. Verify correct dosage with another licensed provider before administration. Rotate sites for SUBQ administration. Once patient is stabilized, an oral anticoagulant is added for long-term tx. Decrease intake of green leafy vegetables if adding warfarin to tx regimen. Interact with aspirin, NSAIDS, antiplatelets, and green leafy vegetables.

THROMBOPHLEBITIS

HARD & CORDLIKE VEIN HEAT, REDNESS, & TENDERNESS AT THE SITE IV INFUSION SLUGGISH

PHLEBITIS

HEAT, REDNESS, TENDERNESS AT THE SITE, NOY HARD OR SWOLLEN IV INFUSION SLUGGISH

Melatonin

Herbal Antianxiety Agent

A priority nursing diagnosis for a patient receiving anticholinergic (parasympatholytic) drugs would be

Impaired gas exchange related to thickened respiratory secretions

-pril

angiotensin converting enzyme (ACE) inhibitor, antihypertensive * captopril, lisinopril

Montelukast sodium (Singulair)

Leukotriene-Receptor Blocker

Dexedrine

Medications used for ADHD

Ritalin

Medications used for ADHD

Flomax

Men's Health Medications

Hytrin

Men's Health Medications

Levitra

Men's Health Medications

Proscar

Men's Health Medications

Viagra

Men's Health Medications

Vitamin D

Mineral

Magnesium Sulfate

Mineral & Electrolyte Replacement

Florinef

Mineralocorticoids

gastric protectant: an antisecretory med that enhances mucosal defenses & suppresses secretion of gastric acid & maintains submucosal blood flow by promoting vasodilation; used to prevent ulcers w/ NSAID use

Misoprostyl (cytotec)

Glucosamine

Musculoskeletal Medications

Neostigmine (Prostigmin)

Musculoskeletal Medications

Pyridostigmine bromide (Mestinon)

Musculoskeletal Medications

Atropine sulfate

Mydriatic and Cycloplegic Medication

Cyclopentolate (Cydogyl)

Mydriatic and Cycloplegic Medication

Darvon

Narcotic (Opioid Anagesic)

Demerol

Narcotic (Opioid Anagesic)

Dilaudid

Narcotic (Opioid Anagesic)

Dolophine

Narcotic (Opioid Anagesic)

Hydrocodone/ Acetaminophen (Vicodin)

Narcotic (Opioid Anagesic)

beginning or ending in: nitr

Nitrate. Contraindicated in hypotension and/or hypovolemia, severe bradycardia/tachycardia, and right ventricular MI. Side effects include h/a, postural hypotension, flushing, local burning or tingling sensation, contact dermatitis (topical administration), GI upset (oral administration), blurred vision, dry mouth, weakness, dizziness, vertigo, faintness, and fecal & urinary incontinence. Nursing considerations: Wear gloves when applying NTG paste or patch. Rotate application sites. Make sure patient is sitting or lying down when taking NTG to prevent dizziness or fainting. Give one tablet q5min (no more than 3 tablets total) and notify physician or emergency services if pain is unrelieved after 3rd dose. Store in a cool, dry, dark place with lid attached tightly. Patient may swim/bathe with a NTG patch in place. Take sublingual or spray NTG before events that cause angina (sex, exercise). Do not mix with any other meds in the bottle or IV tubing. Interact with Viagra, antihypertensives, vasodilators, heparin (if NTG is given IV), and alcohol.

A patient prescribed prazosin (Minipress) does not have a history of hypertension. The nurse would assess for what disorder for which this medication is also used?

Obstructive benign prostatic hyperplasia

ANTIDEPRESSANT Selected seratonin reuptake inhibitors (SSRI)

Prozac, Paxil, Zoloft Uses: depression, obsessive compulsion disorder, bulimia side effects: anxiety, GI upset, change in appetite and bowel function, urinary retention SUICIDE PRECAUTIONS takes 4 weeks for full effect take AM Monitor for suicide. Ask yes or no question: Do you have a plan? biggest concern 2-3-4 weeks into treatment Monitor nutrition and fluids will turn urine pink GI upset possible Side effect: sedation, anticholinergic effects, confusion, postural hypertension, urinary retention

ending in: cillin

Penicillin. Antibiotic. Contraindicated in anemia, thrombocytopenia, & bone marrow depression. Side effects include rash, urticaria, pruritis, angioedema, colitis, increased liver labs, taste alteration, sore mouth, sore/discolored tongue, increased bleeding time, lethargy, anxiety, depression, hallucinations, twitching/convulsions, coma, metabolic alkalosis, hypo/hyperkalemia. Nursing considerations: Do not admin with fruit juices, milk or carbonated beverages. Interact with diuretics, oral contraceptives, aminoglycides, tetracyclines, lithium, aspirin, & sulfonamides.

sulfa-

antibiotic; anti-infective; anti-inflammatory

A nurse prepares to administer a measles, mumps, and rubella (MMR) vaccine to a 5-year-old child. The nurse plans to administer this vaccine: 1. Intramuscularly in the anterolateral aspect of the thigh 2. Intramuscularly in the deltoid muscle 3. Subcutaneously in the outer aspect of the upper arm 4. Subcutaneously in the gluteal muscle

Rationale: MMR is administered subcutaneously in the outer aspect of the upper arm. Each child should receive two vaccinations, the first between 12 and 15 months of age and the second between 4 and 6 years or 11 and 12 years. **Knowledge that MMR is administered subcutaneously will assist in eliminating options 1 and 2. Knowing that the gluteal muscle is not incorporated in the subcutaneous tissue will eliminate option 4.**

182.) A client with angina pectoris is experiencing chest pain that radiates down the left arm. The nurse administers a sublingual nitroglycerin tablet to the client. The client's pain is unrelieved, and the nurse determines that the client needs another nitroglycerin tablet. Which of the following vital signs is most important for the nurse to check before administering the medication? 1. Temperature 2. Respirations 3. Blood pressure 4. Radial pulse rate

Rationale: Nitroglycerin acts directly on the smooth muscle of the blood vessels, causing relaxation and dilation. As a result, hypotension can occur. The nurse would check the client's blood pressure before administering the second nitroglycerin tablet. Although the respirations and apical pulse may be checked, these vital signs are not affected as a result of this medication. The temperature also is not associated with the administration of this medication.

beginning or ending with: sal

Salicylate. Nonopioid analgesic. Contraindicated in GI bleeding, bleeding disorders, younger than 12 years (due to risk of Reye's syndrome), children/teens with flu-like syndromes, pregnancy/lactation, with vitamin K deficiency, PUD, anemia, renal or hepatic dysfunction. Side effects include increased bleeding time, drowsiness, dizziness, confusion, h/a, hallucinations, heartburn, anorexia, rash, urticaria, bruising, tinnitus, seizures, coma, GI bleeding, hepatitis, and Reye's syndrome. Nursing considerations: Dosage dependent upon age of patient & condition being treated. GI upset may be decreased by giving with full glass of water, milk, food or an antacid. Administer at least 30 minutes prior to PT or planned activity to minimize discomfort. Report any signs/symptoms of hepatic/renal toxicity Report visual changes, tinnitus, allergic reactions, and bleeding. Do not exceed recommended dosage. Avoid alcohol to reduce chance of GI bleeding. Interact with antacids, urinary alkalizers, anticoagulants, alcohol, corticosteroids, warfarin, insulin, thrombolytic agents, penicillins, phenytoin, valproic acid, oral hypoglycemics, sulfonamides, ammonium chloride, urinary acidifiers, nizatidine, probenecid, spironolactone, NSAIDs, beta blockers, ginger, & caffeine.

Humalog R

Short-Acting Insulin

Iletin II Regular

Short-Acting Insulin

sulfa-

sulfonamide, antibiotic (check allergy to sulfa) *

2."Heparin does not thin the blood but prevents clots from forming as easily in the blood vessels." Rationale: Anticoagulants do not change the viscosity of the blood. Instead, anticoagulants exert a negative charge on the surface of the platelets, so that clumping or aggregation of cells is inhibited.

The patient receiving heparin therapy asks how the "blood thinner" works. The best response by the nurse would be: 1."Heparin makes the blood less thick." 2."Heparin does not thin the blood but prevents clots from forming as easily in the blood vessels." 3."Heparin decreases the number of platelets so that blood clots more slowly." 4."Heparin dissolves the clot."

-gliptin

antidiabetic; inhibitor of the DPP-4 enzyme

-glitazone

antidiabetic; thiazolidinedione

Loperamide is an

antidiarrheal med

-ylate

antidiarrheals (also immodium) adfx: anticholinergic, constipation cxn in unknown abd pain

6. For the client who is taking acetaminophen (Tylenol), what should the nurse do? (Select all that apply.) a. Monitor routine liver enzyme tests. b. Encourage the client to check package labels of OTC drugs to avoid overdosing. c. Teach the diabetic client taking acetaminophen to check blood glucose more frequently. d. Teach the female client that oral contraceptives can increase the effect of acetaminophen. e. Teach the client that caffeine decreases the effects of acetaminophen.

a. Monitor routine liver enzyme tests. b. Encourage the client to check package labels of OTC drugs to avoid overdosing. c. Teach the diabetic client taking acetaminophen to check blood glucose more frequently.

-azine

antiemetic

-nazole

antifungal

-azole

antifungal antimicrobial * flucanazole, miconazole

An older adult client comments, "It seems that all I do is take medicines" What does this comment reflect? a. That older adults consume 30% of all prescription medications b. That older adults may have multiple chronic conditions c. That older adults may take too many OTC preparations d. That older adults may take too many herbal preparations

a. That older adults consume 30% of all prescription medications

-iramine

antihistamine

-zine

antihistamine

-tadine

antihistamine; antiviral; anti-influenza-A

Viagra, Levitra, Cialis

Vasodilator side effect: headache, hypotension, priapism (painful erection that lasts 4 hours or more but is not accompanied by sexual excitement) CONTRAINDICATED: ALPHA BLOCKERS & NITRATES DO NOT TAKE GRAPEFRUIT JUICE ( also lipitor and anticonvulsant not to be taken with grapefruit juice)

-ide, -ine, plus( nitrates, minoxidil)

Vasodilators (relaxes blood vessels, & increases blood & O2 supply to heart, reducing heart's workload. tx: Angina

ANTIEMETICS page 84

Tigan, compazine, raglan, antivert, zofran. dramamine action: increases GI motility, block effect of dopamnie in chemoreceptor trigger zone side effect : sedation, anticholinergic Used before chemotherapy when used with viral infection may cause REY syndrome (toxic encepalopathy)

Nystatin cream (Mycostatin)

Topical Antibacterial

Povidone -iodine solution

Topical Antibacterial

Silver sulfadiazine cream (Silvadene)

Topical Antibacterial

Tolna flake cream (Tinactin)

Topical Antibacterial

c. Apply the nitroglycerin patch for 14 hours and remove it for 10 hours at night.

What will the nurse instruct the client to do to prevent the development of tolerance to nitroglycerin? a. Apply the nitroglycerin patch every other day. b. Switch to sublingual nitroglycerin when the client's systolic blood pressure elevates to more than 140 mm Hg. c. Apply the nitroglycerin patch for 14 hours and remove it for 10 hours at night. d. Use the nitroglycerin patch for acute episodes of angina only.

Sinequan

Tricyclics

Tofranil

Tricyclics

When teaching a patient regarding the administration of digoxin, the nurse instructs the patient not to take this medication with which food?

Wheat bran

-statin

antihyperlipidemia- statins block synthesis of cholesterol and triglycerides (sequestrans-Questran-blocks absorption) adfx: constipation, fat-soluble vitamin deficiency take hs or at least 30 minutes before meals. assess diet cannot take with other meds

One of the client's drugs has a potential adverse effect of nephrotoxicity. Which test is most accurate to determine renal function? a. Creatinine clearance b. Blood urea nitrogen c. Glomerular filtration rate d. Renal clearance

a. Creatinine clearance

2. The nurse acknowledges that the first-line drug for treating this client's blood pressure might be which drug?

a. Diuretic

ADRENERGICS

Used in Cardiac Arrest and COPD Action: stimulate beta 2 receptors in lungs (BETA 2 =LUNGS=2 lobes) restrict peripheral system, Causes bronchodilation Examples: Norepinephrine (Levophed), Dopamine (Intropine), Adrenaline (epinephrine), Doputamine (Dobutrex), Albuterol Side effects: dysrhythmias, tremors, anticholinergic effects, e.g.: DRY MOUTH AND URINARY RETENTION CNS: confusion, drowsy, inability to concentrate SAFTEY ISSUES (dont drive car, ambulation issues etc) Urinary bladder: retention and infection, CHECK OUTPUT Cardiovascular system: conduction issues (tachy cardia), exacerbation of angina, increased HR, MONITOR BP, MONITOR PERIPHERAL PULSES Salivary glands: decreased saliva, increased dental decay Gastrointestinal tract: constipation, reduced peristalsis Eye: inability to accomodate

Dobutamine hydrocholoride (Dobutrex)

Used to treat shock, cardiac arrest and anaphylaxis

Epinephrine

Used to treat shock, cardiac arrest and anaphylaxis

Isoproterenol

Used to treat shock, cardiac arrest and anaphylaxis

Milrinone (Primacor)

Used to treat shock, cardiac arrest and anaphylaxis

Norepinephrine

Used to treat shock, cardiac arrest and anaphylaxis

Phenylephrine (Neo-Synephrine)

Used to treat shock, cardiac arrest and anaphylaxis

a. Client states that she has no chest pain.

Which client assessment would assist the nurse in evaluating therapeutic effects of a calcium channel blocker? a. Client states that she has no chest pain. b. Client states that the swelling in her feet is reduced. c. Client states the she does not feel dizzy. d. Client states that she feels stronger.

d. "I will continue my exercise program to help increase my high-density lipoprotein serum levels."

Which statement made by the client indicates understanding about discharge instructions on antihyperlipidemic medications? a. "Antihyperlipidemic medications will replace the other interventions I have been doing to try to decrease my cholesterol." b. "It is important to double my dose if I miss one in order to maintain therapeutic blood levels." c. "I will stop taking the medication if it causes nausea and vomiting." d. "I will continue my exercise program to help increase my high-density lipoprotein serum levels."

Provera, Depo-Provera

Women's Health Medication

4. A client with multiple sclerosis is in the chronic progressive phase. The nurse should expect which drug to be most helpful at this time? a. Interferon β-1a (Avonex, Rebif) b. Glucocorticoids c. Azathioprine (Imuran) d. Cyclophosphamide (Cytoxan)

d. Cyclophosphamide (Cytoxan)

The client complains of anorexia, nausea, and vomiting. The client's list of medications include multiple large doses of vitamins. The nurse notes that the client's complaints may be related to early sign of toxicity of what vitamin? a. A b. B c. C d. D

d. D

The client refuses to take his prescribed medications. Which is the nurse's best response to this client? a. Explain the benefits and side effects of the drug b. Leave the medication at the patient's bedside to be taken later c. Persuade the client to take the medication d. Explain the risks of not taking the medication

d. Explain the risks of not taking the medication

patients who are receiving vancomycin by IV infusion should be assessed before administration and during for

hearing damage; this drug is both ototoxic and nephrotoxic

-mycin (micin)

aminoglycosides-inhibit protein synthesis gram neg adfx:oto and nephrotoxic check cranial nerve 8 (ear)

parnate

an MAOI usually usedin treating severe depression in patients who have failed to respond to other treatments

what labs are studied due to TPN causing excess amino acid intake will increase these levels

total protein, prealbumin, albumin, BUN/creatinine will all increase

cholinergic

treat myasthenia gravis, glaucoma, Alzheimer's disease and delayed gastric emptying

-tyline

tricyclic antidepressant

-pramine

tricyclic antidepressant (TCA)

-tyline

tricyclic antidepressant (TCA)

A child is ordered to receive naloxone (Narcan) IV, STAT. The child's weight is 20kg, and the recommended child's drug dosage is 0.01 mg/kg. Naloxone is available in 400 mcg/mL solution. The nurse should administer:

0.5 mL

Your patient is receiving dobutamine as a continuous infusion. Titration of this medication is based upon which factors? (Select all that apply.)

*Heart rate *Blood pressure *Urine output

Neostigmine

Antimyasthenics

Levodopa

Antiparkinson

Acyclovir (Zovirax)

Antiviral

Didanosine (Videx)

Antiviral

Anticonvulsants

used for the management of seizure disorder and treatment of bipolar disorder

Budesonide (Pulmicort)

Glucocorticoid

-vir

antiviral

Blood glucose

a test ordered for suspected diabetes

acetaminophen (Tylenol)

0.2-0.6 (>5 is toxic)

235.) A tricyclic antidepressant is administered to a client daily. The nurse plans to monitor for the common side effects of the medication and includes which of the following in the plan of care? 1. Offer hard candy or gum periodically. 2. Offer a nutritious snack between meals. 3. Monitor the blood pressure every 2 hours. 4. Review the white blood cell (WBC) count results daily.

1. Offer hard candy or gum periodically. Rationale: Dry mouth is a common side effect of tricyclic antidepressants. Frequent mouth rinsing with water, sucking on hard candy, and chewing gum will alleviate this common side effect. It is not necessary to monitor the blood pressure every 2 hours. In addition, it is not necessary to check the WBC daily. Weight gain is a common side effect and frequent snacks will aggravate this problem.

29.) A client is taking Humulin NPH insulin daily every morning. The nurse reinforces instructions for the client and tells the client that the most likely time for a hypoglycemic reaction to occur is: 1. 2 to 4 hours after administration 2. 4 to 12 hours after administration 3. 16 to 18 hours after administration 4. 18 to 24 hours after administration

2. 4 to 12 hours after administration Rationale: Humulin NPH is an intermediate-acting insulin. The onset of action is 1.5 hours, it peaks in 4 to 12 hours, and its duration of action is 24 hours. Hypoglycemic reactions most likely occur during peak time.

180.) A health care provider (HCP) writes a prescription for digoxin (Lanoxin), 0.25 mg daily. The nurse teaches the client about the medication and tells the client that it is important to: 1. Count the radial and carotid pulses every morning. 2. Check the blood pressure every morning and evening. 3. Stop taking the medication if the pulse is higher than 100 beats per minute. 4. Withhold the medication and call the HCP if the pulse is less than 60 beats per minute.

4. Withhold the medication and call the HCP if the pulse is less than 60 beats per minute. Rationale: An important component of taking this medication is monitoring the pulse rate; however, it is not necessary for the client to take both the radial and carotid pulses. It is not necessary for the client to check the blood pressure every morning and evening because the medication does not directly affect blood pressure. It is most important for the client to know the guidelines related to withholding the medication and calling the HCP. The client should not stop taking a medication.

Aluminum hydroxide gel (Amphojel)

Antacid Medication

Magnesium hydroxide (Milk of Magnesia)

Antacid Medication

Gentamicin

Anti-infective

Neomycin

Anti-infective

Plaquenil

Antimalaria & antirheumatic

Amantadine (Symmetrel)\

Antiviral

Famciclovir (Famvir)

Antiviral

Ganciclovir (Cytovene)

Antiviral

Oseltamivir (Tamiflu)

Antiviral

Cortisone acetate

Glucocorticoid

Dexamethasone (Decadron)

Glucocorticoid

ending in: statin

HMG-Coenzyme A (HMG-CoA) reductase inhibitors. Lipid lowering agent. Contraindicated in active liver disease and in pregnancy/lactation. Side effects include GI upset, dyspepsia, flatulence, pain, myalgias, h/a, rash, dizziness, sinusitis, & altered liver function tests. Nursing considerations: Usually administered at night. Not recommended for us in patients less than 20 years old. Lab monitoring is required is to maintain compliance and assess client response. Report immediately any unexplained muscle pain, tenderness, yellowing of skin/eyes, or loss of appetite. Alcohol intake should be minimized/avoided. Women of child-bearing age should use contraception during tx. Interact with immunosuppressants, antifungals, erythromycin, cholestyramine, and warfarin.

Tagamet

Medication for Ulcer

Zantac

Medication for Ulcer

Glucagon

Medication that Reverses Hypoglycemia

ANTHISTAMINE

Phenergan (promethazine HCl), Dioenhydramine HCl (Benadryl), lratidine, Zyrtec, Allegra, chlor-trimeton side effects: drowsiness, dry mouth, photosensitivity bronchospasm avoid alcohol give with food use sunscreen ASSES RESPIRATORY hard candy good implementation to deal with mouth care

The nurse formulates a plan of care for a patient receiving heparin based upon which nursing diagnosis?

Risk for injury

ELECTROLYTE OVERLOAD

SIGNS DEPEND ON THE SPECIFIC ELECTROLYTE OVERLOAD IMBALANCE

TISSUE DAMAGE

SKIN COLOR CHANGES, SLOUGHING OF THE SKIN DISCOMFORT AT THE SITE

Ultralente, Humulin U

SLOW ACTING onset 4 hrs peak 8-20 hrs Duration 24 - 36 hrs if given in the morning, late evening or morning might the time of concern

ANTICONVULSANTS pg 76

Side effects: respiratory depression, aplastic anemia, GINGIVAL HYPERTROPHY, ataxia DON'T discontinue abruptly Monito I&O's Caution with the use of medication that lower seizure threshold (work AGAINST this med: MAOI's - antidepressants, antipsychotics) TAKE WITH FOOD CNA depressant (alcohol bad!!! leads to respiratory depression) Drowsy (don't drive car, decision making, safety ) May turn urine PINK or BROWN) MEDS TO KNOW: (Phenytoin) Dilantin (lots of rules, read pg 76) if given IV too quickly causes cardiac arrest urine pink NEVER EVER MIX WITH ANY OTHER MED NEEDS IT'S OWN PORT INCREASE VITAMIN D MAGSULFATE used in OB Check DEEP TENDON REFLEXES RESPIRATORY ARREST SIDE EFFECT Depakote (Valproic Acid) DO NOT TAKE WITH CARBONATED BEVERAGES TAKE WITH FOOD

b. Crackles in the lungs

Which assessment finding will alert the nurse to possible toxic effects of amiodarone? a. Heart rate 100 beats per minute b. Crackles in the lungs c. Elevated blood urea nitrogen d. Decreased hemoglobin

A 97-year old client asks why a protein supplement has been prescribed. What Is the nurse's best response to the client? a. "you have increased circulation of free drug" b. "you have decreased hepatic size" c. "you have decreased calcium absorption" d. "you have increased motility"

a. "you have increased circulation of free drug"

1. When a newly admitted client is placed on heparin, the nurse acknowledges that heparin is effective for preventing new clot formation in clients who have which disorder(s)? (Select all that apply.)

a.Coronary thrombosis b.Acute myocardial infarction c.Deep vein thrombosis (DVT) d.Cerebrovascular accident (CVA) (stroke) e.Venous disorders

antimetabolites

anti-neoplastic-inhibit DNA polymerase fluorouracil (5-FU), Methotrexate used: ALL, CA of breast, colon, pancreas adfx: basic- GI-N/V, ulcers, decreased RBC, WBC, plt

CA -mycin (trade name)

anti-tumor antineoplastic-interfere with DNA and RNA synthesis. ex:Adriamycin use: CA adfx: basic chemo: stomatitis, alopecia, bone marrow suppression

Acyclovir:

anti-viral medication used in treating shingles.

Pyridium

antianelgesic URINARY TRACT PAIN REDUCTION UNIQUE: ORNAGE URINE side effect headache vertigo Teach: about color Should be taken with meals

cephalosporins

antibacterial

penicillins

antibacterial

parin; -parin

antithrombotic; anticoagulant (blood thinner)

2. For the client receiving pyridostigmine administration, the nurse should monitor for which adverse reaction? a. Hypertension b. Bronchospasm c. Thrombocytopenia d. Stevens-Johnson syndrome

b. Bronchospasm

2. The nurse teaches the client receiving atropine to expect which side effect? a. Diarrhea b. Bradycardia c. Blurred vision d. Frequent urination

c. Blurred vision

6. A client is taking triazolam (Halcion). Which instructions about this drug are important for the nurse to include? a. It may be used as a barbiturate for only 4 weeks. b. Use as a nonbenzodiazepine to reduce anxiety. c. This drug does not lead to vivid dreams or nightmares. d. Avoid alcohol and smoking to prevent rebound insomnia.

d. Avoid alcohol and smoking to prevent rebound insomnia.

The nurse reviews the client's medication regimen, including the interval of drug dosage, which is related to which of the following? a. Stimulation of receptors b. Trough level c. Therapeutic index d. Half-life

d. Half-life

The nurse is performing a health assessment on a newly admitted client who is of Asian descent. The client looks at the floor whenever the nurse asks a question. Communication is enhanced when the nurse does which action? a. Frequently touches the client b. Asks questions that only require "yes" or "no" for answers c. Discontinues the health assessment d. Uses eye contact sparingly

d. Uses eye contact sparingly

Chapter 28 1. A client is admitted with bipolar affective disorder. The nurse acknowledges that which medication is used to treat this disorder for some clients in place of lithium? a. thiopental b. gingko biloba c. fluvoxamine (Luvox) d. divalproex (Depakote)

d. divalproex (Depakote)

The client has been on TPN for 1 month, and there is an order to discontinue TPN tomorrow. The nurse contacts the health care provider because sudden interruption of TPN therapy may cause which condition? a. Dehydration b. Tremors c. Hyperglycemia d. hypoglycemia

d. hypoglycemia

6. Captopril (Capoten) has been ordered for a client. The nurse teaches the client that ACE inhibitors have which common side effects?

d.Constant, irritating cough

5. A client is taking lovastatin (Mevacor). Which serum level is most important for the nurse to monitor?

d.Liver enzymes

3. The nurse is assessing the client for possible evidence of digitalis toxicity. The nurse acknowledges that which is included in the signs and symptoms for digitalis toxicity?

d.Pulse below 60 beats/min and irregular rate

5. A client is prescribed dalteparin (Fragmin). LMWH is administered via which route?

d.Subcutaneously

8. Cilostazol (Pletal) is being prescribed for a client with coronary artery disease. The nurse knows that which is the major purpose for antiplatelet drug therapy?

d.To suppress platelet aggregation

AMINOCLYCOCIDES (antibiotics)

family: aminoglycocides examples: gentamycin, neomycin, streptomycin, tobramycin, amikacin action: inhibits protein synthesis in gram negative bacteria uses: pseudomonias, e coli, severe infections of CNS, respiratory, GI, urinary tract, bone, skin, soft tissue, acute pelvic inflammatory disease, TB MYACINS=MICE=Mice have big ear and ears look like kidneys OTOTOXICTY, NEPHROTOXCITY NV and diarrhea (due to given PO) check 8th cranial nerve (ringing in ears) Monitor Renal function, BUN IMPORTANT urinary output 1200ml/day normal Take for 7-10 days 3000ml/day look at peak (1 hr after dose given) and troughs (just before you hang a dose) encourage fluids

-trel

female hormone (progestin)

The doctor has ordered dopamine to treat your patient's hypovolemic shock due to severe blood loss. For the medication to be effective, the physician must also order

fluid replacement

-sone

glucocorticoids uses: asthma, COPD, some leukemias, Addison's, Crohn's adfx: infxn, mask s/s of infxn, insomnia, hypoglycemia, hypokalemia, psychosis, depression, stunted growth wean over time

ANTIDIABETICS

push client pancreas to produce more insulin used for type 2 (non-insulin diabetes mellitus) stimulate insulin release from the beta cells in pancreas Diabinese, Orinase, dymelor, micronase side effects: hypoglycemia allergic skin reaction, GI upset TAKE BEFORE BREAKFAST PO Even if GI upset...must eat once med is taken normal sugar 70-100 ml/dcl Alcohol will affect action of meds, oral birth control, sulfonamides, aspirin, MAOIs (hard med to coordinate with anything). All of these increase chance of a hypoglycemic reaction. TEACH family members what hypoglycemia looks like

Antacids

reduce hydrochloric acid in the stomach

Infliximab is an immunodulator that

reduces inflammation in Crohns disease, thereb reducing diarrhea

in malnourished clients watch for this of a drop in potassium, magnesium and phosphate

refeeding syndrome

the type of insulin used in an emergency situation is

regular

Narcotics/analgesics

relieve moderate to severe pain.

characteristics of mild CNS depression

slow in initiating conversation

Digoxin

slows heart side effects: bradycardia asystole nursing considerations:

Liver Impairment Nursing Considerations

small frequent feedings good skin care comfort measures rest

gastric protectant: creates a protective barrier against acid & pepsin

sucralfate (Carafate)

sulf, zulf, bactrim

sulfonamide-antagonizes folic acid synth UC, crohn's, uri adfx: peripheral neuropathy, photosensitive, crystalluria, stomatitis take with food, encourage fluids, mouth care

-cycline

tetracycline-prevents protein synthesis baby eye drops, acne adfx: discoloration of primary teeth in utero or in early childhood, phototoxic, toxic when expired take without food

A patient using Afrin nasal spray complains of worsening cold symptoms and tells the nurse, "I don't understand why this is not working. I am using it almost every 3 hours!" The nurse's response based on knowledge that

the patient is suffering from rebound congestion related to excessive use of the Afrin nasal spray

antiemetics

trimethobenzamide (Tigan), procorperazine (Compazine), metoclopramide (Reglan), meclizine (Antivert) block dopamine, increase GI motility adfx: anticholinergic, sedation prophylaxis for chemotherapy as well may cause Reye's used during viral infx

gantrinsin

used in urinary tract infections


Conjuntos de estudio relacionados

chapter 21 ap euro online assignment

View Set

дієслово як частина мови

View Set

Individual diff Emotional intelligence

View Set

Unit 7: Social Media and Marketing Part 2

View Set